Вы находитесь на странице: 1из 62

LABOR LAW

CONTINENTAL STEEL MANUFACTURING V. HON. ACCREDITED VOLUNTARY ARBITRATOR


MONTANO
G.R. NO. 182836
OCTOBER 13, 2009
J. CHICO-NAZARIO
CONSTRUCTION OF LABOR LAWS
FACTS:
Hortillano, an employee of petitioner Continental Steel and a member of respondent Union filed a claim for Paternity
Leave, Bereavement Leave and Death and Accident Insurance for dependent, pursuant to the CBA concluded between
Continental and the Union.
Section 2. BEREAVEMENT LEAVE--The Company agrees to grant a bereavement leave with pay to any
employee in case of death of the employee's legitimate dependent (parents, spouse, children, brothers and
sisters) based on the following: x x x
Section 4. DEATH AND ACCIDENT INSURANCE--The Company shall grant death and accidental
insurance
to
the
employee
or
his
family
in
the
following
manner:
x

4.3 DEPENDENTS--Eleven Thousand Five Hundred Fifty Pesos (Php11,550.00) in case of death of the
employees legitimate dependents (parents, spouse, and children). In case the employee is single, this
benefit covers the legitimate parents, brothers and sisters only with proper legal document to be presented
(e.g. death certificate).
The claim was based on the death of Hortillano's unborn child. Hortillano's wife had a premature delivery while she was
in the 38th week of pregnancy.
Continental Steel immediately granted Hortillano's claim for paternity leave but denied his claims for bereavement leave
and other death benefits, consisting of the death and accident insurance.
Upon grievance, the VA ruled that Hortillano was entitled to bereavement leave with pay and death benefits. The VA
found that there was no dispute that the death of an employee's legitimate dependent occurred. The fetus had the right to
be supported by the parents from the very moment he/she was conceived. The fetus had to rely on another for support;
he/she could not have existed or sustained himself/herself without the power or aid of someone else, specifically, his/her
mother. Therefore, the fetus was already a dependent, although he/she died during the labor or delivery.
Continental Steel claimed that the death of a fetus, at whatever stage of pregnancy, was excluded from the coverage of the
CBA since what was contemplated by the CBA was the death of a legal person, and not that of a fetus, which did not
acquire any juridical personality.
ISSUE:
Whether Hortillano was entitled to bereavement leave and other death benefits.
HELD:
Yes. When conflicting interests of labor and capital are to be weighed on the scales of social justice, the heavier influence
of the latter should be counter-balanced by sympathy and compassion the law must accord the underprivileged worker.
Any doubt concerning the rights of labor should be resolved in its favor pursuant to the social justice policy.
We emphasize that bereavement leave and other death benefits are granted to an employee to give aid to, and if possible,
lessen the grief of, the said employee and his family who suffered the loss of a loved one. It cannot be said that the
parents' grief and sense of loss arising from the death of their unborn child, who, in this case, had a gestational life of 3839 weeks but died during delivery, is any less than that of parents whose child was born alive but died subsequently.
Being for the benefit of the employee, CBA provisions on bereavement leave and other death benefits should be
interpreted liberally to give life to the intentions thereof. Time and again, the Labor Code is specific in enunciating that in
case of doubt in the interpretation of any law or provision affecting labor, such should be interpreted in favor of labor.
MIRANT PHILIPPINES CORPORATION V. CARO
G.R. NO. 181490
APRIL 23, 2014
1

J. VILLARAMA
CONSTRUCTION OF LABOR LAWS; MANAGEMENT PREROGATIVE; ILLEGAL DISMISSAL
FACTS:
Respondent filed a complaint for illegal dismissal and money claims for 13 th and 14th month pay, bonuses and other
benefits, as well as the payment of moral and exemplary damages and attorneys fees.
Respondent alleged that on January 3, 1994, he was hired by petitioner as its Logistics Officer and was assigned at
petitioners corporate office in Pasay City. At the time of the filing of the complaint, respondent was already a Supervisor
at the Logistics and Purchasing Department.
On November 3, 2004, petitioner conducted a random drug test where respondent was randomly chosen among its
employees who would be tested for illegal drug use. Respondent was duly notified that he was scheduled to be tested after
lunch on that day. His receipt of the notice was evidenced by his signature on the correspondence.
Respondent avers that at around 11:30 a.m. of the same day, he received a phone call from his wifes colleague who
informed him that a bombing incident occurred near his wifes work station in Tel Aviv, Israel where his wife was then
working as a caregiver. Respondent claims that after the said phone call, he proceeded to the Israeli Embassy to confirm
the news on the alleged bombing incident.
On that same day, at around 6:15 p.m., respondent returned to the office. When he was finally able to charge his cellphone
at the office, he received a text message from Cecilia, a member of the Drug Watch Committee that conducted the drug
test, informing him to participate in the said drug test.
On November 8, 2004, respondent received a Show Cause Notice from petitioner, requiring him to explain in writing why
he should not be charged with unjustified refusal to submit to random drug testing. Respondent submitted his written
explanation on November 11, 2004. Respondent requested for a hearing to explain that he could not submit proof that he
was indeed present at the Israeli Embassy during the said day because he was not allegedly allowed entry by the embassy
due to security reasons.
Petitioner found respondent guilty of unjustified refusal to submit to random drug testing. On February 14, 2005,
respondent received a letter from petitioner, terminating him on the same date. Respondent filed a Motion to Appeal his
termination on February 23, 2005. The motion was denied.
The LA found respondent to have been illegally dismissed. While petitioner observed the proper procedure in the
termination of an employee for a purported authorized cause, such just cause did not exist in the case at bar.
On appeal, the NLRC gave weight and emphasis to the inconsistencies in respondents explanations and considered his
omission as unjustified refusal in violation of petitioners drug policy. The NLRC found that respondent was not only
validly dismissed for cause he was also properly accorded his constitutional right to due process. The NLRC granted
financial assistance to respondent on equitable grounds.
The CA disagreed with the NLRC and ruled that it was immaterial whether respondent failed, refused, or avoided being
tested. To the appellate court, the singular fact material to this case was that respondent did not get himself tested in clear
disobedience of company instructions and policy. Despite such disobedience, however, the appellate court considered the
penalty of dismissal to be too harsh to be imposed on respondent. The CA, however, found that award of moral and
exemplary damages are without basis due to lack of bad faith on the part of the petitioner corporation which merely acted
within its management prerogative.
ISSUE:
Whether or not respondent was illegally dismissed.
HELD:
Yes. While the adoption and enforcement by petitioner of its Anti-Drugs Policy is recognized as a valid exercise of its
management prerogative as an employer, such exercise is not absolute and unbridled. Managerial prerogatives are subject
to limitations provided by law, collective bargaining agreements, and the general principles of fair play and justice. In the
exercise of its management prerogative, an employer must therefore ensure that the policies, rules and regulations on
work-related activities of the employees must always be fair and reasonable and the corresponding penalties, when
prescribed, commensurate to the offense involved and to the degree of the infraction. The Anti-Drugs Policy fell short of
being fair and reasonable.
First. The policy was not clear on what constitutes unjustified refusal when the subject drug policy prescribed that an
employees unjustified refusal to submit to a random drug test shall be punishable by the penalty of termination for the
2

first offense. The fact that petitioner corporations own personnel had to dissect the intended meaning of unjustified
refusal is further proof that it is not clear on what context the term unjustified refusal applies to. In Article 4 of the
Labor Code, as amended, all doubts in the implementation and interpretation of the provisions of the Labor Code,
including its implementing rules and regulations, shall be resolved in favor of labor. In Article 1702 of the New Civil
Code, a similar provision states that in case of doubt, all labor legislation and all labor contracts shall be construed in
favor of the safety and decent living for the laborer. Applying these provisions of law to the circumstances in the case at
bar, it is not fair for this Court to allow an ambiguous policy to prejudice the rights of an employee against illegal
dismissal.
Second. The penalty of termination imposed by petitioner upon respondent fell short of being reasonable. Company
policies and regulations are generally valid and binding between the employer and the employee unless shown to be
grossly oppressive or contrary to law. The unreasonableness of the penalty of termination as imposed in this case is further
highlighted by a fact admitted by petitioner itself: that for the ten-year period that respondent had been employed by
petitioner corporation, he did not have any record of a violation of its company policies.
BPI V. BPI EMPLOYEES UNION-DAVAO CHAPTER-FEDERATION OF UNIONS
G.R. NO. 164301
AUGUST 10, 2010
J. DE CASTRO
COVERAGE IN THE CBA; UNION SHOP CLAUSE
FACTS:
On March 23, 2000, the BSP approved the Articles of Merger between BPI and FEBTC. Pursuant to the Article and Plan
of Merger, all the assets and liabilities of FEBTC were transferred to and absorbed by BPI as the surviving corporation.
FEBTC employees, including those in its different branches across the country, were hired by petitioner as its own
employees, with their status and tenure recognized and salaries and benefits maintained.
Respondent BPI Employees Union-Davao Chapter - Federation of Unions in BPI Unibank is the exclusive bargaining
agent of BPI's rank and file employees in Davao City. The former FEBTC rank-and-file employees in Davao City did not
belong to any labor union at the time of the merger. Prior to the effectivity of the merger, respondent Union invited said
FEBTC employees to a meeting regarding the Union Shop Clause (Article II, Section 2) of the existing CBA between
petitioner BPI and respondent Union. The parties both advert to certain provisions of the existing CBA, which are quoted
below:
ARTICLE I
Section 1. Recognition and Bargaining Unit - The BANK recognizes the UNION as the sole and
exclusive collective bargaining representative of all the regular rank and file employees of the Bank
offices
in
Davao
City.
ARTICLE II
Section 1. Maintenance of Membership - All employees within the bargaining unit who are members of
the Union on the date of the effectivity of this Agreement as well as employees within the bargaining unit
who subsequently join or become members of the Union during the lifetime of this Agreement shall as a
condition of their continued employment with the Bank, maintain their membership in the Union in good
standing.
Section 2. Union Shop - New employees falling within the bargaining unit as defined in Article I of this
Agreement, who may hereafter be regularly employed by the Bank shall, within thirty (30) days after
they become regular employees, join the Union as a condition of their continued employment. It is
understood that membership in good standing in the Union is a condition of their continued employment
with the Bank.
After the meeting called by the Union, some of the former FEBTC employees joined the Union, while others refused.
Later, however, some of those who initially joined retracted their membership. Respondent Union then sent notices to the
former FEBTC employees who refused to join, as well as those who retracted their membership, and called them to a
hearing regarding the matter. When these former FEBTC employees refused to attend the hearing, the president of the
Union requested BPI to implement the Union Shop Clause of the CBA and to terminate their employment pursuant
thereto.
3

The issue remained unresolved in the grievance committee and so it was subsequently submitted for voluntary arbitration.
The VA ruled in favor of petitioner BPI's interpretation that the former FEBTC employees were not covered by the Union
Security Clause of the CBA between the Union and the Bank on the ground that the said employees were not new
employees who were hired and subsequently regularized, but were absorbed employees "by operation of law" because the
"former employees of FEBTC can be considered assets and liabilities of the absorbed corporation." The VA concluded
that the former FEBTC employees could not be compelled to join the Union, as it was their constitutional right to join or
not to join any organization.
The CA reversed and set aside the Decision of the VA.
Petitioner is of the position that the former FEBTC employees are not new employees of BPI for purposes of applying the
Union Shop Clause of the CBA because they became absorbed employees by operation of law. Petitioner argues that
the term "new employees" in the Union Shop Clause of the CBA is qualified by the phrases "who may hereafter be
regularly employed" and "after they become regular employees" which led petitioner to conclude that the "new
employees" referred to in, and contemplated by, the Union Shop Clause of the CBA were only those employees who were
"new" to BPI, on account of having been hired initially on a temporary or probationary status for possible regular
employment at some future date.
ISSUE:
(1) Distinguish Union Shop from Maintenance of Membership Shop and Closed Shop.
(2) Whether or not the former FEBTC employees that were absorbed by petitioner upon the merger between FEBTC and
BPI should be covered by the Union Shop Clause found in the existing CBA between petitioner and respondent Union.
HELD:
(1) Union Shop: There is union shop when all new regular employees are required to join the union within a certain
period for their continued employment.
Maintenance of Membership Shop: There is maintenance of membership shop when employees, who are union
members as of the effective date of the agreement, or who thereafter become members, must maintain union
membership as a condition for continued employment until they are promoted or transferred out of the bargaining
unit or the agreement is terminated.
Closed Shop: A closed-shop, on the other hand, may be defined as an enterprise in which, by agreement between
the employer and his employees or their representatives, no person may be employed in any or certain agreed
departments of the enterprise unless he or she is, becomes, and, for the duration of the agreement, remains a
member in good standing of a union entirely comprised of or of which the employees in interest are a part.
(2) Yes. We cannot agree with petitioners contention that the FEBTC employees were not considered as new
employees under the CBA because they were absorbed employees by operation of law. In the first place,
human beings are never embraced in the term assets and liabilities.
All employees in the bargaining unit covered by a Union Shop Clause in their CBA with management are subject
to its terms. However, under law and jurisprudence, the following kinds of employees are exempted from its
coverage, namely, employees who at the time the union shop agreement takes effect are bona fide members of a
religious organization which prohibits its members from joining labor unions on religious grounds; employees
already in the service and already members of a union other than the majority at the time the union shop
agreement took effect; confidential employees who are excluded from the rank and file bargaining unit; and
employees excluded from the union shop by express terms of the agreement.
Indeed, the situation of the former FEBTC employees in this case clearly does not fall within the first three
exceptions to the application of the Union Shop Clause discussed earlier. The sole category therefore in which
petitioner may prove its claim is the fourth recognized exception or whether the former FEBTC employees are
excluded by the express terms of the existing CBA between petitioner and respondents. Section 2, Article II of the
CBA is silent as to how one becomes a "regular employee" of the BPI for the first time. There is nothing in the
said provision which requires that a "new" regular employee first undergo a temporary or probationary
status before being deemed as such under the union shop clause of the CBA.
ABOSTA SHIP MANAGEMENT V. HILARIO
G.R. NO. 195792 NOVEMBER 24, 2014
CJ. SERENO
4

MANAGEMENT PREROGATIVE
FACTS:
On 24 October 2002, an employment contract was executed by petitioner, on behalf of its foreign principal Panstar
Shipping and respondent. In this contract, the latter was hired as a bosun (boatswain) of the foreign vessel Grand Mark for
a period of nine months, with a monthly salary of USD566. The contract was duly approved by the POEA.
On 27 November 2002, respondent was informed that the latter's deployment had been postponed. It appears, though, that
the foreign principal decided to promote an able seaman on board the vessel instead of hiring respondent. Petitioner thus
requested respondent to wait for another two to three months for a vacancy to occur.
Eventually, on 28 January 2003, respondent filed a Complaint with the POEA against petitioner for its failure to deploy
respondent within the prescribed period without any valid reason. Respondent likewise filed a Complaint with the Labor
Arbiter on 6 February 2003 based on the same ground and sought actual, moral and exemplary damages and attorney's
fees.
Petitioner moved for the dismissal of the Complaint, alleging that the Labor Arbiter had no jurisdiction over the matter, as
jurisdiction was supposedly lodged with the POEA. However, the Labor Arbiter denied the motion, stating that the action
for damages arising from employment relations was clearly within its jurisdiction.
The NLRC granted petitioner's appeal and reversed the Labor Arbiter's Order. Respondent consequently filed a Petition
for Certiorari with the CA questioning the ruling of the NLRC.
The CA granted the Petition and respondent's Complaint was reinstated. After the parties submitted their respective
Position Papers, the Labor Arbiter ordered petitioner to pay respondent his salary for nine months in the amount of USD
10,071. The Labor Arbiter found that the contract executed between the parties and the non-fulfillment thereof entitled
respondent to his salary for the whole duration of the contract. However, the arbiter did not find bad faith, which would
have merited the award of moral damages.
Upon appeal, the NLRC held that respondent's non-deployment was due to a valid exercise of the foreign principal's
management prerogative, which should be given due respect. Thus, the NLRC dismissed the Complaint, but ordered
petitioner "to comply with our directive to deploy respondent as soon as possible or face the inevitable consequences.
Dissatisfied with the NLRC's ruling, respondent filed a Petition for Certiorari with the CA.
The CA granted the Petition and held that the NLRC committed grave abuse of discretion by holding that the able
seaman's promotion was a valid management prerogative.
It is the contention of petitioner that respondent's non-deployment was due to the foreign principal's management
prerogative to promote an able seaman. Supposedly, this exercise of management prerogative is a valid and justifiable
reason that would negate any liability for damages.
ISSUE:
Whether the Court of Appeals committed reversible error when it disregarded the factual findings of the NLRC, that, if
properly considered, would justify petitioner's use of management prerogative.
HELD:
No. True, the promotion and choice of personnel is an exercise of management prerogative. However, there are limitations
on the exercise of management prerogatives, such as existing laws and the principle of equity and substantial justice.
Under the principle of equity and substantial justice, change of mind was not a valid reason for the non-deployment of
respondent.
There was an apparent violation of the contract at the time that the foreign principal decided to promote another person as
expressed in its communications dated 10 November 2002 and 14 November 2002. The vacancy for the position of
boatswain ceased to exist upon the execution of the contract between petitioner and respondent on 24 October 2002, a
contract subsequently approved by the POEA on 25 October 2002. The unilateral and unreasonable failure to deploy
respondent constitutes breach of contract, which gives rise to a liability to pay actual damages. The failure to deploy
respondent was an exercise of a management prerogative that went beyond its limits and resulted in a breach of contract.
In turn, petitioner's breach gave rise to respondent's cause of action to claim actual damages for the pecuniary loss
suffered by the latter in the form of the loss of nine months' worth of salary as provided in the POEA-approved contract of
employment.
PRINCESS JOY PLACEMENT AND GENERAL SERVICES V. BINALLA
G.R. NO. 197005
JUNE 4, 2014

J. BRION
ILLEGAL RECRUITMENT
FACTS:
On August 9, 2004, Binalla filed a complaint against local manning agent CBM and/or Princess Joy/Al Adwani General
Hospital for various money claims arising from his employment with Al Adwani, in Taif, Saudi Arabia from April 19,
2002 to April 28, 2004.
Binalla, a registered nurse, alleged that in April 2002, he applied for employment with Princess Joy who referred him to
Reginaldo Paguio and Cynthia Latea for processing of his papers. On April 12, 2002, he signed a four-year contract with
Al Adwani as staff nurse. He paid Latea P4,500.00 and Paguio, P3,000.00, although no receipts were issued to him.
Later, he was given a telegram notifying him of his departure on April 19, 2002.
Binalla further alleged that on the day of his departure, Paguio met him at the airport and gave him a copy of his
employment contract, plane ticket, passport, a copy of his Overseas Employment Certificate from the POEA and other
documents. It was only after boarding his Saudi Arabia Airlines plane that he examined his papers and discovered that
CBM was his deploying agency. Under the contract certified by the POEA, his salary was supposed to be US$550.00 for
24 months.
Binalla also saw that under the four-year contract he signed, his monthly salary was only $400. Left with no choice as he
was then already bound for Saudi Arabia, he worked under his contract for only two years and returned to the Philippines
in April 2004 after posting a bond of SR 3,000.00, supposedly to guarantee that he would come back to finish his contract.
Upon his return to the Philippines, Binalla verified his employment contract with the POEA. He learned that the POEA
indeed certified a different contract for him, with CBM as his recruiting or deploying agency. He disowned the contract,
claiming that his supposed signature appearing in the document was a forgery. Binalla argued before the labor arbiter that
he was re-processed an arrangement where Princess Joy recruited and deployed him, but made it appear that it was
undertaken by CBM under a different contract submitted to and certified by the POEA.
Princess Joy denied that it recruited and deployed Binalla for overseas employment, repudiating the involvement of
Paguio and Lateo in Binallas engagement by Al Adwani.
The LA considered the complaint a monetary claim and therefore under his jurisdiction under the law. The LA found that
Princess Joy and CBM jointly undertook Binallas recruitment and deployment in Saudi Arabia through reprocessing.
The LA declared CBM and Princess Joy jointly and severally liable to pay Binalla. Princess Joy appealed the LAs ruling.
The NLRC deleted the award to Binalla of salary differentials, food allowance, and moral and exemplary damages. It
awarded Binalla $2,200.00 in unpaid salaries for four months; $550.00 for unused vacation and sick leave credits, plus
10% attorneys fees. In the petition for review filed before the CA, the appellate court granted the petition and set aside
the NLRC rulings.
ISSUE:
Whether the NLRC acted in grave abuse of discretion in ignoring the presence of substantial evidence in the records
which indicated that Princess Joy is as responsible and liable as CBM.
HELD:
Yes. Under Article 34 (i) of the Labor Code on prohibited practices, it shall be unlawful for any individual, entity,
licensee, or holder of authority to substitute or alter employment contracts approved and verified by the Department of
Labor and Employment from the time of actual signing thereof by the parties up to and including the periods of expiration
of the same without the approval of the Secretary of Labor. Further, contract substitution constitutes illegal
recruitment under Article 38 (I) of the Code.
Under the circumstances, Princess Joy is as liable as CBM and Al Adwani for the contract substitution, no matter how it
tries to avoid liability by disclaiming any participation in the recruitment and deployment of Binalla to Al Adwani. Before
the LA, Princess Joy claimed that Paguio and Lateo were not its employees/representatives or that the principal piece of
evidence relied upon by the labor arbiter, the ticket/telegram/advise handed to Binalla by Paguio had no probative value
as it was merely an unsigned and unauthenticated printout or that the four-year employment contract was signed only by
Binalla and there was no showing that it was the contract implemented by Al Adwani.
Princess Joys protestations fail to convince us. We believe, as the labor arbiter did, that the ticket telegram/advice is
proof enough that Princess Joy recruited Binalla. Binalla was able to hold onto the ticket/telegram/advise handed to him
by Paguio which carried the names PRINCESS JOY and REGIE/ It would not be an unreasonable presumption that
Princess Joy recruited Binalla and that the latter had been transacting with Paguio (Regie).
6

TABANGAO SHELL REFINERY EMPLOYEES V. PILIPINAS SHELL


G.R. NO. 157485
MARCH 26, 2014
J. DE CASTRO
ASSUMPTION OF JURISDICTION
FACTS:
In anticipation of the expiration on April 30, 2004 of the 2001-2004 Collective Bargaining Agreement (CBA) between the
petitioner and the respondent Pilipinas Shell Petroleum Corporation, the parties started negotiations for a new CBA.
On the parties 41st meeting held on September 2, 2004, the company proposed the declaration of a deadlock and
recommended that the help of a third party be sought. The union filed a Notice of Strike in the National Conciliation and
Mediation Board (NCMB), alleging bad faith bargaining on the part of the company.
The company filed a Petition for Assumption of Jurisdiction with the Secretary of Labor and Employment. The Secretary
of Labor and Employment took notice of the Notice of Strike filed by the union in the NCMB which charged the company
with unfair labor practice consisting of bad faith in bargaining negotiations. The Secretary of Labor and Employment also
found that the intended strike would likely affect the companys capacity to provide petroleum products to the companys
various clientele, including the transportation sector, the energy sector, and the manufacturing and industrial sectors. Thus,
the Secretary of Labor and Employment assumed jurisdiction over the dispute of the parties.
The union thereafter filed a petition for certiorari alleging in its petition that the Secretary of Labor and Employment
acted with grave abuse of discretion in grossly misappreciating the facts and issue of the case. It contended that the issue
is the unfair labor practice of the company in the form of bad faith bargaining and not the CBA deadlock.
The Court of Appeals dismissed the petition for certiorari of the union. Article 263(g) of the Labor Code vests in the
Secretary of Labor and Employment not only the discretion to determine what industries are indispensable to national
interest but also the power to assume jurisdiction over such industries labor disputes, including all questions and
controversies arising from the said disputes.
ISSUE:
Whether or not the Secretary of Labor correctly assumed jurisdiction over the labor dispute between the parties.
HELD:
YES. The Secretary of the DOLE has been explicitly granted by Article 263(g) of the Labor Code the authority to assume
jurisdiction over a labor dispute causing or likely to cause a strike or lockout in an industry indispensable to the national
interest, and decide the same accordingly. And, as a matter of necessity, it includes questions incidental to the labor
dispute; that is, issues that are necessarily involved in the dispute itself, and not just to that ascribed in the Notice of Strike
or otherwise submitted to him for resolution.
The power of the Secretary of Labor and Employment to assume jurisdiction over this dispute includes and extends to all
questions and controversies arising from the said dispute, such as, but not limited to the unions allegation of bad faith
bargaining. It also includes and extends to the various unresolved provisions of the new CBA such as compensation,
particularly the matter of annual wage increase or yearly lump sum payment in lieu of such wage increase, whether or not
there was deadlock in the negotiations.
As the term assume jurisdiction connotes, the intent of the law is to give the Labor Secretary full authority to resolve all
matters within the dispute that gave rise to or which arose out of the strike or lockout; it includes and extends to all
questions and controversies arising from or related to the dispute, including cases over which the labor arbiter has
exclusive jurisdiction.
INDOPHIL TEXTILE MILLS V. ADVIENTO
G.R. NO. 171212
AUGUST 20, 2014
J. PERALTA
JURISDICTION
FACTS:

Petitioner Indophil hired respondent Engr. Adviento as Civil Engineer to maintain its facilities in Lambakin, Marilao,
Bulacan. On August 7, 2002, respondent consulted a physician due to recurring weakness and dizziness. Few days later,
he was diagnosed with Chronic Poly Sinusitis, and thereafter, with moderate, severe and persistent Allergic Rhinitis.
Accordingly, respondent was advised by his doctor to totally avoid house dust mite and textile dust as it will transmute
into health problems.
Distressed, respondent filed a complaint against petitioner with the NLRC for alleged illegal dismissal and for the payment
of backwages, separation pay, actual damages and attorneys fees. Subsequently, respondent filed another Complaint with
the RTC, alleging that he contracted such occupational disease by reason of the gross negligence of petitioner to provide
him with a safe, healthy and workable environment.
In his Complaint, respondent alleged that as part of his job description, he conducts regular maintenance check on
petitioners facilities including its dye house area, which is very hot and emits foul chemical odor with no adequate safety
measures introduced by petitioner. According to respondent, these health hazards have been the persistent complaints of
most, if not all, workers of petitioner. Nevertheless, said complaints fell on deaf ears as petitioner callously ignored the
health problems of its workers and even tended to be apathetic to their plight, including respondent. Respondent averred
that he made several attempts to apply for a new job, but to his dismay and frustration, employers who knew of his present
health condition discriminated against him and turned down his application.
In reply, petitioner filed a Motion to Dismiss on the ground that the RTC has no jurisdiction over the subject matter of the
complaint because the same falls under the original and exclusive jurisdiction of the LA under Article 217(a)(4) of the
Labor Code. The RTC issued a Resolution denying the aforesaid Motion and sustaining its jurisdiction over the instant
case. It held that petitioners alleged failure to provide its employees with a safe, healthy and workable environment is an
act of negligence, a case of quasi-delict. As such, it is not within the jurisdiction of the LA under Article 217 of the Labor
Code.
ISSUE:
Whether the RTC has jurisdiction over the subject matter of respondents complaint.
HELD:
Yes, ART. 217. Jurisdiction of Labor Arbiters and the Commission -- (a) Except as otherwise provided under this Code
the Labor Arbiter shall have original and exclusive jurisdiction to hear and decide, within thirty (30) calendar days after
the submission of the case by the parties for decision without extension, even in the absence of stenographic notes, the
following cases involving all workers, whether agricultural or non-agricultural:
1. Unfair labor practice cases;
2. Termination disputes;
3. If accompanied with a claim for reinstatement, those cases that workers may file involving wages,
rates of pay, hours of work and other terms and conditions of employment;
4. Claims for actual, moral, exemplary and other forms of damages arising from employer-employee
relations;
5. Cases arising from any violation of Article 264 of this Code including questions involving the
legality of strikes and lockouts; and
6. Except claims for Employees Compensation, Social Security, Medicare and maternity benefits, all
other claims, arising from employer-employee relations, including those of persons in domestic or
household service, involving an amount exceeding five thousand pesos (P5,000.00) regardless of
whether accompanied with a claim for reinstatement.
Indeed, jurisprudence has evolved the rule that claims for damages under Article 217(a)(4) of the Labor Code, to be
cognizable by the LA, must have a reasonable causal connection with any of the claims provided for in that article. Only if
there is such a connection with the other claims can a claim for damages be considered as arising from employeremployee relations. In the case at bench, there is no reasonable connection.
True, the maintenance of a safe and healthy workplace is ordinarily a subject of labor cases. More, the acts complained of
appear to constitute matters involving employee-employer relations since respondent used to be the Civil Engineer of
petitioner. However, it should be stressed that respondents claim for damages is specifically grounded on petitioners
gross negligence to provide a safe, healthy and workable environment for its employees - a case of quasi-delict.
In the case at bar, respondent alleges that due to the continued and prolonged exposure to textile dust seriously inimical to
his health, he suffered work-contracted disease which is now irreversible and incurable, and deprived him of job
opportunities. Clearly, injury and damages were allegedly suffered by respondent, an element of quasi-delict. Secondly,
the previous contract of employment between petitioner and respondent cannot be used to counter the element of no preexisting contractual relation since petitioners alleged gross negligence in maintaining a hazardous work environment
8

cannot be considered a mere breach of such contract of employment, but falls squarely within the elements of quasi-delict
under Article 2176 of the Civil Code since the negligence is direct, substantive and independent.
AMECOS INNOVATIONS V. LOPEZ
G.R. NO. 178055
JULY 2, 2014
J. DEL CASTILLO
JURISDICTION
FACTS:
Amecos received a Subpoena from the Office of the City Prosecutor in connection with a complaint filed by the SSS for
alleged delinquency in the remittance of SSS contributions and penalty liabilities. By way of explanation, Amecos
attributed its failure to remit the SSS contributions to herein respondent Lopez. Amecos claimed that it hired respondent
on January 15, 2001 as Marketing Assistant. Upon hiring, respondent refused to provide Amecos with her SSS Number
and to be deducted her contributions and so, Amecos no longer enrolled respondent with the SSS and did not deduct her
corresponding contributions up to the time of her termination in February 2002.
Amecos eventually settled its obligations with the SSS; consequently, SSS filed a Motion to Withdraw Complaint.
Thereafter, Amecos sent a demand letter to respondent for P27,791.65 representing her share in the SSS contributions and
expenses for processing, but to no avail. Thus, petitioners filed a Complaint for sum of money and damages against
respondent before the MeTC.
Respondent filed her Answer with Motion to Dismiss claiming that Amecos deliberately failed to deduct and remit her
SSS contributions; and that petitioners filed the Complaint in retaliation to her filing of an illegal dismissal case.
Respondent averred that the regular courts do not have jurisdiction over the instant case as it arose out of their employeremployee relationship.
The MeTC dismissed the case for lack of jurisdiction. The RTC affirmed the view taken by the MeTC that under Article
217(a)(4) of the Labor Code, claims for actual, moral, exemplary and other forms of damages arising from employeremployee relationship are under the jurisdiction of the Labor Arbiters or the NLRC; that since petitioners and respondent
were in an employer-employee relationship at the time, the matter of SSS contributions was thus an integral part of that
relationship; and as a result, petitioners cause of action for recovery of damages from respondent falls under the
jurisdiction of the Labor Arbiters, pursuant to Article 217(a)(4) of the Labor Code.
ISSUE:
(1) Whether the regular civil court has jurisdiction over claims for reimbursement and damages for misrepresentation
arising from employer-employee relations.
(2) Whether the dispute should be referred to the Social Security Commission.
HELD:
(1) No. As between the parties, Article 217(a)(4) of the Labor Code is applicable. Said provision bestows upon the
Labor Arbiter original and exclusive jurisdiction over claims for damages arising from employer-employee
relations. The observation that the matter of SSS contributions necessarily flowed from the employer-employee
relationship between the parties shared by the lower courts and the CA is correct; thus, petitioners claims
should have been referred to the labor tribunals. In this connection, it is noteworthy to state that the Labor
Arbiter has jurisdiction to award not only the reliefs provided by labor laws, but also damages governed by the
Civil Code.
(2) No. As far as SSS is concerned, there is no longer a dispute with respect to petitioners accountability to the
System; petitioners already settled their pecuniary obligations to it. Since there is no longer any dispute regarding
coverage, benefits, contributions and penalties to speak of, the SSC need not be unnecessarily dragged into the
picture.
Remedial Law issue: Petitioner has no cause of action against respondent. By the time Amecos was forced to pay,
respondent no longer worked for them. In fine, respondent was ever covered by the System. Hence, there is no sense in
making her answerable for the required contributions during the period of her employment. It follows that claims for
damages founded on the foregoing non-existent cause of action should likewise fail.
COSARE V. BROADCOM ASIA
G.R. NO. 201298
FEBRUARY 5, 2014
9

J. REYES
JURISDICTION
FACTS:
The case stems from a complaint for constructive dismissal, illegal suspension and monetary claims filed with the NLRC
by Cosare against the respondents. In December 2000, Arevalo set up the company Broadcom, still to continue the
business of trading communication and broadcast equipment. Cosare was named an incorporator of Broadcom, having
been assigned 100 shares of stock with par value of P1.00 per share. In October 2001, Cosare was promoted to the
position of AVP for Sales and Head of the Technical Coordination.
Sometime in 2003, Abiog was appointed as Broadcoms Vice President for Sales and thus, became Cosares immediate
superior. On March 23, 2009, Cosare sent a confidential memo to Arevalo to inform him of the following anomalies
which were allegedly being committed by Abiog against the company. Arevalo failed to act on Cosares accusations.
Cosare claimed that he was instead called for a meeting by Arevalo on March 25, 2009, wherein he was asked to tender
his resignation in exchange for financial assistance in the amount of P300,000.00.
On March 30, 2009, Cosare received from Villareal, Broadcoms Manager for Finance and Administration, a memo signed
by Arevalo, charging him of serious misconduct and willful breach of trust. Cosare was given 48 hours to present his
explanation on the charges. He was also suspended from having access to any and all company files/records and use of
company assets effective immediately. Thus, Cosare claimed that he was precluded from reporting for work on March
31, 2009, and was instead instructed to wait at the offices receiving section. On April 1, 2009, Cosare was totally barred
from entering the company premises, and was told to merely wait outside the office building for further instructions. The
respondents refused to receive the memo on the ground of late filing, prompting Cosare to serve a copy thereof by
registered mail. The following day, April 3, 2009, Cosare filed the subject labor complaint, claiming that he was
constructively dismissed from employment by the respondents.
On January 6, 2010, LA Menese rendered his Decision dismissing the complaint on the ground of Cosares failure to
establish that he was dismissed, constructively or otherwise, from his employment. Cosare appealed the LA decision to
the NLRC which reversed the Decision of LA Menese. In a manifestation filed by the respondents during the pendency of
the CA appeal, they raised a new argument, i.e., the case involved an intra-corporate controversy which was within the
jurisdiction of the RTC, instead of the LA. They argued that the case involved a complaint against a corporation filed by a
stockholder, who, at the same time, was a corporate officer.
The CA rendered the assailed Decision granting the respondents petition. Thus, the CA reversed the NLRC decision and
resolution, and then entered a new one dismissing the labor complaint on the ground of lack of jurisdiction.
ISSUE:
Whether the regular court or the LA has jurisdiction over the instant case.
HELD:
The LA has jurisdiction over the instant case.
Settled jurisprudence, however, qualifies that when the dispute involves a charge of illegal dismissal, the action may fall
under the jurisdiction of the LAs upon whose jurisdiction, as a rule, falls termination disputes and claims for damages
arising from employer-employee relations as provided in Article 217 of the Labor Code. Consistent with this
jurisprudence, the mere fact that Cosare was a stockholder and an officer of Broadcom at the time the subject
controversy developed failed to necessarily make the case an intra-corporate dispute.
There are two circumstances which must concur in order for an individual to be considered a corporate officer, as against
an ordinary employee or officer, namely: (1) the creation of the position is under the corporations charter or by-laws; and
(2) the election of the officer is by the directors or stockholders. It is only when the officer claiming to have been illegally
dismissed is classified as such corporate officer that the issue is deemed an intra-corporate dispute which falls within the
jurisdiction of the trial courts. The Court disagrees with the respondents and the CA. The only officers who are
specifically listed, and thus with offices that are created under Broadcoms by-laws are the following: the President, VicePresident, Treasurer and Secretary. Although a blanket authority provides for the Boards appointment of such other
officers as it may deem necessary and proper, the respondents failed to sufficiently establish that the position of AVP
for Sales was created by virtue of an act of Broadcoms board, and that Cosare was specifically elected or
appointed to such position by the directors. No board resolutions to establish such facts form part of the case records.
Considering that the pending dispute particularly relates to Cosares rights and obligations as a regular officer of
Broadcom, instead of as a stockholder of the corporation, the controversy cannot be deemed intra-corporate.
10

MILAN V. NLRC
G.R. NO. 202961

FEBRUARY 4, 2015

J. LEONEN
JURISDICTION; WITHHOLDING OF TERMINAL PAY AND BENEFITS PENDING RETURN OF PROPERTY
FACTS:
Petitioners are Solid Mills employees, represented by the National Federation of Labor Unions (NAFLU), their collective
bargaining agent. As Solid Mills employees, petitioners and their families were allowed to occupy SMI Village, a
property owned by Solid Mills.
Petitioners were informed that Solid Mills would cease its operations due to serious business losses. NAFLU recognized
Solid Mills closure due to serious business losses in the memorandum of agreement dated September 1, 2003. The
memorandum of agreement provided for Solid Mills grant of separation pay less accountabilities, accrued sick leave
benefits, vacation leave benefits, and 13th month pay to the employees. Solid Mills filed its DOLE termination report on
September 2, 2003. Later, Solid Mills sent to petitioners individual notices to vacate SMI Village.
Petitioners were no longer allowed to report for work by October 10, 2003. They were required to sign a memorandum of
agreement with release and quitclaim before their vacation and sick leave benefits, 13th month pay, and separation pay
would be released. Employees who signed the memorandum of agreement were considered to have agreed to vacate SMI
Village, and to the demolition of the constructed houses inside as condition for the release of their termination benefits
and separation pay. Petitioners refused to sign the documents and demanded to be paid their benefits and separation pay.
Hence, petitioners filed complaints before the LA for alleged non-payment of separation pay, accrued sick and vacation
leaves, and 13th month pay. Their possession of Solid Mills property is not an accountability that is subject to clearance
procedures. They had already turned over to Solid Mills their uniforms and equipment when Solid Mills ceased
operations. On the other hand, Solid Mills argued that petitioners complaint was premature because they had not vacated
its property.
The LA ruled in favor of petitioners. Upon appeal, the NLRC ruled that because of petitioners failure to vacate Solid
Mills property, Solid Mills was justified in withholding their benefits and separation pay. Petitioners filed a motion for
partial reconsideration but this was denied. Petitioners, thus, filed a petition for certiorari before the CA but the same was
dismissed. The CA ruled that Solid Mills act of allowing its employees to make temporary dwellings in its property was a
liberality on its part. It may be revoked any time at its discretion. As a consequence of Solid Mills closure and the
resulting termination of petitioners, the employer-employee relationship between them ceased to exist. There was no
more reason for them to stay in Solid Mills property. The CA agreed with the NLRCs deletion of interest since it found
that Solid Mills act of withholding payment of benefits and separation pay was proper. Petitioners terminal benefits and
pay were withheld because of petitioners failure to vacate Solid Mills property.
In its petition for review, petitioners argue that respondent Solid Mills and NAFLUs memorandum of agreement has no
provision stating that benefits shall be paid only upon return of the possession of respondent Solid Mills
property.Accountabilities should be interpreted to refer only to accountabilities that were incurred by petitioners while
they were performing their duties as employees at the worksite. Petitioners also point out that the NLRC and the CA have
no jurisdiction to declare that petitioners act of withholding possession of respondent Solid Mills property is illegal. The
regular courts have jurisdiction over this issue. It is independent from the issue of payment of petitioners monetary
benefits.
ISSUE:
(1) Whether the NLRC has jurisdiction to declare that petitioners act of withholding possession of Solid Mills property
is illegal.
(2) Whether or not the CA committed reversible error when it ruled that payment of the monetary claims of petitioners
should be held in abeyance pending compliance of their accountabilities to respondent solid mills by turning over the
subject lots they respectively occupy.
HELD:
(1) Yes. The NLRC has jurisdiction to determine, preliminarily, the parties rights over a property, when it is
necessary to determine an issue related to rights or claims arising from an employer-employee relationship.
Claims arising from an employer-employee relationship are not limited to claims by an employee. Employers
may also have claims against the employee, which arise from the same relationship.
In this case, respondent Solid Mills claims that its properties are in petitioners possession by virtue of their status
as its employees. Respondent Solid Mills allowed petitioners to use its property as an act of liberality. Put in
11

other words, it would not have allowed petitioners to use its property had they not been its employees. The return
of its properties in petitioners possession by virtue of their status as employees is an issue that must be resolved
to determine whether benefits can be released immediately. The issue raised by the employer is, therefore,
connected to petitioners claim for benefits and is sufficiently intertwined with the parties employer-employee
relationship. Thus, it is properly within the labor tribunals jurisdiction.
(2) No. As a general rule, employers are prohibited from withholding wages from employees. The Labor Code
provides:
Art. 116. Withholding of wages and kickbacks prohibited. It shall be unlawful for any person,
directly or indirectly, to withhold any amount from the wages of a worker or induce him to give up
any part of his wages by force, stealth, intimidation, threat or by any other means whatsoever without
the workers consent.
As an exception to the general rule that wages may not be withheld and benefits may not be diminished, the Labor
Code provides:
Art. 113. Wage deduction. No employer, in his own behalf or in behalf of any person, shall make
any
deduction
from
the
wages
of
his
employees,
except:
1. In cases where the worker is insured with his consent by the employer, and the deduction is to
recompense the employer for the amount paid by him as premium on the insurance;
2. For union dues, in cases where the right of the worker or his union to check-off has been
recognized by the employer or authorized in writing by the individual worker concerned; and
3. In cases where the employer is authorized by law or regulations issued by the Secretary of Labor
and Employment.
The Civil Code provides that the employer is authorized to withhold wages for debts due: Debt in this case refers to any
obligation due from the employee to the employer. It includes any accountability that the employee may have to the
employer. There is no reason to limit its scope to uniforms and equipment, as petitioners would argue. Accountability, in
its ordinary sense, means obligation or debt. The ordinary meaning of the term accountability does not limit the
definition of accountability to those incurred in the worksite. As long as the debt or obligation was incurred by virtue of
the employer-employee relationship, generally, it shall be included in the employees accountabilities that are subject to
clearance procedures. Accountabilities of employees are personal. They need not be uniform among all employees in
order to be included in accountabilities incurred by virtue of an employer-employee relationship.
The return of the propertys possession became an obligation or liability on the part of the employees when the employeremployee relationship ceased. Thus, respondent Solid Mills has the right to withhold petitioners wages and benefits
because of this existing debt or liability.
REPUBLIC (DOLE) V. NAMBOKU PEAK
G.R. NO. 169745
JULY 18, 2014
J. DEL CASTILLO
REAL PARTY I INTEREST TO FILE APPEAL
FACTS:
These are two separate petitions which were consolidated because they involve a similar issue.
First case: Namboku is a domestic corporation engaged in the business of providing manpower services to various
clients, mainly airline companies. The Philippine Aircraft Loaders and Cargo Employees Association-Solidarity of Unions
in the Philippines for Empowerment and Reforms (PALCEA-SUPER) filed a Petition for direct certification election
before the Med-Arbiter seeking to represent the rank-and-file employees of Namboku assigned at the Cargo and Loading
Station of the PAL in NAIA. Namboku opposed the Petition on the ground of inappropriateness. It claimed that the
members of the PALCEA-SUPER are project employees. Hence, they cannot represent its regular rank-and-file
employees.
On June 17, 2003, the Med-Arbiter thus granted the Petition and ordered the conduct of certification election. Namboku
appealed the Med-Arbiters Order to the Secretary of the Labor. In the meantime, Namboku received a summons setting
the pre-election conference on July 31, 2003 and stating that the Order granting the conduct of a certification election in
an unorganized establishment is not appealable. Namboku filed a Manifestation and Motion, seeking to suspend the
12

conduct of certification election pending resolution of its appeal. It contended that Section 17, Rule VIII of Department
Order No. 40-03 prohibiting the filing of an appeal from an order granting the conduct of a certification election in an
unorganized establishment is unconstitutional because it runs counter to Article 259 of the Labor Code. In a letterresolution dated October 22, 2003, the Secretary of Labor denied the appeal and affirmed the Med-Arbiters Order.
Second case: Phil-Japan is a domestic corporation engaged in manufacturing mufflers, chassis and other car accessories
for local and international markets. On June 6, 2003, PJWU-SUPER filed before the Med-Arbiter a Petition seeking to
determine the sole and exclusive bargaining representative of rank-and-file employees in Phil-Japan. Phil-Japan opposed
the Petition, claiming that the members of PJWU- SUPER are not its employees.
On August 25, 2003, the Med-Arbiter rendered a Decision ordering the conduct of certification election.
Aggrieved, Phil-Japan appealed the Decision of the Med-Arbiter to the Office of the Secretary of Labor asserting that the
Med-Arbiter gravely abused her discretion in not resolving the issue of whether employer-employee relationship existed
between the parties. In a hearing held on October 7, 2003, Hearing Officer Ching informed Phil-Japan that its appeal will
not be acted upon pursuant to Section 17, Rule VIII of Department Order No. 40-03 and that the certification election will
proceed accordingly. Namboku and Phil-Japan filed their respective petitions before the CA, imputing grave abyse
of discretion on the part of the Secretary of Labor.
The CA nullified Section 17, Rule VIII of Department Order No. 40-03 for being in conflict with Article 259 of the
Labor Code. The Secretary of Labor filed a Motion for Reconsideration. This prompted Namboku to file a Motion to
Expunge on the ground that the Secretary of Labor is a mere nominal party who has no legal standing to participate or
prosecute the case. On September 15, 2005, the CA issued a Resolution denying the Secretary of Labors Motion for
Reconsideration on the ground, among others, that she is merely a nominal party to the case and has no personal interest
therein. The CA likewise agreed with Phil-Japan that before extending labor benefits, the determination of whether an
employer-employee relationship exists is a primordial consideration. The CA further declared that for being violative of
Article 259 of the Labor Code, Section 17, Rule VIII of Department Order No. 40-03 has no legal force and effect.
On November 3, 2005, the Secretary of Labor filed before this Court a Petition for Review on Certiorari assailing the CA
decision in the first case. Then on November 11, 2005, the Secretary of Labor filed another Petition for Review on
Certiorari challenging the CA decision on the second case.
ISSUE:
Whether or not the Secretary of Labor is a real-party in interest with personality to file the petitions.
HELD:
No. A real party-in-interest is the party who stands to be benefited or injured by the judgment in the suit, or the party
entitled to the avails of the suit. As thus defined, the real parties-in-interest in these cases would have been PALCEASUPER and PJWU-SUPER. It would have been their duty to appear and defend the ruling of the Secretary of Labor for
they are the ones who were interested that the same be sustained.
As to the Secretary of Labor, she was impleaded in the Petitions for Certiorari filed before the CA as a nominal party
because one of the issues involved therein was whether she committed an error of jurisdiction. But that does not make her
a real party-in-interest or vests her with authority to appeal the Decisions of the CA in case it reverses her ruling. Another
reason that heavily militates against entertaining these Petitions is that the Secretary of Labor should have remained
impartial and detached from the cases she has decided even if the same are appealed to a higher court for review.
PHILIPPINE ELECTRIC CORPORATION (PHILEC) V. CA
G.R. NO. 168612
DECEMBER 10, 2014
J. LEONEN
PROPER REMEDY TO REVERSE OR MODIFY VAS DECISION
FACTS:
PHILEC is a domestic corporation engaged in the manufacture and repairs of high voltage transformers. Among its
rank-and-file employees were Lipio and Ignacio Sr., former members of the PHILEC Workers Union (PWU). PWU is a
legitimate labor organization and the exclusive bargaining representative of PHILECs rank-and-file employees.
From June 1, 1989 to May 31, 1997, PHILEC and its rank-and-file employees were governed by collective bargaining
agreements providing for step increases in an employees basic salary in case of promotion.
13

On August 18, 1997 and with the previous collective bargaining agreements already expired, PHILEC selected Lipio for
promotion from Machinist under Pay Grade VIII to Foreman I under Pay Grade B. Ignacio, Sr., then DT-Assembler with
Pay Grade VII, was likewise selected for training for the position of Foreman I.
On September 17, 1997, PHILEC and PWU entered into a new collective bargaining agreement, effective retroactively on
June 1, 1997 and expiring on May 31, 1999.
Claiming that the schedule of training allowance stated in the memoranda served on Lipio and Ignacio, Sr. did not
conform to Article X, Section 4 of the June 1, 1997 collective bargaining agreement, PWU submitted the grievance to the
grievance machinery. PWU and PHILEC failed to amicably settle their grievance. Thus, on December 21, 1998, the
parties filed a submission agreement with the NCMB.
Voluntary Arbitrator Jimenez held in the decision dated August 13, 1999, that PHILEC violated its collective bargaining
agreement with PWU. PHILEC should have computed Lipios and Ignacio, Sr.s training allowance based on Article X,
Section 4 of the June 1, 1997 collective bargaining agreement. Voluntary Arbitrator Jimenez ordered PHILEC to pay Lipio
and Ignacio, Sr. training allowance based on Article X, Section 4 and Article IX, Section 1 of the June 1, 1997 collective
bargaining agreement.
PHILEC received a copy of Voluntary Arbitrator Jimenezs decision on August 16, 1999. On August 26, 1999, PHILEC
filed a motion for partial reconsideration of Voluntary Arbitrator Jimenezs decision. In the resolution dated July 7, 2000,
Voluntary Arbitrator Jimenez denied PHILECs motion for partial reconsideration for lack of merit. PHILEC received a
copy of the July 7, 2000 resolution on August 11, 2000.
On August 29, 2000, PHILEC filed a petition for certiorari before the CA, alleging that Voluntary Arbitrator Jimenez
gravely abused his discretion in rendering his decision. The CA affirmed Voluntary Arbitrator Jimenezs decision.
PHILEC filed a motion for reconsideration, which the Court of Appeals denied in the resolution dated June 23, 2005.
On August 3, 2005, PHILEC filed its petition for review on certiorari before this court, insisting that it did not violate its
collective bargaining agreement with PWU.
ISSUE:
(1) Whether PHILEC erred in filing the petition for certiorari under Rule 65 before the CA against VA Jimenezs
decision.
(2) Whether the decision of the VA must be appealed within 10 days under Article 262-A of the Labor Code or within
15 days under Rule 43 of the Rules of Civil Procedure.
HELD:
(1) Yes. The proper remedy to reverse or modify a Voluntary Arbitrators or a panel of Voluntary Arbitrators decision
or award is to appeal the award or decision before the Court of Appeals. Rule 43, Sections 1 and 3 of the Rules of
Court provide:
Section

1.

Scope.

This Rule shall apply to appeals from judgments or final orders of the Court of Tax Appeals and from
awards, judgments, final orders or resolutions of or authorized by any quasi-judicial agency in the
exercise of its quasi-judicial functions. Among these agencies are the Civil Service Commission,
Central Board of Assessment Appeals, Securities and Exchange Commission, Office of the President,
Land Registration Authority, Social Security Commission, Civil Aeronautics Board, Bureau of
Patents, Trademarks and Technology Transfer, National Electrification Administration, Energy
Regulatory Board, National Telecommunications Commission, Department of Agrarian Reform under
Republic Act No. 6657, Government Service Insurance System, Employees Compensation
Commission, Agricultural Inventions Board, Insurance Commission, Philippine Atomic Energy
Commission, Board of Investments, Construction Industry Arbitration Commission, and voluntary
arbitrators authorized by law.
Since the office of a Voluntary Arbitrator or a panel of Voluntary Arbitrators is considered a quasi-judicial agency,
this court concluded that a decision or award rendered by a Voluntary Arbitrator is appealable before the Court of
Appeals.
(2) Despite Rule 43 providing for a 15-day period to appeal, we rule that the Voluntary Arbitrators decision must
be appealed before the Court of Appeals within 10 calendar days from receipt of the decision as provided in
the Labor Code.
Article 262-A of the Labor Code allows the appeal of decisions rendered by Voluntary Arbitrators. Statute
provides that the Voluntary Arbitrators decision shall be final and executory after 10 calendar days from receipt
of the copy of the award or decision by the parties. Being provided in the statute, this 10-day period must be
14

complied with; otherwise, no appellate court will have jurisdiction over the appeal. Furthermore, under Article
VIII, Section 5(5) of the Constitution, this court shall not diminish, increase, or modify substantive rights in
promulgating rules of procedure in courts. The 10-day period to appeal under the Labor Code being a substantive
right, this period cannot be diminished, increased, or modified through the Rules of Court.
The rule, therefore, is that a Voluntary Arbitrators award or decision shall be appealed before the Court of
Appeals within 10 days from receipt of the award or decision. Should the aggrieved party choose to file a motion
for reconsideration with the Voluntary Arbitrator, the motion must be filed within the same 10-day period since a
motion for reconsideration is filed within the period for taking an appeal.
OLORES V. MANILA DOCTORS COLLEGE
G.R. NO. 201663
MARCH 31, 2014
J. PERALTA
PERFECTION OF APPEAL
FACTS:
Petitioner was hired as a part-time faculty of respondent on 07 November 2005. He was assigned at the Humanities
Department of the College of Arts and Sciences. Thereafter, he signed fixed term employment contracts as part-time
instructor. From 03 November 2008, petitioner signed fixed term employment contracts, this time as a full- time
instructor.
On 13 April 2010, Bernardo charged petitioner with gross misconduct and gross inefficiency in the performance of duty.
Petitioner was accused of employing a grading system not in accordance with the system because he: a) added 50 pts to
the final examination raw scores; b) added 50 pts to students who have not been attending classes; c) credited only 40%
instead of 60% of the final examination; d) did not credit the essay questions; and e) added further incentives (1-4 pts)
aside from 50 pts.
Acting on the report of Bernardo, respondent created the Manila Doctors Tribunal (MDT) which was tasked to ascertain
the truth. The MDT sent notices of hearing to petitioner. On 31 May 2010, the MDT recommended petitioners dismissal.
On 07 June 2010, respondent terminated the services of petitioner for grave misconduct and gross inefficiency and
incompetence.
Aggrieved by the decision of respondent, petitioner filed a case for: a) illegal dismissal with a claim for reinstatement; b)
non-payment of service incentive leave and 13 th month pay; c) moral and exemplary damages; d) attorneys fees; and e)
regularization.
In a Decision dated December 8, 2010, the Labor Arbiter found merit in petitioners charge for illegal dismissal. However,
it dismissed petitioners claim for regularization. Respondent appealed from the aforesaid decision to the NLRC.
However, the same was denied in a Resolution dated February 10, 2011. The NLRC reasoned that respondents appeal was
not accompanied by neither a cash nor surety bond, thus, no appeal was perfected from the decision of the Labor Arbiter.
Respondent, thus, sought reconsideration of the NLRCs resolution. In a Decision dated September 30, 2011, the NLRC
granted respondents appeal and reversed its earlier resolution. Resultantly, petitioner filed a certiorari petition with the
CA.
In a Resolution dated January 9, 2012, the CA held that since petitioner failed to file a motion for reconsideration against
the NLRC decision before seeking recourse to it via a certiorari petition, the CA dismissed petitioners special civil action
for certiorari. Petitioner filed a motion for reconsideration. In a Resolution dated April 27, 2012, the CA denied
petitioners motion for reconsideration.
ISSUE:
(1) Whether respondents appeal with the NLRC was perfected despite its failure to post a bond.
(2) Assuming that the NLRC has jurisdiction over the instant case, is a motion for reconsideration still necessary
prior to the filing of the certiorari petition?
HELD:
(1) No. Article 223 of the Labor Code states that an appeal by the employer to the NLRC from a judgment of a Labor
Arbiter, which involves a monetary award, may be perfected only upon the posting of a cash or surety bond issued
by a reputable bonding company duly accredited by the NLRC, in an amount equivalent to the monetary award in
the judgment appealed from. The posting of a bond is indispensable to the perfection of an appeal in cases
involving monetary awards from the decisions of the Labor Arbiter. Moreover, the filing of the bond is not only
15

mandatory, but a jurisdictional requirement as well, that must be complied with in order to confer jurisdiction
upon the NLRC. Non-compliance therewith renders the decision of the Labor Arbiter final and executory.
Here, it is undisputed that respondents appeal was not accompanied by any appeal bond despite the clear
monetary obligation to pay petitioner his separation pay in the amount of P100,000.00. Since the posting of a
bond for the perfection of an appeal is both mandatory and jurisdictional, the decision of the Labor Arbiter sought
to be appealed before the NLRC had already become final and executory. Therefore, the NLRC had no authority
to entertain the appeal, much less to reverse the decision of the Labor Arbiter.
(2) No. The general rule is that a motion for reconsideration is indispensable before resort to the special civil action
for certiorari to afford the court or tribunal the opportunity to correct its error, if any. However, the same is
subject to several exceptions, one of which is where the questions raised in the certiorari proceedings have
been duly raised and passed upon by the lower court, or are the same as those raised and passed upon in
the lower court.
In the instant case, the NLRC had all the opportunity to review its ruling and correct itself. The NLRC issued a
ruling on February 10, 2011 in favor of petitioner dismissing respondents appeal on the ground that the latter
failed to file an appeal bond. However, upon a motion for reconsideration filed by respondent, the NLRC
completely reversed itself and set aside its earlier resolution dismissing the appeal. The NLRC had more than
enough opportunity to pass upon the issues raised by both parties on appeal of the ruling of the Labor Arbiter and
the subsequent motion for reconsideration of its resolution disposing the appeal. Thus, another motion for
reconsideration would have been useless under the circumstances since the questions raised in the certiorari
proceedings have already been duly raised and passed upon by the NLRC.
BALITE V. SS VENTURES INTERNATIONAL
G.R. NO. 195109
FEBRUARY 4, 2015
J. PEREZ
PERFECTION OF APPEAL
FACTS:
SS Ventures International, Inc. is a domestic corporation represented in this action by respondents Sung Sik Lee and
Evelyn Rayala. Petitioners Andy Balite, Monaliza Bihasa and Delfin Anzaldo were regular employees of the respondent
company until their employments were severed for violation of various company policies.||
Balite was issued a Show Cause Memorandum by the company on 4 August 2005 charging him with the following
infractions: (1) making false reports, malicious and fraudulent statements and rumor-mongering against the company; (2)
threatening and intimidating co-workers; (3) refusing to cooperate in the conduct of investigation; and (4) gross
negligence in the care and use of the company property resulting in the damage of the finished products. He was
dismissed from employment, through a Notice of Termination on 6 September 2005. Bihasa was charged with absence
without leave on two occasions and with improper behavior, stubbornness, arrogance and uncooperative attitude towards
superiors and employees. She was terminated from the service on 5 May 2006. Anzaldo was also dismissed from
employment. The records of the infractions he committed as well as the date of his termination, however, are not borne by
the records.
Consequently, the three employees charged respondents with illegal dismissal and recovery of backwages, 13th month pay
and attorney's fees before the Labor Arbiter.
On 30 December 2007, the Labor Arbiter rendered a Decision in favor of petitioners and held that respondents are liable
for illegal dismissal for failing to comply with the procedural and substantive requirements in terminating employment.
Aggrieved, respondents interposed an appeal by filing a Notice of Appeal and paying the corresponding appeal fee.
However, instead of filing the required appeal bond equivalent to the total amount of the monetary award which is
P490,308.00, respondents filed a Motion to Reduce the Appeal Bond to P100,000.00 and appended therein a manager's
check bearing the said amount. Respondents cited financial difficulty as justification for their inability to post the appeal
bond in full owing to the partial shutdown of respondent company's operations. The NLRC dismissed the appeal filed by
the respondents for non-perfection.|||
ISSUE:
Whether respondent perfected its appeal.
HELD:

16

Yes. An appeal from the Labor Arbiter to the NLRC must be perfected within ten calendar days from receipt of such
decisions, awards or orders of the Labor Arbiter. In a judgment involving a monetary award, the appeal shall be perfected
only upon (1) proof of payment of the required appeal fee; (2) posting of a cash or surety bond issued by a reputable
bonding company; and (3) filing of a memorandum of appeal.|||
The posting of a cash or surety bond in an amount equivalent to 10% of the monetary award pending resolution of the
motion to reduce appeal bond shall be deemed sufficient to perfect an appeal. The Court holds that the appeal bond posted
by the respondent in the amount of P100,000.00 which is equivalent to around 20% of the total amount of monetary bond
is sufficient to perfect an appeal. With the employer's demonstrated good faith in filing the motion to reduce the bond on
demonstrable grounds coupled with the posting of the appeal bond in the requested amount, as well as the filing of the
memorandum of appeal, the right of the employer to appeal must be upheld. This is in recognition of the importance of the
remedy of appeal, which is an essential part of our judicial system and the need to ensure that every party litigant is given
the amplest opportunity for the proper and just disposition of his cause freed from the constraints of technicalities.
FERNANDEZ V. BOTICA CLAUDIO
G.R. NO. 205870
AUGUST 13, 2014
J. PERLAS BERNABE
FILING OF A MOTION FOR RECONSIDERATION INDISPENSABLE BEFORE AGGRIEVED PARTY CAN
AVAIL OF RULE 65
FACTS:
On November 14, 2002, Fernandez was hired as a trainee at Botica Claudio. In January 2003, she was promoted as sales
clerk/pharmacy aide, which position she held until the termination of her services on January 15, 2006. Due to her
termination, Fernandez filed a complaint for illegal dismissal with prayer for the payment of her statutory benefits against
Jose before the NLRC Regional Arbitration Branch.
Jose, owner of Botica Claudia, contended that Fernandezs dismissal was valid given that she went on AWOL; this, in
addition to the various infractions she committed during her employment, particularly, her acts of (a) dispensing wrong
medicines, (b) allowing some clients to buy medicines on credit without her employers consent, and (c) dishonesty.
Further, Jose claimed that all of her employees, including Fernandez, were paid their corresponding benefits, and that their
SSS contributions were all duly remitted.
In a Decision dated December 11, 2007, the LA held that while just cause attended Fernandezs dismissal from work
based on the finding that she went on AWOL, the same was nonetheless effected without procedural due process.
Dissatisfied with the LAs ruling, Fernandez filed a Notice of Appeal with Memorandum of Appeal on February 8, 2008
before the NLRC. Copies of the same were purportedly sent by registered mail to one Atty. Ramon E. Solis, Jr., Counsel
for respondents, No. 5 Sto. Nino St., SFDM, 1100 Quezon City. On March 15, 2010, the NLRC rendered a Resolution
granting Fernandezs appeal, and thereby reversing the LAs ruling.
On June 1, 2010, an Entry of Judgment was issued by the NLRC, declaring its Resolution to have become final and
executory on May 18, 2010. Consequently, the LA issued an Order dated August 17, 2010 granting Fernandezs motion
for execution. Without disclosing the date when the foregoing resolution was received, Jose filed a motion for
reconsideration dated January 20, 2011 before the NLRC, insisting that just causes attended Fernandezs dismissal, albeit
the same was made without procedural due process. Despite the fact that the NLRC had yet to act on the aforesaid motion
for reconsideration, Jose filed a second motion for reconsideration dated February 2, 2011 before the same tribunal.
Notwithstanding the pendency of the aforesaid motions for reconsideration, Jose filed a petition for certiorari before the
CA, claiming to have secured a copy of the NLRC Resolution and LA Order only upon personal verification on February
8, 2010 and filed a motion for reconsideration therefrom on April 12, 2011, referring to her second motion for
reconsideration dated February 2, 2011.
The CA granted Joses petition for certiorari, holding that the NLRC gravely abused its discretion in taking cognizance of
Fernandezs appeal despite the latters failure to furnish Jose copies of her notice of appeal and appeal memorandum in
violation of Article 223 of the Labor Code and the NLRC Rules of Procedure. Aggrieved, Fernandez sought
reconsideration but the same was denied in a Resolution dated February 11, 2013, hence, the instant petition.
ISSUE:
(1)Whether or not the CA gravely abused its discretion in giving due course to respondents Ryle 65 certiorari.
(2)Whether the CA erred in declaring that the failure of Fernandez to furnish Jose with copies of her notice of appeal
and memorandum of appeal before the NLRC deprived the latter of her right to due process.
HELD:
17

(1) Yes. The CA gravely abused its discretion in giving due course to respondents Rule 65 certiorari petition despite
its finding that the latter still had a pending motion for reconsideration from the Decision dated March 15, 2010
before the NLRC. It is settled that the filing of a motion for reconsideration from the order, resolution or decision
of the NLRC is an indispensable condition before an aggrieved party can avail of a petition for certiorari. This is
to afford the NLRC an opportunity to rectify its perceived errors or mistakes, if any. Hence, the more prudent
recourse for respondent should have been to move for the immediate resolution of its motion for reconsideration
before the NLRC instead of filing a petition for certiorari before the CA. Having failed to do so, her petition for
certiorari was prematurely filed, and the CA should have dismissed the same.
(2) Yes. The mere failure to serve the same upon the opposing party does not bar the NLRC from giving due course
to an appeal. Such failure is only treated as a formal lapse, an excusable neglect, and, hence, not a jurisdictional
defect warranting the dismissal of an appeal. Instead, the NLRC should require the appellant to provide the
opposing party copies of the notice of appeal and memorandum of appeal.
In this case, however, the NLRC could not be expected to require compliance from Fernandez, the appellant, since
it was not aware that the opposing party, Jose, was not notified of her appeal. Hence, it cannot be faulted in
relying on Fernandezs representation that she had sent Jose, through her counsel, a copy of her memorandum of
appeal by registered mail. More significantly, it is undisputed that Jose eventually participated in the appeal
proceedings by filing not only one but two motions for reconsideration from the NLRC Resolution, thereby
negating any supposed denial of due process on her part.
ABING V. NLRC
G.R. NO. 185345

SEPTEMBER 10, 2014

J. REYES
FOUR-FOLD TEST
FACTS:
In December 1991, petitioner Abing sought employment with respondent Allied Bank, and was instructed to go to
respondent Marilag, which had a service contract with the said bank. The petitioner filled out an application form with
Marilag, passed the medical examination, and was told to report at Allied Bank. Assigned at its legal department, the
petitioner was progressively assigned various tasks such as messenger, skip tracer, checker and verifier of properties, and
receiving clerk/vault keeper. He was issued an Allied Bank ID as its contractual employee.
On August 26, 2002, Allied Bank's service contract with Marilag was terminated, and Allied Bank entered into a new
service contract with respondent FGSI. On September 3, 2002, the petitioner was instructed to report to FGSI, where he
filled out an application form. Thereafter, he resumed his work at Allied Bank.
In October 2003, Allied Bank terminated its contract with FGSI, and thus told the petitioner to stop reporting at its main
office by October 17, 2003. Claiming that he was an employee of the said bank and that he was being illegally terminated
without due process, the petitioner filed a complaint against Allied Bank for illegal dismissal.
Allied Bank denied that the petitioner was its employee, pointing out that it was Marilag and FGSI which hired him to
perform services for the bank under their service agreements.
FGSI tried to show that it was an independent job contractor, employing the petitioner as a bookkeeper/receiving
clerk/messenger with a daily salary of P250.00 plus P30.00 ECOLA, as evidenced by the Employment Agreement and
Manifestation signed by the petitioner. It denied illegally dismissing the petitioner, claiming that when its service contract
with Allied Bank was terminated, they re-assigned him to another workplace but the petitioner refused to be re-assigned.
Realizing that the said assignment was no longer possible due to the termination of its service agreement with FGSI,
petitioner executed a Quitclaim and Release after he was paid his 13 th month pay and service incentive leave pay. Marilag
also manifested that the petitioner also executed a quitclaim in its favor, after its service contract with Allied Bank was
terminated, and by then, the petitioner had resumed his assignment at Allied Bank under FGSI's service contract.
The LA dismissed the petitioner's complaint for illegal dismissal. On appeal, the NLRC reversed the LA, having found
that an employer-employee relationship existed between the petitioner and Allied Bank, in view of the fact that his
services were usually necessary and desirable to the business of the said bank.
On motions for reconsideration filed by Allied Bank and FGSI, however, the NLRC granted the same. The NLRC
reinstated the decision of the LA, finding that the petitioner was an employee of a legitimate job contractor, FGSI, which
exercised control and supervision over him. Moreover, the NLRC noted that he signed a release and quitclaim in favor of
FGSI. The CA upheld the NLRC, finding that FGSI is a legitimate job contractor.
ISSUE:
18

Whether an employer-employee relationship exists between Allied Bank and petitioner.


HELD:
No. Applying the four-fold test used in determining an employer-employee relationship, which are: (1) the selection and
engagement of employee; (2) the payment of wages; (3) the power of dismissal; and (4) the power to control the
employee's conduct, the LA, the NLRC and the CA are all in agreement that these elements are possessed by FGSI.
As to the employer's power of selection and engagement, it was FGSI which hired the petitioner and assigned him to work
at Allied Bank. In his Employment Agreement and Manifestation dated September 1, 2002, the petitioner explicitly
acknowledged that he was hired by FGSI, and in his position paper he also admitted that it was FGSI which instructed
him to report to Allied Bank.
As to the payment of wages, the petitioner collected his pay and benefits from FGSI. In his Quitclaim and Release dated
October 28, 2003, he also acknowledged the release and payment of all his monetary benefits by FGSI.
As to the power of dismissal, by signing the quitclaim, the petitioner acknowledged that it was FGSI which hired him and
had the power to terminate his services. Also in the petitioner's employment agreement, he bound himself to inform FGSI
if and when he was transferring to another agency, even as he also acknowledged the right of FGSI to terminate him in
case of any violation of its rules and regulations.
As to the power of control or supervision over the petitioner, FGSI through its Personnel Officer Marysol Gongona
regularly visited Allied Bank's premises for this very purpose. It also had the power to reassign the petitioner to other
clients.
ROYALE HOMES MARKETING V. ALCANTARA
G.R. NO. 195190
JULY 28, 2014
J. REYES
CONTROL TEST; INDEPENDENT CONTRACTOR
FACTS:
In 1994, Royale Homes appointed Alcantara as its Marketing Director for a fixed period of one year. His work consisted
mainly of marketing Royale Homes real estate inventories on an exclusive basis. Royale Homes reappointed him for
several consecutive years, the last of which covered the period January 1 to December 31, 2003 where he held the position
of Division 5 Vice-President-Sales.
On December 17, 2003, Alcantara filed a Complaint for Illegal Dismissal against Royale Homes and its President Matilde
Robles, Executive Vice-President for Administration and Finance Ma. Melinda Bernardino, and Executive Vice- President
for Sales Carmina Sotto. Alcantara alleged that he is a regular employee of Royale Homes since he is performing tasks
that are necessary and desirable to its business; that in 2003 the company gave him P1.2 million for the services he
rendered to it; that in the first week of November 2003, however, the executive officers of Royale Homes told him that
they were wondering why he still had the gall to come to office and sit at his table; and that the acts of the executive
officers of Royale Homes amounted to his dismissal from work without any valid or just cause and in gross disregard of
the proper procedure for dismissing employees.
Alcantara prayed to be reinstated to his former position without loss of seniority rights and other privileges, as well as to
be paid backwages, moral and exemplary damages, and attorneys fees. He further sought that the ownership of the
Mitsubishi Adventure with Plate No. WHD-945 be transferred to his name.
Royale Homes denied that Alcantara is its employee. It argued that the appointment paper of Alcantara is clear that it
engaged his services as an independent sales contractor for a fixed term of one year only. He was paid purely on
commission basis. In addition, Royale Homes had no control on how Alcantara would accomplish his tasks and
responsibilities as he was free to solicit sales at any time and by any manner which he may deem appropriate and
necessary. He is even free to recruit his own sales personnel to assist him in pursuance of his sales target. In a special
management committee meeting on October 8, 2003, however, Alcantara announced publicly and openly that he would
leave the company by the end of October 2003 and that he would no longer finish the unexpired term of his contract.
ISSUE:
Whether Alcantara was an independent contractor or an employee of Royale Homes.
HELD:
Alcantara is a mere independent contractor.
19

In determining the existence of an employer-employee relationship, this Court has generally relied on the four-fold test, to
wit: (1) the selection and engagement of the employee; (2) the payment of wages; (3) the power of dismissal; and (4) the
employers power to control the employee with respect to the means and methods by which the work is to be
accomplished.
Not every form of control is indicative of employer-employee relationship. A person who performs work for another and
is subjected to its rules, regulations, and code of ethics does not necessarily become an employee. As long as the level of
control does not interfere with the means and methods of accomplishing the assigned tasks, the rules imposed by the
hiring party on the hired party do not amount to the labor law concept of control that is indicative of employer-employee
relationship.
In this case, the rules, regulations, code of ethics, and periodic evaluation alluded to by Alcantara do not involve control
over the means and methods by which he was to perform his job. Understandably, Royale Homes has to fix the price,
impose requirements on prospective buyers, and lay down the terms and conditions of the sale, including the mode of
payment, which the independent contractors must follow. It is also necessary for Royale Homes to allocate its inventories
among its independent contractors, determine who has priority in selling the same, grant commission or allowance based
on predetermined criteria, and regularly monitor the result of their marketing and sales efforts. These do not pertain to the
means and methods of how Alcantara was to perform and accomplish his task of soliciting sales. They do not dictate upon
him the details of how he would solicit sales or the manner as to how he would transact business with prospective clients.
Notably, Alcantara was not required to observe definite working hours. Except for soliciting sales, Royale Homes did not
assign other tasks to him. He had full control over the means and methods of accomplishing his tasks as he can solicit
sales at any time and by any manner which he may deem appropriate and necessary. He performed his tasks on his own
account free from the control and direction of Royale Homes in all matters connected therewith, except as to the results
thereof.
PEOPLE V. VELASCO
G.R. NO. 195668
JUNE 25, 2014
J. BERSAMIN
SOLIDARY LIABILITY OF ILLEGAL RECRUITERS; JOINT TORTFEASORS
FACTS:
Maricar Inovero, Harleta Velasco, Marissa Diala and Berna Paulino were charged with illegal recruitment and 11
informations charging the same accused with estafa as defined and penalized under Article 315 2(a). Only Inovero was
arrested and prosecuted, the other accused having remained at large.
Baful testified that she and her sister-in-law, went to Harvel upon learning that recruitment for caregivers to Japan was ongoing. She testified that Diala, the alleged talent manager, directed her to submit certain documents, and to pay the
training fee, placement and processing fees. After complying with the requirements and after paying Diala, she was
promised deployment but she was never deployed.
Brizuela was another applicant for the caregiver recruitment. Diala told him the amount required as processing fee and the
documents to be submitted. And when he submitted the required documents and payments, it was Paulino who received
them. He likewise attended an orientation conducted by Inovero. On the eve of his supposed pre-departure orientation
seminar, Paulino texted him that the seminar was cancelled because Inovero, who had the applicants money, did not
show up. He testified that he was not deployed.
Aguirre, the third victim, alleged that she also went to HARVEL to apply as caregiver in Japan. Diala informed her that
Inovero was one of the owners of HARVEL and Velasco was its President; she paid and submitted her documents but
despite her undergoing medical examination and several training seminars, she was not deployed to Japan.
Marbella alleged that she applied for the position of janitress at HARVEL. Just like the rest of the complainants, she was
required to submit certain documents and to pay the as processing fee; after paying said fee, Diala and Inovero promised
her and the other applicants that they will be deployed however, the promised deployment never materialized.
Inovero denied all the allegations against her, claiming that she is the niece of Velasco, the owner of Harvel. She alleged
that she worked for her uncle, Velascos husband, as office assistant. She likewise denied receiving money from the
complainants, nor issuing receipts thereof. The RTC acquitted Inovero of 5 counts of estafa but convicted her for illegal
recruitment committed in large scale. The CA affirmed the conviction.
ISSUE:
What was the extent of Inoveros civil liability?
20

HELD:
The nature of the obligation of the co-conspirators in the commission of the crime requires solidarity, and each debtor may
be compelled to pay the entire obligation. As a co-conspirator, then, Inoveros civil liability was similar to that of a joint
tortfeasor under the rules of the civil law. Under Article 2194 of the Civil Code, joint tortfeasors are solidarily liable for
the resulting damage. In other words, joint tortfeasors are each liable as principals, to the same extent and in the same
manner as if they had performed the wrongful act themselves. Joint tortfeasors are those who command, instigate,
promote, encourage, advise, countenance, cooperate in, aid or abet the commission of a tort, or who approve of it after it is
done, if done for their benefit.
It would not be an excuse for any of the joint tortfeasors to assert that her individual participation in the wrong was
insignificant as compared to those of the others. Joint tortfeasors are not liable pro rata. The damages cannot be
apportioned among them, except by themselves. They cannot insist upon an apportionment, for the purpose of each
paying an aliquot part. They are jointly and severally liable for the whole amount. Hence, Inoveros liability towards the
victims of their illegal recruitment was solidary, regardless of whether she actually received the amounts paid or not, and
notwithstanding that her co-accused, having escaped arrest until now, have remained untried.
GAMBOA JR. V. VILLEGAS
G.R. NO. 179654 MARCH 18, 2015
J. PERALTA
PAYMENT ON PIECE-RATE BASIS DOES NOT NEGATE REGULAR EMPLOYMENT
FACTS:
Villegas is an employee at the Hacienda Leddy as early as 1960, when it was still named Hacienda Teresa. Later on named
Hacienda Leddy owned by Gamboa Sr., the same was succeeded by his son, petitioner Gamboa, Jr. During his
employment up to the time of his dismissal, Villegas performed sugar farming job 8 hours a day, 6 days a week work,
continuously for not less than 302 days a year, and for which services he was paid P45.00 per day. He likewise worked in
petitioner's coconut lumber business where he was paid P34.00 a day for 8 hours work.
On June 9, 1993, petitioner went to Villegas' house and told him that his services were no longer needed without prior
notice or valid reason. Hence, Villegas filed the complaint for illegal dismissal.
Petitioner denied having dismissed Villegas but admitted in his earlier position paper that Villegas indeed worked with the
said farm owned by his father, doing casual and odd jobs until the latter's death in 1993. He was even given the benefit of
occupying a small portion of the land where his house was erected. He, however, maintained that Villegas ceased working
at the farm as early as 1992, contrary to his allegation that he was dismissed.
Later, Gamboa instead insisted that the farm records reveal that the only time Villegas rendered service for the hacienda
was only in the year 1993, specifically February 9, 1993 and February 11, 1993 when he was contracted by the farm to cut
coconut lumber which were given to regular workers for the repairs of their houses.
The LA found that there was illegal dismissal. On appeal, the NLRC set aside and vacated the LA's decision. Complainant
moved for reconsideration, but was denied. Thus, via petition for certiorari under Rule 65 of the Rules of Court, Villegas
appealed before the CA and sought the annulment of the Resolutions of the NLRC. The CA annulled and set aside the
NLRC Decision. It further reinstated the LAs Decision.
Petitioner disputed that there exists an employer-employee relationship between him and Villegas. He claimed that
respondent was paid on a piece-rate basis without supervision. Petitioner added that since his job was not necessary or
desirable in the usual business or trade of the hacienda, he cannot be considered as a regular employee.
ISSUE:
Whether the CA erred in affirming the LAs decision which held that Villegas is a regular employee.
HELD:
No. Article 280 of the Labor Code, describes a regular employee as one who is either (1) engaged to perform activities
which are necessary or desirable in the usual business or trade of the employer; and (2) those casual employees who have
rendered at least one year of service, whether continuous or broken, with respect to the activity in which he is employed.
First, even Gamboa admitted that by act of generosity and compassion, Villegas was given a privilege of erecting his
house inside the hacienda during his employment. While it may indeed be an act of good will on the part of the Gamboas,
still, such act is usually done by the employer either out of gratitude for the employees service or for the employer's
21

convenience as the nature of the work calls for it. Second, length of service is an indication of the regularity of his
employment. Even assuming that Villegas was doing odd jobs around the farm, such long period of doing said odd jobs is
indicative that the same was either necessary or desirable to petitioner's trade or business. While length of time may not be
the controlling test to determine if Villegas is indeed a regular employee, it is vital in establishing if he was hired to
perform tasks which are necessary and indispensable to the usual business or trade of the employer. Petitioner himself
admitted that Villegas had worked in the hacienda until his father's demise. Clearly, even assuming that Villegas'
employment was only for a specific duration, the fact that he was repeatedly re-hired over a long period of time shows
that his job is necessary and indispensable to the usual business or trade of the employer.
Third, while petitioner argued that Villegas was paid on a piece-rate basis, this does not negate regular employment. The
term wage is broadly defined in Article 97 of the Labor Code as remuneration or earnings, capable of being expressed in
terms of money whether fixed or ascertained on a time, task, piece or commission basis. Payment by the piece is just a
method of compensation and does not define the essence of the relations.
BASAN V. COCA-COLA BOTTLERS
G.R. NO. 174365-66 FEBRUARY 4, 2015
J. PERALTA
REGULAR EMPLOYEES
FACTS:
On February 18, 1997, petitioners filed a complaint for illegal dismissal with money claims against respondent Coca-Cola
Bottlers Philippines, alleging that respondent dismissed them without just cause and prior written notice required by law.
In their position paper, petitioners provided for the following material dates:
Name of Petitioner

Date of Hiring

Date of Dismissal

Dela Rama

November 16, 1995

February 13, 1997

Dizon

October 1988

December 15, 1996

Tumabiao

February 2, 1992

February 13, 1997

Basan

July 13, 1996

January 31, 1997

Donor

September 16, 1995

February 13, 1997

Nicolas

May 10, 1996

January 30, 1997

Falguera

January 15, 1991

April 1996

Respondent corporation, however, countered that it hired petitioners as temporary route helpers to act as substitutes for its
absent regular route helpers merely for a fixed period in anticipation of the high volume of work in its plants or sales
offices. As such, petitioners claims have no basis for they knew that their assignment as route helpers was temporary in
duration.
The Labor Arbiter ruled in favor of petitioners and found that since they were performing activities necessary and
desirable to the usual business of petitioner for more than the period for regularization, petitioners are considered as
regular employees. The NLRC affirmed the Labor Arbiters decision.
On December 9, 2003, respondent filed a petition for certiorari with the CA alleging grave abuse of discretion on the part
of the NLRC. In the meantime, petitioners filed before the Labor Arbiter a Motion for Issuance of a Writ of Execution. On
March 25, 2004, the Labor Arbiter ordered that the Writ of Execution be issued, which was affirmed by the NLRC on June
21, 2004. Consequently, respondent filed another petition for certiorari, claiming that the NLRC committed grave abuse
of discretion in directing the execution of a judgment, the propriety and validity of which was still under determination of
the appellate court.
The CA consolidated respondents 2 petitions for certiorari and reversed the rulings of the NLRC and the Labor Arbiter.
The CA ruled that respondents repeated hiring for various periods (ranging from more than six months for private
respondent Basan to eight years in the case of private respondent Dizon) would not automatically categorize them as
REGULAR EMPLOYEES. That the respondents performed duties which are necessary or desirable in the usual trade or
business of Coca-Cola, is of no moment because this is not the only standard for determining the status of ones
employment. Such fact does not prevent them from being considered as fixed term employees of Coca-Cola whose
engagement was fixed for a specific period. Petitioners sought a reconsideration of the CAs Decision. Petitioners assert
that they are regular employees entitled to security of tenure.
ISSUE:
22

Whether the CA erred in ruling that petitioners were not regular employees.
HELD:
Yes. There are two kinds of regular employees, namely: (1) those who are engaged to perform activities which are usually
necessary or desirable in the usual business or trade of the employer; and (2) those who have rendered at least one year of
service, whether continuous or broken, with respect to the activities in which they are employed. Simply stated, regular
employees are classified into: (1) regular employees by nature of work; and (2) regular employees by years of service.
The former refers to those employees who perform a particular activity which is necessary or desirable in the usual
business or trade of the employer, regardless of their length of service; while the latter refers to those employees who have
been performing the job, regardless of the nature thereof, for at least a year.
Petitioners fall under the first kind of regular employee above. As route helpers who are engaged in the service of loading
and unloading softdrink products of respondent company to its various delivery points, which is necessary or desirable in
its usual business or trade, petitioners are considered as regular employees. That they merely rendered services for periods
of less than a year is of no moment since for as long as they were performing activities necessary to the business of
respondent, they are deemed as regular employees under the Labor Code, irrespective of the length of their service.
Petitioners cannot be considered as fixed term employees because there is no proof which will show that petitioners freely
entered into agreements with respondent to perform services for a specified length of time. In fact, there is nothing in the
records to show that there was any agreement at all, the contracts of employment not having been presented. While
respondent company persistently asserted that petitioners knowingly agreed upon a fixed period of employment and
repeatedly made reference to their contracts of employment, the expiration thereof being made known to petitioners at the
time of their engagement, respondent failed to present the same in spite of all the opportunities to do so. Hence, in the
absence of proof showing that petitioners knowingly agreed upon a fixed term of employment, petitioners are, indeed,
regular employees, entitled to security of tenure.
HACIENDA CATAYWA V. LOREZO
G.R. NO. 179640
MARCH 18, 2015
J. PERALTA
SEASONAL EMPLOYEES
FACTS:
On October 22, 2002, respondent Lorezo received, upon inquiry, a letter from the SSS informing her that she cannot avail
of their retirement benefits since per their record she has only paid 16 months. Such is 104 months short of the minimum
requirement of 120 months payment to be entitled to the benefit. She was also informed that their investigation of her
alleged employment under employer Hda. Cataywa could not be confirmed because Manuel Villanueva was permanently
residing in Manila and Joemarie Villanueva denied having managed the farm. She was also advised of her options:
continue paying contributions as voluntary member; request for refund; leave her contributions in-trust with the System,
or file a petition before the Social Security Commission (SSC) so that liabilities, if any, of her employer may be
determined.
Aggrieved, respondent then filed her Amended Petition dated September 30, 2003, before the SSC. She alleged that she
was employed as laborer in Hda. Cataywa managed by Jose Marie Villanueva in 1970 but was reported to the SSS only in
1978. She alleged that SSS contributions were deducted from her wages from 1970 to 1995, but not all were remitted to
the SSS which, subsequently, caused the rejection of her claim. She also impleaded Talisay Farms, Inc. by virtue of its
Investment Agreement with Mancy and Sons Enterprises.
Petitioners Manuel and Jose Villanueva refuted in their answer, the allegation that not all contributions of respondent were
remitted. Consequently, the SSC rendered its Resolution dated October 12, 2005 and held that Lorezo was a regular
employee subject to compulsory coverage of Hda. Cataywa/Manuel Villanueva/ Mancy and Sons Enterprises, Inc. within
the period of 1970 to February 25, 1990.
ISSUE:
(1) What pieces or form of evidence are required to prove the existence of employer-employee relationship.
(2) Whether petitioner is correct in arguing that respondent was a casual worker as records revealed that she only had
16 months worth of contributions.
HELD:
23

(1) There is no particular form of evidence required to prove the existence of the employer-employee relationship.
Any competent and relevant evidence to prove such relationship may be admitted. This may entirely be
testimonial. If only documentary evidence would be required to demonstrate the relationship, no scheming
employer would be brought before the bar of justice.
Petitioners is wrong in insisting that SSS Form R-1A is the only remaining source of information available to
prove when respondent started working for them. Such form merely reflected the time in which the petitioners
reported the respondent for coverage of the SSS benefit. They failed to substantiate their claim that it was only in
1978 that respondent reported for work.
(2) No. Jurisprudence has identified the three types of employees mentioned in the provision of the Labor Code: (1)
regular employees or those who have been engaged to perform activities that are usually necessary or desirable in
the usual business or trade of the employer; (2) project employees or those whose employment has been fixed for
a specific project or undertaking, the completion or termination of which has been determined at the time of their
engagement, or those whose work or service is seasonal in nature and is performed for the duration of the season;
and (3) casual employees or those who are neither regular nor project employees.
Farm workers generally fall under the definition of seasonal employees. It was also consistently held that seasonal
employees may be considered as regular employees when they are called to work from time to time. They are in
regular employment because of the nature of the job, and not because of the length of time they have worked.
However, seasonal workers who have worked for one season only may not be considered regular employees. The
nature of the services performed and not the duration thereof, is determinative of coverage under the law. Thus, it
is erroneous for the petitioners to conclude that the respondent was a very casual worker simply because the SSS
form revealed that she had 16 months of contributions.
Here, petitioners failed to dispute the allegation that the respondent performed hacienda work, such as planting
sugarcane point, fertilizing, weeding, replanting dead sugarcane fields and routine miscellaneous hacienda work.
Thus, respondent is considered a regular seasonal worker and not a casual worker as the petitioners
alleged. Since petitioners provided that the cultivation of sugarcane is only for six months, respondent
cannot be considered as regular employee during the months when there is no cultivation.
PAZ V. NORTHERN TOBACCO REDRYING CO.
G.R. NO. 199554 FEBRUARY 18, 2015
J. LEONEN
REGULAR SEASONAL WORKER; ILLEGAL DISMISSAL; RETIREMENT; SEPARATION PAY; FINANCIAL
ASSISTANCE
FACTS:
Northern Tobacco Redrying Co., Inc. (NTRCI), a flue-curing and redrying of tobacco leaves business, employs
approximately 100 employees with seasonal workers tasked to sort, process, store and transport tobacco leaves during
the tobacco season of March to September.
NTRCI hired Paz sometime in 1974 as a seasonal sorter, paid P185.00 daily. NTRCI regularly re-hired her every tobacco
season since then. She signed a seasonal job contract at the start of her employment and a pro-forma application letter
prepared by NTRCI in order to qualify for the next season.
On May 18, 2003, Paz was 63 years old when NTRCI informed her that she was considered retired under company policy.
A year later, NTRCI told her she would receive P12,000.00 as retirement pay.
Paz filed a Complaint for illegal dismissal against NTRCI on March 4, 2004. She amended her Complaint on April 27,
2004 into a Complaint for payment of retirement benefits, damages, and attorneys fees as P12,000.00 seemed inadequate
for her 29 years of service.
ISSUE:
(1) Is Paz considered a regular seasonal employee of NTRCI?
(2) Was Paz illegally dismissed?
(3) Is Paz entitled to retirement pay, financial assistance and nominal damages?
HELD:
(1) Yes. The primary standard of determining regular employment is the reasonable connection between the particular
activity performed by the employee in relation to the usual trade or business of the employer. The test is whether
the former is usually necessary or desirable in the usual business or trade of the employer. The connection can be
determined by considering the nature of the work performed and its relation to the scheme of the particular
24

business or trade in its entirety. Also if the employee has been performing the job for at least a year, even if the
performance is not continuous and merely intermittent, the law deems repeated and continuing need for its
performance as sufficient evidence of the necessity if not indispensability of that activity to the business. Hence,
the employment is considered regular, but only with respect to such activity, and while such activity exists.
The services petitioner Paz performed as a sorter were necessary and indispensable to respondent NTRCIs
business of flue-curing and redrying tobacco leaves. She was also regularly rehired as a sorter during the tobacco
seasons for 29 years since 1974. These considerations taken together allowed the conclusion that petitioner Paz
was a regular seasonal employee.
(2) Yes. Retirement is the result of a bilateral act of the parties, a voluntary agreement between the employer and the
employee whereby the latter, after reaching a certain age, agrees to sever his or her employment with the former.
Article 287, as amended, allows for optional retirement at the age of at least 60 years old. Consequently, if the
intent to retire is not clearly established or if the retirement is involuntary, it is to be treated as a discharge.
Petitioner Paz was only 63 years old on May 18, 2003 with two more years remaining before she would reach the
compulsory retirement age of 65. Again, petitioner Paz never abandoned her argument of illegal dismissal despite
the amendment of her Complaint. This implied lack of intent to retire until she reached the compulsory age of
65. Thus, she should be considered as illegally dismissed from May 18, 2003 until she reached the compulsory
retirement age of 65 in 2005 and should be entitled to full backwages for this period.
(3) Yes.
Nominal Damages: Four possible situations may be derived: (1) the dismissal is for a just cause under Article 282
of the Labor Code, for an authorized cause under Article 283, or for health reasons under Article 284, and due
process was observed; (2) the dismissal is without just or authorized cause but due process was observed; (3) the
dismissal is without just or authorized cause and there was no due process; and (4) the dismissal is for just or
authorized cause but due process was not observed. In the second and third situations where the dismissals are
illegal, Article 279 mandates that the employee is entitled to reinstatement without loss of seniority rights and
other privileges and full backwages, inclusive of allowances, and other benefits or their monetary equivalent
computed from the time the compensation was not paid up to the time of actual reinstatement.
Petitioner Pazs case does not fall under the fourth situation but under the third situation on illegal dismissal for
having no just or authorized cause and violation of due process. There was no showing that respondent NTRCI
complied with due process requisites. Thus, consistent with jurisprudence, petitioner Paz should be awarded
P30,000.00 as nominal damages.
Retirement Pay: While the present case involves retirement pay and not separation pay, Article 287 of the Labor
Code on retirement pay similarly provides that a fraction of at least six (6) months being considered as one
whole year. The amount of separation pay is based on two factors: the amount of monthly salary and the number
of years of service. Although the Labor Code provides different definitions as to what constitutes one year of
service, Book Six does not specifically define one year of service for purposes of computing separation pay.
However, Articles 283 and 284 both state in connection with separation pay that a fraction of at least six months
shall be considered one whole year.
Since respondent NTRCI failed to present a copy of a Collective Bargaining Agreement on the alleged retirement
policy, we apply Article 287 of the Labor Code, as amended by Republic Act No. 7641. Respondent NTRCI
followed the formula in Article 287 and offered petitioner Paz the amount of P12,487.50 as retirement pay based
on the three years she worked for at least six months in 1995, 1999, and 2000. Based on these factual findings,
retirement pay pursuant to Article 287 of the Labor Code was correctly computed at P12,487.50 and was awarded
to petitioner Paz.
Financial Assitance: This court has awarded financial assistance as a measure of social justice in exceptional
circumstances, and as an equitable concession.
The Court of Appeals recognized and emphasized petitioner Pazs three decades of hard work and service with
respondent NTRCI. She had no record of any malfeasance or violation of company rules in her long years of
service. Her advanced age has rendered her weak and lessened her employment opportunities.Hence, this court
agrees with the Court of Appeals award of financial assistance in the amount of P60,356.25 by applying the
following formula: one-half-month pay multiplied by 29 years in service and then divided by 2.
FONTERRA BRANDS PHILS. V. LARGADO
G.R. NO. 205300
MARCH 18, 2015

25

J. VELASCO
LEGITIMATE LABOR CONTRACTING V. LABOR ONLY CONTRACTING; FIXED TERM EMPLOYMENT
FACTS:
Fonterra contracted the services of Zytron for the marketing and promotion of its milk and dairy products. Pursuant to the
contract, Zytron provided Fonterra with trade merchandising representatives (TMRs), including respondents Largado and
Estrellado. The engagement of their services began on September 15, 2003 and May 27, 2002, respectively, and ended on
June 6, 2006.
On May 3, 2006, Fonterra sent Zytron a letter terminating its promotions contract, effective June 5, 2006. Fonterra then
entered into an agreement for manpower supply with A.C. Sicat. Desirous of continuing their work as TMRs, respondents
submitted their job applications with A.C. Sicat, which hired them for a term of 5 months, beginning June 7, 2006 up to
November 6, 2006. When respondents 5-month contracts with A.C. Sicat were about to expire, they allegedly sought
renewal thereof, but were allegedly refused. This prompted respondents to file complaints for illegal dismissal,
regularization, non-payment of service incentive leave and 13 th month pay, and actual and moral damages, against
petitioner, Zytron, and A.C. Sicat.
The Labor Arbiter dismissed the complaint. The NLRC affirmed the Labor Arbiter. The NLRC decision was assailed in a
petition under Rule 65 before the CA. Ruling on the petition, the CA found that A.C. Sicat satisfies the requirements of
legitimate job contracting, but Zytron does not. According to the CA: (1) Zytrons paid-in capital of P250,000 cannot be
considered as substantial capital; (2) its Certificate of Registration was issued by the DOLE months after respondents
supposed employment ended; and (3) its claim that it has the necessary tools and equipment for its business is
unsubstantiated. Therefore, according to the CA, respondents were Fonterras employees.
Additionally, the CA held that respondents were illegally dismissed since Fonterra itself failed to prove that their dismissal
is lawful. However, the illegal dismissal should be reckoned from the termination of their supposed employment with
Zytron on June 6, 2006. Furthermore, respondents transfer to A.C. Sicat is tantamount to a completely new engagement
by another employer. Lastly, the termination of their contract with A.C. Sicat arose from the expiration of their respective
contracts with the latter. The CA, thus, ruled that Fonterra is liable to respondents and ordered the reinstatement of
respondents without loss of seniority rights, with full backwages, and other benefits from the time of their illegal dismissal
up to the time of their actual reinstatement.
ISSUE:
(1)

Whether or not Zytron and A.C. Sicat are labor-only contractors, making Fonterra the employer of herein
respondents.

(2)

Whether or not respondents were illegally dismissed.

HELD:
(1) With respect to Zyton:
CAs conclusion that Zytron is not a legitimate job contractor is immaterial to the resolution of the illegal
dismissal issue for one reason: Respondents voluntarily terminated their employment with Zytron, contrary to
their allegation that their employment with Zytron was illegally terminated. The termination of respondents
employment with Zytron was brought about by the cessation of their contracts with the latter.
By refusing to renew their contracts with Zytron, respondents effectively resigned from the latter. Resignation is
the voluntary act of employees who are compelled by personal reasons to dissociate themselves from their
employment, done with the intention of relinquishing an office, accompanied by the act of abandonment. Here, it
is obvious that respondents were no longer interested in continuing their employment with Zytron. Their
voluntary refusal to renew their contracts was brought about by their desire to continue their assignment in
Fonterra which could not happen in view of the conclusion of Zytrons contract with Fonterra. Hence, to be able
to continue with their assignment, they applied for work with A.C. Sicat with the hope that they will be able to
continue rendering services as TMRs at Fonterra since A.C. Sicat is Fonterras new manpower supplier.
With respect to A.C. Sicat:
A.C. Sicats is a legitimate job contractor.
A person is considered engaged in legitimate job contracting or subcontracting if the following conditions concur:
1. The contractor or subcontractor carries on a distinct and independent business and undertakes to perform the
job, work or service on its own account and under its own responsibility according to its own manner and
26

method, and free from the control and direction of the principal in all matters connected with the
performance of the work except as to the results thereof;
2. The contractor or subcontractor has substantial capital or investment; and
3. The agreement between the principal and contractor or subcontractor assures the contractual employees
entitlement to all labor and occupational safety and health standards, free exercise of the right to selforganization, security of tenure, and social and welfare benefits.
On the other hand, contracting is prohibited when the contractor or subcontractor merely recruits, supplies or
places workers to perform a job, work or service for a principal and if any of the following elements are present,
thus:
2. The contractor or subcontractor does not have substantial capital or investment which relates to the job,
work or service to be performed and the employees recruited, supplied or placed by such contractor or
subcontractor are performing activities which are directly related to the main business of the principal; or
3. The contractor does not exercise the right to control over the performance of the work of the contractual
employee.
The CA correctly found that A.C. Sicat is engaged in legitimate job contracting. It duly noted that A.C. Sicat was
able to prove its status as a legitimate job contractor for having presented the following evidence, to wit: (1)
Certificate of Business Registration; (2) Certificate of Registration with the Bureau of Internal Revenue; (3)
Mayors Permit; (4) Certificate of Membership with the Social Security System; (5) Certificate of Registration
with the Department of Labor and Employment; (6) Company Profile; and (7) Certifications issued by its
clients. Furthermore, A.C. Sicat has substantial capital, having assets totaling P5,926,155.76 as of December 31,
2006. Too, its Agreement with Fonterra clearly sets forth that A.C. Sicat shall be liable for the wages and salaries
of its employees or workers, including benefits, premiums, and protection due them, as well as remittance to the
proper government entities of all withholding taxes, Social Security Service, and Medicare premiums, in
accordance with relevant laws.
(2) No. The termination of respondents employment with the latter was simply brought about by the expiration of
their employment contracts.
Respondents were fixed-term employees. Fixed-term employment contracts are not limited, as they are under the
present Labor Code, to those by nature seasonal or for specific projects with predetermined dates of completion;
they also include those to which the parties by free choice have assigned a specific date of termination. The
determining factor of such contracts is not the duty of the employee but the day certain agreed upon by the
parties for the commencement and termination of the employment relationship.
In the case at bar, it is clear that respondents were employed by A.C. Sicat as project employees. In their
employment contract with the latter, it is clearly stated that A.C. Sicat is temporarily employing respondents as
TMR[s] effective June 6, 2006 under the following terms and conditions: The need for your service being only for
a specific project, your temporary employment will be for the duration only of said project of our client, namely
to promote FONTERRA BRANDS products xxx which is expected to be finished on or before Nov. 06, 2006.
Respondents, by accepting the conditions of the contract with A.C. Sicat, were well aware of and even acceded to
the condition that their employment thereat will end on said pre-determined date of termination. The expiration of
their contract with A.C. Sicat simply caused the natural cessation of their fixed-term employment there at.
ALILIN V. PETRON CORPORATION
G.R. NO. 177592
JUNE 9, 2014
J. DEL CASTILLO
LEGITIMATE LABOR CONTRACTING V. LABOR ONLY CONTRACTING
FACTS:
In 1968, Romualdo D. Gindang Contractor, which was owned and operated by Romualdo, started recruiting laborers for
fielding to Petrons Mandaue Bulk Plant. When Romualdo died in 1989, his son Romeo, through RDG, took over the
business and continued to provide manpower services to Petron. Petitioners were among those recruited by Romualdo D.
Gindang Contractor and RDG to work in the premises of the said bulk plant, with the corresponding dates of hiring and
work duties.
27

On June 1, 2000, Petron and RDG entered into a Contract for Services for the period from June 1, 2000 to May 31, 2002,
whereby RDG undertook to provide Petron with janitorial, maintenance, tanker receiving, packaging and other utility
services in its Mandaue Bulk Plant. This contract was extended on July 31, 2002 and further extended until September
30, 2002.
Alleging that they were barred from continuing their services on October 16, 2002, petitioners Alilin, Calesa, Hindang,
Gindang, Sungahid, Lee, Morato and Gabilan filed a Complaint for illegal dismissal, underpayment of wages, damages
and attorneys fees against Petron and RDG on November 12, 2002. Petitioner Laurente filed another Complaint for
illegal dismissal, underpayment of wages, non-payment of overtime pay, holiday pay, premium pay for holiday, rest day,
13th month pay, service incentive leave pay, allowances, separation pay, retirement benefits, damages and attorneys fees
against Petron and RDG. The said complaints were later consolidated.
Petitioners did not deny that RDG hired them and paid their salaries. They, however, claimed that the latter is a labor-only
contractor, which merely acted as an agent of Petron, their true employer. Claiming to be regular employees, petitioners
thus asserted that their dismissal allegedly in view of the expiration of the service contract between Petron and RDG is
illegal.
RDG corroborated petitioners claim that they are regular employees of Petron. Petron, on the other hand, maintained that
RDG is an independent contractor and the real employer of the petitioners.
The Labor Arbiter ruled that petitioners are regular employees of Petron. It found that their jobs were directly related to
Petrons business operations; they worked under the supervision of Petrons foreman and supervisor; and they were using
Petrons tools and equipment in the performance of their works. Upon appeal, the NLRC ruled that petitioners are Petrons
regular employees because they are performing job assignments which are germane to its main business. Petron thennfiled
a Petition for Certiorari with prayer for the issuance of a temporary restraining order or writ of injunction before the CA.
The CA found no employer-employee relationship between the parties. According to it, the records of the case do not
show that petitioners were directly hired, selected or employed by Petron; that their wages and other wage related benefits
were paid by the said company; and that Petron controlled the manner by which they carried out their tasks. On the other
hand, RDG was shown to be responsible for paying petitioners wages. In fact, SSS records show that RDG is their
employer and actually the one remitting their contributions thereto. Also, two former employees of RDG who were
likewise assigned in the Mandaue Bulk Plant confirmed by way of a joint affidavit that it was Romeo and his brother
Alejandre Gindang who supervised their work, not Petrons foreman or supervisor. This was even corroborated by the
Terminal Superintendent of the Mandaue Bulk Plant. The CA also found RDG to be an independent labor contractor
with sufficient capitalization and investment as shown by its financial statement for year-end 2000. In addition, the
works for which RDG was contracted to provide were menial which were neither directly related nor sensitive and critical
to Petrons principal business.
ISSUE:
Whether RDG is a legitimate job contractor.
HELD:
No, RDG is a labor-only contractor. Permissible job contracting or subcontracting refers to an arrangement whereby a
principal agrees to farm out with a contractor or subcontractor the performance of a specific job, work, or service within a
definite or predetermined period, regardless of whether such job, work or, service is to be performed or completed within
or outside the premises of the principal. Under this arrangement, the following conditions must be met: (a) the contractor
carries on a distinct and independent business and undertakes the contract work on his account under his own
responsibility according to his own manner and method, free from the control and direction of his employer or principal in
all matters connected with the performance of his work except as to the results thereof; (b) the contractor has substantial
capital or investment; and (c) the agreement between the principal and contractor or subcontractor assures the contractual
employees entitlement to all labor and occupational safety and health standards, free exercise of the right to selforganization, security of tenure, and social welfare benefits.
Labor-only contracting, on the other hand, is a prohibited act, defined as supplying workers to an employer who does not
have substantial capital or investment in the form of tools, equipment, machineries, work premises, among others, and the
workers recruited and placed by such person are performing activities which are directly related to the principal business
of such employer. In distinguishing between prohibited labor-only contracting and permissible job contracting, the
totality of the facts and the surrounding circumstances of the case shall be considered.
Here, the audited financial statements and other financial documents of RDG for the years 1999 to 2001 establish that it
does have sufficient working capital to meet the requirements of its service contract. The evidence adduced merely
proves that RDG was financially qualified as a legitimate contractor but only with respect to its last service
contract with Petron in the year 2000. As may be recalled, petitioners have rendered work for Petron for a long period
of time even before the service contract was executed in 2000. Hence, while Petron was able to establish that RDG was
financially capable as a legitimate contractor at the time of the execution of the service contract in 2000, it nevertheless
28

failed to establish the financial capability of RDG at the time when petitioners actually started to work for Petron in 1968,
1979, 1981, 1987, 1990, 1992 and 1993. On this score alone, RDG is a mere labor-only contractor.
BUREAU OF CUSTOMS EMPLOYEES ASSOCIATION V. HON. TEVES
G.R. NO.181704
DECEMBER 6, 2011
J. VILLARAMA
SECURITY OF TENURE; DUE PROCESS
FACTS:
RA 9335 was enacted to optimize the revenue-generation capability and collection of the BIR and the BOC. The law
intends to encourage BIR and BOC officials and employees to exceed their revenue targets by providing a system of
rewards and sanctions through the creation of a Rewards and Incentives Fund and a Revenue Performance Evaluation
Board.
Sometime in 2008, high-ranking officials of the BOC pursuant to the mandate of R.A. No. 9335 and its IRR, and in order
to comply with the stringent deadlines thereof, started to disseminate Collection District Performance Contracts for the
lower ranking officials and rank-and-file employees to sign. The Performance Contract pertinently provided:
WHEREAS, pursuant to the provisions of Sec. 25 (b) of the Implementing Rules and Regulations (IRR)
of the Attrition Act of 2005, that provides for the setting of criteria and procedures for removing from the
service Officials and Employees whose revenue collection fall short of the target in accordance with
Section 7 of Republic Act 9335.
BOCEA opined that the revenue target was impossible to meet due to the Government's own policies on reduced tariff
rates and tax breaks to big businesses, the occurrence of natural calamities and because of other economic factors. Said
personnel were threatened that if they do not sign their respective Performance Contracts, they would face possible
reassignment, reshuffling, or worse, be placed on floating status.
BOCEA argued that R.A. No. 9335 and its IRR violate the BIR and BOC employees' right to security of tenure because
R.A. No. 9335 and its IRR effectively removed remedies provided in the ordinary course of administrative procedure
afforded to government employees. The law likewise created another ground for dismissal, i.e., non-attainment of revenue
collection target, which is not provided under CSC rules and which is, by its nature, unpredictable and therefore arbitrary
and unreasonable.
ISSUE:
Whether R.A. No. 9335 and its IRR violate the rights of BOCEA's members to security of tenure and due process.
HELD:
No. The guarantee of security of tenure only means that an employee cannot be dismissed from the service for causes
other than those provided by law and only after due process is accorded the employee.
Clearly, RA 9335 in no way violates the security of tenure of officials and employees of the BIR and the BOC. In the case
of RA 9335, it lays down a reasonable yardstick for removal (when the revenue collection falls short of the target by at
least 7.5%) with due consideration of all relevant factors affecting the level of collection. This standard is analogous to
inefficiency and incompetence in the performance of official duties, a ground for disciplinary action under civil service
laws. The action for removal is also subject to civil service laws, rules and regulations and compliance with substantive
and procedural due process.
In addition, the essence of due process is simply an opportunity to be heard, or as applied to administrative proceedings, a
fair and reasonable opportunity to explain one's side. The concerned BIR or BOC official or employee is not simply given
a target revenue collection and capriciously left without any quarter. R.A. No. 9335 and its IRR clearly give due
consideration to all relevant factors that may affect the level of collection. Moreover, an employee's right to be heard is not
at all prevented and his right to appeal is not deprived of him. In fine, a BIR or BOC official or employee in this case
cannot be arbitrarily removed from the service without according him his constitutional right to due process.
THE COFFEE BEAN AND TEA LEAF V. ARENAS
G.R. NO. 208908
MARCH 11, 2015
29

J. BRION
WILLFUL DISOBEDIENCE, GROSS NEGLIGENCE, SERIOUS MISCONDUCT
FACTS:
On April 1, 2008, CBTL hired Arenas to work as a "barista" at its Paseo Center Branch. His principal functions included
taking orders from customers and preparing their ordered food and beverages. Upon signing the employment
contract, Arenas was informed of CBTL's existing employment policies. To ensure the quality of its crew's services,
CBTL regularly employs a "mystery guest shopper" who poses as a customer, for the purpose of covertly inspecting the
baristas' job performance.
In April 2009, a mystery guest shopper submitted a report stating that on March 30, 2009, Arenas was seen eating nonCBTL products at CBTL's al fresco dining area while on duty. As a result, the counter was left empty without anyone to
take and prepare the customers' orders.||| Basallo prepared a store manager's report which listed Arenas' recent infractions,
as follows: 1. Leaving the counter unattended and eating chips in an unauthorized area while on duty (March 30, 2009);
2. Reporting late for work on several occasions (April 1, 3 and 22); and 3. Placing an iced tea bottle in the ice bin despite
having knowledge of company policy prohibiting the same (April 28, 2009).
Based on the mystery guest shopper and duty manager's reports, Arenas was required to explain his alleged violations.
However, CBTL found Arenas' written explanation unsatisfactory, hence CBTL terminated his employment.||| Arenas filed
a complaint for illegal dismissal. The LA ruled in his favor. The NLRC affirmed the LA's decision.|||
CBTL filed a petition for certiorari under Rule 65 before the CA. CBTL insisted that Arenas' infractions amounted to
serious misconduct or willful disobedience, gross and habitual neglect of duties, and breach of trust and confidence. The
CA dismissed the petition.
ISSUE:
Whether the CA erred in affirming the LA and NLRC findings.
HELD:
No.
(A) For willful disobedience to be a valid cause for dismissal, these two elements must concur: (1) the employee's
assailed conduct must have been willful, that is, characterized by a wrongful and perverse attitude; and (2) the order
violated must have been reasonable, lawful, made known to the employee, and must pertain to the duties which he had
been engaged to discharge.
Arenas' alleged infractions do not amount to such a wrongful and perverse attitude. Though Arenas may have admitted
these wrongdoings, these do not amount to a wanton disregard of CBTL's company policies. As Arenas mentioned in his
written explanation, he was on a scheduled break when he was caught eating at CBTL's al fresco dining area. During that
time, the other service crews were the one in charge of manning the counter. Notably, CBTL's employee handbook
imposes only the penalty of written warning for the offense of eating non-CBTL products inside the store's premises.
(B) For gross negligence, the same implies a want or absence of, or failure to exercise even a slight care or diligence, or
the entire absence of care. It evinces a thoughtless disregard of consequences without exerting any effort to avoid them.
There is habitual neglect if based on the circumstances, there is a repeated failure to perform one's duties for a period of
time.
Arenas' three counts of tardiness cannot be considered as gross and habitual neglect of duty. The infrequency of his
tardiness already removes the character of habitualness. These late attendances were also broadly spaced out, negating the
complete absence of care on Arenas' part in the performance of his duties. Even CBTL admitted in its notice to explain
that this violation does not merit yet a disciplinary action and is only an aggravating circumstance to Arenas' other
violations.|||
(C) For misconduct or improper behavior to be a just cause for dismissal, (a) it must be serious; (b) it must relate to the
performance of the employee's duties; and (c) it must show that the employee has become unfit to continue working
for the employer.
There was no active dishonesty on the part of Arenas. When questioned about who placed the bottled iced tea inside the
ice bin, his immediate reaction was not to deny his mistake, but to remove the bottle inside the bin and throw it
outside. Thus, even if there was an initial reticence on Arenas' part, his subsequent act of owing to his mistake only shows
the absence of a deliberate intent to lie or deceive his CBTL superiors. On this score, we conclude that Arenas' action did
not amount to serious misconduct.|||

30

MAERSK-FILIPINAS V. AVESTRUZ
G.R. NO. 207010
FEBRUARY 18, 2015
J. PERLAS BERNABE
INSUBORDINATION
FACTS:
On April 28, 2011, Maersk, on behalf of its foreign principal, petitioner A.P. Moller, hired Avestruz as Chief Cook on
board the vessel M/V Nedlloyd Drake for a period of six months, with a basic monthly salary of US$698.00. Avestruz
boarded the vessel on May 4, 2011.
On June 22, 2011, in the course of the weekly inspection of the vessel's galley, Captain Woodward noticed that the cover
of the garbage bin in the kitchen near the washing area was oily. As part of Avestruz's job was to ensure the cleanliness of
the galley, Captain Woodward called Avestruz and asked him to stand near the garbage bin where the former took the
latter's right hand and swiped it on the oily cover of the garbage bin, telling Avestruz to feel it. Shocked, Avestruz
remarked, "Sir if you are looking for dirt, you can find it; the ship is big. Tell us if you want to clean and we will clean
it." Captain Woodward replied by shoving Avestruz's chest, to which the latter complained and said, "Don't touch
me," causing an argument to ensue between them.|||
Later that afternoon, Captain Woodward required Avestruz to state in writing what transpired in the galley that morning.
Avestruz complied and submitted his written statement on that same day. On the very same day, Captain Woodward
informed Avestruz that he would be dismissed from service and be disembarked in India. On July 3, 2011, Avestruz was
disembarked in Colombo, Sri Lanka and arrived in the Philippines on July 4, 2011.|
Avestruz filed a complaint for illegal dismissal, payment for the unexpired portion of his contract, damages, and attorney's
fees against Maersk, A.P. Moller, and Jesus Agbayani, an officer of Maersk. In their defense, petitioners claimed that
during his stint on the vessel, Avestruz failed to attend to his tasks, specifically to maintain the cleanliness of the galley,
which prompted Captain Woodward to issue weekly reminders. Unfortunately, despite the reminders, Avestruz still failed
to perform his duties properly. As a result, Captain Woodward initiated disciplinary proceedings and informed Avestruz
during the hearing of the offenses he committed. hereafter, he was informed of his dismissal from service due to
insubordination. Relative thereto, Captain Woodward sent two e-mails to Maersk explaining the decision to terminate
Avestruz's employment and requesting for Avestruz's replacement.
The Labor Arbiter dismissed Avestruz's complaint for lack of merit.||| The NLRC sustained the validity of Avestruz's
dismissal but found that petitioners failed to observe the procedures laid down in Section 17 of the POEA-SEC. The CA
reversed and set aside the rulings of the NLRC and instead, found Avestruz to have been illegally dismissed.
Consequently, it directed petitioners to pay him, jointly and severally, the full amount of his placement fee and deductions
made, with interest at 12% per annum, as well as his salaries for the unexpired portion of his contract, and attorney's fees
of 10% of the total award. All other money claims were denied for lack of merit.|
ISSUE:
Whether or not the CA erred when it reversed and set aside the ruling of the NLRC finding that Avestruz was legally
dismissed and accordingly, dismissing the complaint, albeit with payment of nominal damages for violation of procedural
due process.||
HELD:
No. The Court finds that there was no just or valid cause for his dismissal, hence, he was illegally dismissed.
Insubordination, as a just cause for the dismissal of an employee, necessitates the concurrence of at least two requisites:
(1) the employee's assailed conduct must have been willful, that is, characterized by a wrongful and perverse attitude; and
(2) the order violated must have been reasonable, lawful, made known to the employee, and must pertain to the duties
which he had been engaged to discharge.|| The burden of proving that the termination of an employee was for a just or
authorized cause lies with the employer. If the employer fails to meet this burden, the conclusion would be that the
dismissal was unjustified and, therefore, illegal.|||
In this case, the contents of Captain Woodward's e-mails do not establish that Avestruz's conduct had been willful, or
characterized by a wrongful and perverse attitude.||| Conversely, apart from Captain Woodward's e-mails, no other
evidence was presented by the petitioners to support their claims.||| As in this case, it was incumbent upon the petitioners
to present other substantial evidence to bolster their claim that Avestruz committed acts that constitute insubordination as
would warrant his dismissal. At the least, they could have offered in evidence entries in the ship's official logbook
showing the infractions or acts of insubordination purportedly committed by Avestruz, the ship's logbook being the
official repository of the day-to-day transactions and occurrences on board the vessel. Having failed to do so, their
position that Avestruz was lawfully dismissed cannot be sustained.||
31

TENAZAS V. VILLEGAS TAXI TRANSPORT


G.R. NO. 192998
APRIL 2, 2014
J. REYES
ILLEGAL DISMISSAL; DOCTRINE OF STRAINED RELATIONS
FACTS:
Tenazas and Francisco filed a complaint for illegal dismissal against R. Villegas Taxi Transport and/or Romualdo Villegas
and Andy Villegas (respondents). At that time, a similar case had already been filed by Endraca against the same
respondents. The cases were subsequently consolidated.
Tenazas, Francisco and Endraca (petitioners) alleged that they were hired and dismissed by the respondents on the
following dates:
Name
Bernard A. Tenazas
Jaime M. Francisco
Isidro G. Endraca

Date of Hiring

Date of Dismissal

Salary

10/1997
04/10/04
04/2000

07/03/07
06/04/07
03/06/06

Boundary System
Boundary System
Boundary System

Tenazas alleged that on July 1, 2007, the taxi unit assigned to him was sideswiped by another vehicle. Upon reporting the
incident to the company, he was scolded by respondents Romualdo and Andy and was told to leave the garage for he is
already fired. Despite the warning, Tenazas reported for work on the following day but was told that he can no longer
drive any of the companys units as he is already fired.
Francisco, on the other hand, averred that his dismissal was brought about by the companys unfounded suspicion that he
was organizing a labor union.
Endraca, for his part, alleged that his dismissal was instigated by an occasion when he fell short of the required boundary
for his taxi unit.
The respondents alleged that Tenazas was never terminated by the company. They claimed that on July 3, 2007, Tenazas
went to the company garage to get his taxi unit but was informed that it is due for overhaul because of some mechanical
defects reported by the other driver who takes turns with him in using the same. He was thus advised to wait for further
notice from the company if his unit has already been fixed. however, upon being informed that his unit is ready for
release, Tenazas failed to report back to work for no apparent reason.
As regards Endraca, the respondents alleged that they hired him as a spare driver in February 2001. They allow him to
drive a taxi unit whenever their regular driver will not be able to report for work. In July 2003, however, Endraca stopped
reporting for work without informing the company of his reason.
The LA ruled that there was no illegal dismissal in the case at bar. The NLRC rendered a decision reversing the appealed
decision of the LA. The CA agreed with the NLRCs finding that Tenazas and Endraca were employees of the company,
but ruled otherwise in the case of Francisco for failing to establish his relationship with the company.
The award of Franciscos claims was deleted. The separation pay granted in favor of Tenazas and Endraca was likewise
deleted by the CA.
ISSUE:
Whether or not the CA erred in deleting (1) Franciscos claims, (2) Tenazas and Endracas separation pay.
HELD:
(1)

No. Pivotal to the resolution of the instant case is the determination of the existence of employer-employee
relationship and whether there was an illegal dismissal. In determining the presence or absence of an
employer-employee relationship, the Court has consistently looked for the following incidents, to wit: (a) the
selection and engagement of the employee; (b) the payment of wages; (c) the power of dismissal; and (d) the
employers power to control the employee on the means and methods by which the work is accomplished.
The last element, the so-called control test, is the most important element.
With the respondents denial, the burden of proof shifts to Francisco to establish his regular employment.
Unfortunately, the LA found that Francisco failed to present sufficient evidence to prove regular employment
such as company ID, SSS membership, withholding tax certificates or similar articles. Thus, he was not
considered an employee of the company. Short of the required quantum of proof, the CA correctly ruled that
32

the NLRCs finding of illegal dismissal and the monetary awards which necessarily follow such ruling lacked
factual and legal basis and must therefore be deleted.
(2)

No. An illegally dismissed employee is entitled to two reliefs: backwages and reinstatement. The two reliefs
provided are separate and distinct. In instances where reinstatement is no longer feasible because of strained
relations between the employee and the employer, separation pay is granted. Clearly, it is only when
reinstatement is no longer feasible that the payment of separation pay is ordered in lieu thereof.
This doctrine of strained relations, however, should not be used recklessly or applied loosely nor be based on
impression alone. After a perusal of the NLRC decision, this Court failed to find the factual basis of the award
of separation pay to the petitioners. The NLRC decision did not state the facts which demonstrate that
reinstatement is no longer a feasible option that could have justified the alternative relief of granting
separation pay instead.
A bare claim of strained relations by reason of termination is insufficient to warrant the granting of separation
pay. Likewise, the filing of the complaint by the petitioners does not necessarily translate to strained relations
between the parties. As a rule, no strained relations should arise from a valid and legal act asserting ones
right

BLUER THAN BLUE JOINT VENTURES V. ESTEBAN


G.R. NO. 192582
APRIL 7, 2014
J. REYES
LOSS OF TRUST AND CONFIDENCE; ILLEGAL DISMISSAL
FACTS:
Respondent Glyza Esteban was employed in January 2004 as Sales Clerk, and assigned at Bluer Than Blue Joint Ventures
Companys EGG boutique in SM Marilao beginning the year 2006.
In November 2006, the petitioner received a report that several employees have access to its point-of-sale (POS) system
through a universal password given by Elmer Flores. Upon investigation, it was discovered that it was Esteban who gave
Flores the password.
The petitioner sent a letter memorandum to Esteban on November 8, 2006, asking her to explain in writing why she
should not be disciplinary dealt with for tampering with the companys POS system through the use of an unauthorized
password. Esteban was also placed under preventive suspension for ten days. In her explanation, Esteban admitted that
she used the universal password three times on the same day in December 2005, after she learned of it from two other
employees who she saw browsing through the petitioners sales inquiry.
On November 13, 2006, Estebans preventive suspension was lifted, but at the same time, a notice of termination was sent
to her, finding her explanation unsatisfactory and terminating her employment immediately on the ground of loss of trust
and confidence. Esteban was given her final pay, including benefits and bonuses, less inventory variances incurred by the
store amounting to P8,304.93. Esteban signed a quitclaim and release in favor of the petitioner.
The LA ruled in favor of Esteban and found that she was illegally dismissed. The LA also awarded separation pay,
backwages, unpaid salary during her preventive suspension and attorneys fees. The NLRC reversed the decision of the
LA and dismissed the case for illegal dismissal. The CA granted Estebans petition and reinstated the LA decision.
ISSUE:
Whether Estebans acts constitute just cause to terminate her employment with the company on the ground of loss of trust
and confidence.
HELD:
No. Loss of trust and confidence is premised on the fact that the employee concerned holds a position of responsibility,
trust and confidence. The employee must be invested with confidence on delicate matters, such as the custody, handling,
care and protection of the employers property and funds. Esteban is, no doubt, a rank-and-file employee. Among the
fiduciary rank-and-file employees are cashiers, auditors, property custodians, or those who, in the normal exercise of their
functions, regularly handle significant amounts of money or property. These employees, though rank-and-file, are
routinely charged with the care and custody of the employers money or property, and are thus classified as occupying
positions of trust and confidence.
In this case, Esteban was a sales clerk. Her duties, however, were more than that of a sales clerk. Aside from attending to
customers and tending to the shop, Esteban also assumed cashiering duties.
33

In this case, the Court finds that the acts committed by Esteban do not amount to a wilful breach of trust. The petitioner
also failed to establish a substantial connection between Estebans use of the password and any loss suffered by the
petitioner.
The Court is not saying that Esteban is innocent of any breach of company policy. That she relayed the password to
another employee is likewise demonstrative of her mindless appreciation of her duties as a sales clerk in the petitioners
employ. But absent any showing that her acts were done with moral perverseness that would justify the claimed loss of
trust and confidence attendant to her job, the Court must sustain the conclusion that Esteban was illegally dismissed.
ST. LUKES MEDICAL CENTER V. SANCHEZ
G.R. NO. 212054
MARCH 11, 2015
J. PERLAS BERNABE
DISMISSAL; CONVICTION NOT NECESSARY TO FIND JUST CAUSE IN TERMINATION OF EMPLOYMENT
FACTS:
Sanchez was hired by petitioner St. Luke's Medical Center, Inc. (SLMC) as a Staff Nurse until her termination on July 6,
2011 for her purported violation of SLMC's Code of Discipline, particularly Section 1, Rule 1 on Acts of Dishonesty, i.e.,
Robbery, Theft, Pilferage, and Misappropriation of Funds.|||
Sanchez passed through the SLMC Centralization Entrance/Exit where she was subjected to the standard inspection
procedure by the security personnel. In the course thereof, the Security Guard on-duty noticed a pouch in her bag and
asked her to open the same. When opened, said pouch contained assortment of medical stocks. An initial investigation
was also conducted by the SLMC Division of Nursing which thereafter served Sanchez a notice to explain. Sanchez
submitted an Incident Report Addendum, explaining that the questioned items came from the medication drawers of
patients who had already been discharged. Eventually, she forgot about the same as she got caught up in work, until it was
noticed by the guard on duty on her way out of SMLC's premises.|||
Consequently, Sanchez was placed under preventive suspension effective June 3, 2011 until the conclusion of the
investigation by SLMC's Employee and Labor Relations Department which, thereafter, required her to explain why she
should not be terminated from service for "acts of dishonesty".
After hearing her side, SLMC, on July 4, 2011, informed Sanchez of its decision to terminate her employment effective
closing hours of July 6, 2011. This prompted her to file a complaint for illegal dismissal before the NLRC. In her position
paper, Sanchez maintained her innocence, claiming that she had no intention of bringing outside the SLMC's premises the
questioned items. She also stressed the fact that SLMC did not file any criminal charges against her.
The Labor Arbiter ruled that Sanchez was validly dismissed. SLMC's non-filing of a criminal case against Sanchez did not
preclude a determination of her serious misconduct, considering that the filing of a criminal case is entirely separate and
distinct from the determination of just cause for termination of employment. The NLRC reversed and set aside the LA
ruling, and held that Sanchez was illegally dismissed. The CA upheld the NLRC ruling.
ISSUE:
Whether Sanchez was illegally dismissed.
HELD:
No. Among the employer's management prerogatives is the right to prescribe reasonable rules and regulations necessary
or proper for the conduct of its business or concern, to provide certain disciplinary measures to implement said rules and
to assure that the same would be complied with. For an employee to be validly dismissed on this ground, the employer's
orders, regulations, or instructions must be: (1) reasonable and lawful, (2) sufficiently known to the employee, and
(3) in connection with the duties which the employee has been engaged to discharge.
The Court finds that Sanchez was validly dismissed by SLMC for her willful disregard and disobedience of Section 1,
Rule I of the SLMC Code of Discipline, which reasonably punishes acts of dishonesty. As it is clear that the company
policies subject of this case are reasonable and lawful, sufficiently known to the employee, and evidently connected with
the latter's work, the Court concludes that SLMC dismissed Sanchez for a just cause.
Neither is SLMC's non-filing of the appropriate criminal charges relevant to this analysis. An employee's guilt or
innocence in a criminal case is not determinative of the existence of a just or authorized cause for his or her dismissal. It is
well-settled that conviction in a criminal case is not necessary to find just cause for termination of employment. Finally,
the Court finds it inconsequential that SLMC has not suffered any actual damage. While damage aggravates the charge, its
absence does not mitigate nor negate the employee's liability.||
34

SPI TECHNOLOGIES V. MAPUA


G.R. NO. 191154
APRIL 7, 2014
J. REYES
REDUNDANCY
FACTS:
Victoria Mapua alleged that she was hired in 2003 by SPI as the Corporate Developments Research/Business Intelligence
Unit Head and Manager of the company. Subsequently in August 2006, then Vice President and Corporate Development
Head, Peter Maquera hired Elizabeth Nolan as Mapuas supervisor.
Sometime in October 2006, the hard disk on Mapuas laptop crashed, causing her to lose files and data. Mapua informed
Nolan and her colleagues that she was working on recovering the lost data and asked for their patience for any possible
delay on her part in meeting deadlines. On November 13, 2006, Mapua retrieved the lost data with the NBIs assistance.
Yet, Nolan informed Mapua that she was realigning Mapuas position to become a subordinate of co-manager Sameer
Raina due to her missing a work deadline.
In December 2006, Mapua noticed that her colleagues began to ostracize and avoid her. Nolan and Raina started giving
out majority of her research work and other duties under Healthcare and Legal Division to the rank-and-file staff. Mapua
lost about 95% of her work projects and job responsibilities. On February 28, 2007, Mapua allegedly saw the new table of
organization of the Corporate Development Division which would be renamed as the Marketing Division. The new
structure showed that Mapuas level will be again downgraded because a new manager will be hired and positioned
between her rank and Rainas.
On March 21, 2007, Raina informed Mapua over the phone that her position was considered redundant and that she is
terminated from employment effective immediately. Villanueva notified Mapua that she should cease reporting for work
the next day. Her laptop computer and company mobile phone were taken right away and her office phone ceased to
function.
On May 13, 2007, a recruitment advertisement of SPI was published in the Philippine Daily Inquirer. It listed all
vacancies in SPI, including a position for Marketing Communications Manager under Corporate Support the same group
where Mapua previously belonged. Mapua was convinced that her former position is not redundant.
ISSUE:
Whether Mapua may be dismissed on the ground of redundancy.
HELD:
No. ART. 283. Closure of establishment and reduction of personnel. The employer may also terminate the employment
of any employee due to installation of labor-saving devices, redundancy, retrenchment to prevent losses or the closing or
cessation of operation of the establishment or undertaking unless the closing is for the purpose of circumventing the
provisions of this Title, by serving a written notice on the worker and the Department of Labor and Employment at least
one (1) month before the intended date thereof. In case of termination due to installation of labor-saving devices or
redundancy, the worker affected thereby shall be entitled to a separation pay equivalent to at least one (1) month pay or to
at least one (1) month pay for every year of service, whichever is higher. In case of retrenchment to prevent losses and in
cases of closures or cessation of operations of establishment or undertaking not due to serious business losses and
financial reverses, the separation pay shall be equivalent to one (1) month pay or at least one-half () month pay for every
year of service, whichever is higher. A fraction of at least six (6) months shall be considered as one (1) whole year.
A position cannot be abolished by a mere change of job title. In cases of redundancy, the management should adduce
evidence and prove that a position which was created in place of a previous one should pertain to functions which are
dissimilar and incongruous to the abolished office.
While SPI had no legal duty to hire Mapua as a Marketing Communications Manager, it could have clarified why she is
not qualified for that position. In fact, Mapua brought up the subject of transfer to Villanueva and Raina several times
prior to her termination but to no avail. There was even no showing that Mapua could not perform the duties of a
Marketing Communications Manager. Also connected with the evidence negating redundancy was SPIs publication of job
vacancies after Mapua was terminated from employment.
RIO V. COLEGIO DE STA. ROSA-MAKATI
G.R. NO. 189629
AUGUST 6, 2014

35

J. PEREZ
GROSS NEGLECT OF DUTY; GROSS NEGLIGENCE
FACTS:
In a letter dated 30 July 2002, respondent Gustilo charged petitioner and Mrs. Alonzo, the school nurse, of grave
misconduct, dishonesty and/or gross neglect of duty detrimental not only to the school but, principally, to the health and
well-being of the pupils.
Petitioner was made to answer for the following: (1) 9 students have medical records for school years during which they
were not in the school yet, thus could not have been the subject of medical examination/evaluation; (2) 79 students of
several classes during certain school years were not given any medical/health evaluation/examination; and (3) failure to
conduct medical/health examination on all students of several classes of different grade levels for the school year 20012002.
To investigate the charges against petitioner, respondent Gustilo created an investigation committee. Upon the
recommendation of the investigation committee, the services of petitioner and Alonzo were terminated for their grave
misconduct, dishonesty and gross neglect of duty.
Petitioner denied the charges and thereafter filed a complaint for constructive dismissal and illegal suspension against
respondents. The LA ruled in favor of petitioner and Alonzo, declaring that they were illegally dismissed. The NLRC
reversed the ruling of the Labor Arbiter and likewise denied petitioners subsequent motion for reconsideration on 7 April
2005. Aggrieved, petitioner filed a Petition for Certiorari with the CA, which the CA denied.
ISSUE:
Whether or not the NLRC committed grave abuse of discretion in reversing the ruling of the Labor Arbiter.
HELD:
No. Gross neglect of duty or gross negligence refers to negligence characterized by the want of even slight care, acting or
omitting to act in a situation where there is a duty to act, not inadvertently but willfully and intentionally, with a conscious
indifference to consequences insofar as other persons may be affected.
As borne by the records, petitioners actions fall within the purview of the above-definitions. Petitioner failed to diligently
perform her duties. It was unrefuted that: (1) there were dates when a medical examination was supposed to have been
conducted and yet the dates fell on weekends; (2) failure to conduct medical examination on all students for two (2) to
five (5) consecutive years; (3) lack of medical records on all students; and (4) students having medical records prior to
their enrollment.
WESLEYAN UNIVERSITY V. REYES
G.R. NO. 208321
JULY 30, 2014
J. VELASCO
LOSS OF TRUST AND CONFIDENCE
FACTS:
On March 16, 2004, respondent Nowella Reyes was appointed as WUPs University Treasurer on probationary basis. A
little over a year after, she was appointed as full time University Treasurer.
Discovered following an audit were irregularities in the handling of petitioners finances, mainly, the encashment by its
Treasury Department of checks issued to WUP personnel, a practice purportedly in violation of the imprest system of cash
management, and the encashment of various crossed checks payable to the University Treasurer by Chinabank despite
managements intention to merely have the funds covered thereby transferred from one of petitioners bank accounts to
another. As a result of said audit, petitioner served respondent a Show Cause Order and placed her under preventive
suspension.
Finding respondents Explanation unsatisfactory, the HRDO, recommendrd respondents dismissal as University
Treasurer. Upon receipt of her notice of termination on July 9, 2009, respondent filed a complaint for illegal dismissal
with the Arbitration Branch of the National Labor Relations Commission. She contended that her dismissal was illegal,
void and unjust, for the following reasons:

36

First, her 60-day preventive suspension violated the Labor Code provisions prohibiting such suspensions to last for more
than thirty (30) days. Second, there was a violation of her right to substantive and procedural due process. Finally, the
charges against her were based on mere suspicion and speculations and unsupported by evidence.
Petitioner, for its part, predicated its defense on the contention that respondent was a highly confidential employee who
handled significant amounts of money as University Treasurer and that the irregularities attributed to her in the
performance of her duties justify her dismissal on the basis of loss of trust and confidence.
The Labor Arbiter rendered a Decision finding for respondent. The Labor Artbiter noted, as respondent has insisted, that
the charges against the latter were based on mere rumors and speculations.
Petitioner filed an appeal with the NLRC which was granted in the tribunals Decision dated July 11, 2011, declaring that
respondent was legally dismissed. However, petitioner was ordered to pay respondent her proportionate 13th month pay,
the monetary value of her vacation leave, and attorneys fees. The NLRC found petitioners contention of loss of trust and
confidence in respondent with sufficient basis. While respondent, so the NLRC notes, may not have been guilty of willful
breach of trust, the fact that she held a highly confidential position, and considering that anomalous transactions transpired
under her command responsibility, provided petitioner with ample ground to distrust and dismiss her.
The NLRC denied respondents motion for reconsideration. Therefrom, respondent went on Certiorari to the CA. The CA
reversed and set aside the decision of the NLRC.
ISSUE:
Whether or not the CA erred in finding respondent illegally dismissed by petitioner on the ground of loss of trust and
confidence.
HELD:
Yes. The requisites of a valid dismissal based on loss of trust and confidence are as follows:
The first requisite is that the employee concerned must be one holding a position of trust and confidence, thus, one who is
either: (1) a managerial employee; or (2) a fiduciary rank-and-file employee, who, in the normal exercise of his or her
functions, regularly handles significant amounts of money or property of the employer. The second requisite is that the
loss of confidence must be based on a willful breach of trust and founded on clearly established facts.
There is no doubt that respondent held a position of trust; thus, greater fidelity is expected of her. She was not an ordinary
rank-and-file employee but an employee occupying a very sensitive position. As University Treasurer, she handled and
supervised all monetary transactions and was the highest custodian of funds belonging to WUP.
The presence of the first requisite is certain. So is as regards the second requisite. Indeed, the Court finds that petitioner
adequately proved respondents dismissal was for a just cause, based on a willful breach of trust and founded on clearly
established facts as required by jurisprudence. At the end of the day, the question of whether she was a managerial or
rank-and-file employee does not matter in this case because not only is there basis for believing that she breached the trust
of her employer, her involvement in the irregularities attending to petitioners finances has also been proved.
AM-PHIL FOOD CONCEPTS V. PADILLA
G.R. NO. 188753
OCTOBER 1, 2014
J. LEONEN
RETRENCHMENT
FACTS:
Padilla was hired on April 1, 2002 as a Marketing Associate by Am-Phil, a corporation engaged in the restaurant business.
On September 29, 2002, Am-Phil sent Padilla a letter confirming his regular employment. Sometime in the first week of
March 2004, Am-Phils officers informed Padilla that Am-Phil would be implementing a retrenchment program that
would be affecting 3 of its employees including Padilla. The retrenchment program was allegedly on account of serious
and adverse business conditions, i.e., lack of demand in the market, stiffer competition, devaluation of the Philippine peso,
and escalating operation costs.
Padilla questioned Am-Phils choice to retrench him. He pointed out that Am-Phil was actually then still hiring new
employees. He also noted that Am-Phil's sales have not been lower relative to the previous year. In response, Am-Phil's
officers gave him two options: (1) be retrenched with severance pay or (2) be transferred as a waiter in Am-Phils
restaurant, a move that entailed his demotion.
37

On March 17, 2004, Am-Phil sent Padilla a memorandum notifying him of his retrenchment. Padilla was paid separation
pay. On July 28, 2004, Padilla filed the complaint for illegal dismissal (with claims for backwages, damages, and
attorneys fees).
For its defense, Am-Phil claimed that Padilla was not illegally terminated and that it validly exercised a management
prerogative. It asserted that Padilla was hired merely as part of an experimental marketing program. It added that in
2003, it did suffer serious and adverse business losses and that, in the first quarter of 2004, it was compelled to retrench
employees so as to avoid further losses. Am-Phil also underscored that Padilla executed a quitclaim and release in its
favor.
Labor Arbiter Chuanico rendered the decision finding that Padilla was illegally dismissed. He noted that Am-Phil failed
to substantiate its claim of serious business losses and that it failed to comply with the procedural requirement for a proper
retrenchment (i.e., notifying the Department of Labor and Employment). He also held that the quitclaim and release
executed by Padilla is contrary to law.
Am-Phil filed an appeal with the NLRC. Apart from asserting its position that Padilla was validly retrenched, Am-Phil
claimed that Labor Arbiter Chuanico was in error in deciding the case despite the pendency of its motion for leave to file
supplemental rejoinder. Through this supplemental rejoinder, Am-Phil supposedly intended to submit its audited financial
statements for the years 2001 to 2004 and, thereby, prove that it had suffered business losses. Am-Phil claimed that its
right to due process was violated by Labor Arbiter Chuanicos refusal to consider its 2001 to 2004 audited financial
statements. On February 28, 2007, the NLRC issued the resolution affirming Labor Arbiter Chuanicos ruling.
With respect to Am-Phils claim that Labor Arbiter Chuanico erroneously ignored its 2001 to 2004 audited financial
statements, the NLRC noted that a supplemental rejoinder was not a necessary pleading in proceedings before labor
arbiters. The NLRC denied Am-Phils motion for reconsideration. Am-Phil then filed with the CA a petition for certiorari.
The CA rendered dismissed Am-Phils petition for certiorari.
ISSUE:
(1) Whether Padila was dismissed through a valid retrenchment implemented by petitioner Am-Phil.
(2) Whether it was proper for Labor Arbiter Chuanico to have ruled that Padilla was illegally dismissed
despite Am-Phils pending motion for leave to file supplemental rejoinder.
HELD:
(1) No. For a valid retrenchment, each of which must be shown by clear and convincing evidence, as follows:
a. that the retrenchment is reasonably necessary and likely to prevent business losses which, if
already incurred, are not merely de minimis, but substantial, serious, actual and real, or if only
expected, are reasonably imminent as perceived objectively and in good faith by the employer;
b. that the employer served written notice both to the employees and to the Department of Labor
and Employment at least one month prior to the intended date of retrenchment;
c. that the employer pays the retrenched employees separation pay equivalent to one month pay
or at least month pay for every year of service, whichever is higher;
d. that the employer exercises its prerogative to retrench employees in good faith for the
advancement of its interest and not to defeat or circumvent the employees right to security of
tenure; and
e. that the employer used fair and reasonable criteria in ascertaining who would be dismissed
and who would be retained among the employees, such as status (i.e., whether they are temporary,
casual, regular or managerial employees), efficiency, seniority, physical fitness, age, and financial
hardship for certain workers.
There is no clear and convincing evidence to sustain the substantive ground on which the supposed validity of
Padillas retrenchment rests. Am-Phils 2001 to 2004 audited financial statements, the sole proof upon which AmPhil relies on to establish its claim that it suffered business losses, have been deemed unworthy of
consideration. Moreover, it is admitted that Am-Phil did not serve a written notice to the Department of Labor and
Employment one (1) month before the intended date of Padillas retrenchment
(2) Yes. From the provisions of the 2002 Rules, a supplemental rejoinder, as correctly ruled by the NLRC, is not a
pleading which a labor arbiter is duty-bound to accept. Even following changes to the NLRC Rules of Procedure
in 2005 and 2011, a rejoinder has not been recognized as a pleading that labor arbiters must necessarily admit.
The 2005 and 2011 National Labor Relations Commission Rules of Procedure only go so far as to recognize that a
reply may be filed by the parties.
Thus, Labor Arbiter Chuanico was under no obligation to grant Am-Phils motion for leave to admit supplemental
rejoinder and, thereby, consider the supplemental rejoinders averments and annexes. That Am-Phil had to file a
motion seeking permission to file its supplemental rejoinder (i.e., motion for leave to file) is proof of its own
38

recognition that the labor arbiter is under no compulsion to accept any such pleading and that the supplemental
rejoinders admission rests on the labor arbiters discretion.
MOUNT CARMEL COLLEGE EMPLOYEES UNION (MCCEU) V. MOUNT CARMEL COLLEGE
G.R. NO. 187621
SEPTEMBER 24, 2014
J. REYES
APPEAL; RETRENCHMENT
FACTS:
The petitioners were elementary and high school academic and non-academic personnel employed by respondent. In
April 1999, the petitioners were informed of their retrenchment by the respondent due to the closure of the elementary and
high school departments of the school. The petitioners contend that such closure was merely a subterfuge of their
termination due to their union activities. According to the petitioners, they organized a union in 1997 (Mount Carmel
College Employees Union [MCCEU]), and were in the process of negotiating with the respondent as regards their
collective bargaining agreement when the respondent decided to close the two departments in June 1999. The petitioners
alleged that such closure was motivated by ill-will just to get rid of the petitioners who were all union members because in
June 2001, the school re-opened its elementary and high school departments with newly-hired teachers. They claimed for
the remaining separation pay differentials since what they received was only computed at 15 days for every year of
service when they were retrenched.
The respondent denied committing any act of unfair labor practice and alleged that their retrenchment was valid as it was
due to the financial losses it suffered as result of a decline in its enrolment.
The LA declared the petitioners to have been illegally dismissed. According to the LA, the respondents alleged losses
were not serious as its financial statements even showed a net surplus. Thus, the LA ordered the respondent to pay the
petitioners separation pay in lieu of reinstatement, plus attorneys fees.
Aggrieved, the respondent appealed to the NLRC. The NLRC reversed the LA decision, ruling that: (1) the respondents
failure to attach a copy of the appeal bond and other documents to the Appeal Memorandum furnished to the petitioners is
a minor defect; (2) the respondent acted in good faith when it procured the appeal bond from CBIC, which, it turned out,
was blacklisted at that time and since CBIC was already included in the list of the Supreme Courts accredited bonding
companies from February 1, 2005 until July 31, 2005, there is no more impediment for CBIC to make good its bond;
and (3) the petitioners retrenchment is an exercise by the respondent of its management prerogative and the latters state
of finances justifies the same.
The CA did not find any grave abuse of discretion committed by the NLRC and thus, affirmed its decision.
ISSUE:
(1)Whether the CA correctly ruled that the NLRC did not commit grave abuse of discretion when it allowed the
respondents appeal despite the blacklisting of CBIC at the time it issued the appeal bond.
(2)Whether the CA erred when it ruled that the NLRC did not commit grave abuse of discretion in finding that
petitioners retrenchment was valid.
HELD:
(1) No. Section 6 requiring the issuance of a bond by a reputable bonding company duly accredited by the
NLRC or the Supreme Court was substantially carried over to the 2005 Revised Rules of Procedure of the
NLRC and the 2011 NLRC Rules of Procedure. In this regard, the Court has ruled that in a judgment
involving a monetary award, the appeal shall be perfected only upon: (1) proof of payment of the required
appeal fee; (2) posting of a cash or surety bond issued by a reputable bonding company; and (3)
filing of a memorandum of appeal.
In this case, it was not disputed that at the time CBIC issued the appeal bond, it was already blacklisted
by the NLRC. The latter, however, opined that respondents acted in good faith when they transacted
with the bonding company for the issuance of the surety bond. Good faith, however, is not an excuse for
setting aside the mandatory and jurisdictional requirement of the law. Also, the lifting of CBICs
blacklisting on January 24, 2005 does not render the bond it issued on March 15, 2004 subsequently
valid. It should be stressed that what the law requires is that the appeal bond must be issued by a
reputable bonding company duly accredited by the NLRC or the Supreme Court at the time of the filing
of the appeal.
39

(2) Yes. In termination by retrenchment, not every loss incurred or expected to be incurred by an employer
can justify retrenchment. The employer must prove, among others, that the losses are substantial and that
the retrenchment is reasonably necessary to avert such losses.
In this case, while the respondent may have presented its Financial Statements, the respondent,
nevertheless, failed to establish with reasonable certainty that the proportion of its revenues are largely
expended for its elementary and high school personnel salaries, wages and other benefits. The Financial
Statements pertain to its assets, liabilities, gross revenues and expenses for the entire college system, that
is, from elementary, high school to the college department. The expenses for the elementary and high
school departments were not set out in detail and instead, were lumped together with the college
department. Such detail becomes material in the light of the respondents claim that the personnel
expenses for the elementary and high school departments were eating into the portion of its budget
allocated for other purposes. There could be no practical basis from which the respondents claim finds
support.
HOCHENG PHILIPPINES CORPORATION V. FARRALES
G.R. NO. 211497
MARCH 18, 2015
J. REYES
SERIOUS MISCONDUCT; ILLEGAL DISMISSAL
FACTS:
A report reached HPC management that a motorcycle helmet of an employee, Reymar, was stolen at the parking lot within
its premises on November 27, 2009. On December 3, 2009, Security Officer Paragas confirmed a video sequence recorded
on CCTV around 3:00 p.m. on November 27, 2009 showing Farrales taking the missing helmet from a parked motorcycle.
Later that day, HPC sent Farrales a notice to explain his involvement in the alleged theft. A hearing was held on December
10, 2009 at 1:00 p.m. Present were Farrales, Eric, Andy, Jun Reyes, Antonio Alinda, a witness, and Rolando Garciso,
representing ULO-Hocheng. From Andy it was learned that at the time of the alleged incident, he was already seated on
his motorcycle and about to leave the company compound when Farrales approached and asked him to hand to him a
yellow helmet hanging from a motorcycle parked next to him. When Andy hesitated, Farrales explained that he owned it,
and so Andy complied. But Eric had specifically told Farrales that his helmet was colored red and black and his
motorcycle was a black Honda XRM-125 with plate number 8746-DI, parked near the perimeter fence away from the
walkway to the pedestrian gate. The CCTV showed Farrales instructing Andy to fetch a yellow helmet from a blue Rossi
110 motorcycle with plate number 3653-DN parked in the middle of the parking lot, opposite the location given by Eric.
On his part, Farrales explained that upon seeing Eric in the workplace, he asked him why he did not get the helmet from
his house. Eric told him that, Hindi po sa akin yung nakuha nyong helmet. Farrales was shocked and he immediately
phoned the HPCs guard to report the situation that he mistook the helmet which he thought belonged to Eric. After
several employees were asked as to the ownership of the helmet, he finally found the owner thereof. He promptly
apologized to Jun and undertook to return the helmet the following day and explained that it was an honest mistake.
On February 15, 2010, the HPC issued a Notice of Termination to Farrales dismissing him for violation of Article 69,
Class A, Item No. 29 of the HPC Code of Discipline, which provides that stealing from the company, its employees and
officials, or from its contractors, visitors or clients, is akin to serious misconduct and fraud or willful breach by the
employee of the trust reposed in him by his employer or duly authorized representative, which are just causes for
termination of employment under Article 282 of the Labor Code.
The LA ruled in favor of Farrales but the NLRC reversed the LA. The CA agreed with the LA that Farrales act of taking
Reynars helmet did not amount to theft, holding that HPC failed to prove that Farrales conduct was induced by a
perverse and wrongful intent to gain, in light of the admission of Eric that he did let Farrales borrow one of his two
helmets, only that Farrales mistook Reymars helmet as the one belonging to him.
ISSUE:
Whether there was just cause for Farrales termination.
HELD:
None. Theft committed by an employee against a person other than his employer, if proven by substantial evidence, is a
cause analogous to serious misconduct. Misconduct is improper or wrong conduct, it is the transgression of some
established and definite rule of action, a forbidden act, a dereliction of duty, willful in character, and implies wrongful
intent and not mere error in judgment. The misconduct to be serious must be of such grave and aggravated character and
40

not merely trivial or unimportant. Such misconduct, however serious, must, nevertheless, be in connection with the
employees work to constitute just cause for his separation.
Article 4 of the Labor Code mandates that all doubts in the implementation and interpretation of the provisions thereof
shall be resolved in favor of labor. The penalty imposed on the erring employee ought to be proportionate to the offense,
taking into account its nature and surrounding circumstances.
Farrales committed no serious or willful misconduct or disobedience to warrant his dismissal. It is not disputed that
Farrales lost no time in returning the helmet to Reymar the moment he was apprised of his mistake by Eric, which proves,
according to the CA, that he was not possessed of a depravity of conduct as would justify HPCs claimed loss of trust in
him. Farrales immediately admitted his error to the company guard and sought help to find the owner of the yellow
helmet, and this, the appellate court said, only shows that Farrales did indeed mistakenly think that the helmet he took
belonged to Eric. If doubts exist between the evidence presented by the employer and that of the employee, the scales of
justice must be tilted in favor of the latter. The employer must affirmatively show rationally adequate evidence that the
dismissal was for a justifiable cause.
INTERNATIONAL SCHOOL OF MANILA V. INTERNATIONAL SCHOOL ALLIANCE OF EDUCATORS
G.R. NO. 167286
FEBRUARY 5, 2014
J. DE CASTRO
TERMINATION; GROSS INEFFICIENCY; SEPARATION PAY
FACTS:
Santos was first hired by the School in 1978 as a full-time Spanish language teacher. In April 1992, Santos filed for and
was granted a leave of absence for the school year 1992-1993. She came back from her leave of absence sometime in
August 1993. Upon Santoss return to the School, only one class of Spanish was available for her to teach. Thus, for the
school year 1993-1994, Santos agreed to teach one class of Spanish and four other classes of Filipino that were left behind
by a retired teacher.
Santos was evaluated in the areas of Planning, the Teaching Act, Climate, Management and Communication. Santos was
also noted as needing improvement in the following criteria: (1) uses effective questioning techniques; (2) is punctual and
time efficient; (3) states and enforces academic and classroom behavior expectations in a positive manner; and (4)
reinforces appropriate behavior.
On January 17, 1994, Santos submitted to the Personnel Department of the School a memorandum/form,which stated her
assignment preference for the school year 1994-1995. She indicated therein that she planned to return to the School staff
for the said school year and she did not prefer a change of teaching assignment. For the school year 1994-1995, Santos
agreed to teach five classes of Filipino.
In his memo dated January 24, 1997, Loy made known his apparent frustration at Santoss performance. Other than the
substandard lessons, Loy commented that there was virtually no written work nor adequate direction in her syllabus. Loy
also warned her that continuance in this manner without marked improvement cannot be tolerated.
Subsequently, on April 10, 1997, McCauley sent a letter to Santos directing her to explain in writing why her employment
from the School should not be terminated because of her failure to meet the criteria for improvement set out in her
Professional Growth Plan and her substandard performance as a teacher.
The charge against Santos was gross inefficiency or negligence in the performance of her assigned work. After the parties
made known their positions, the investigating committee informed Santos and Ching that they would consider the views
presented and they would advise Santos of the Schools action on her case.
In a letter dated May 29, 1997, McCauley informed Santos that he was adopting the recommendation of the investigation
committee that Santoss employment from the School cannot be continued.
ISSUE:
Whether or not Santos was illegally terminated from her employment.
HELD:
No. For termination of employment based on just causes as defined in Article 282 of the Labor Code:
(i) A written notice served on the employee specifying the ground or grounds for termination, and giving said employee
reasonable opportunity within which to explain his side.
41

(ii) A hearing or conference during which the employee concerned, with the assistance of counsel if he so desires is given
opportunity to respond to the charge, present his evidence, or rebut the evidence presented against him.
(iii) A written notice of termination served on the employee, indicating that upon due consideration of all the
circumstances, grounds have been established to justify his termination.
In this case, the School complied with the above requirements. After a thorough evaluation of Santoss performance, the
School held a series of conferences and meetings with Santos, in order to improve her performance. On March 29, 1996,
the School required Santos to undertake a Professional Growth Plan. On April 21, 1997, McCauley wrote Santos a letter
informing her that an administrative investigation would be conducted on April 23, 1997 where she would be given the
opportunity to be heard. On April 23, 1997, an administrative investigation was conducted. Santos appeared therein with
the assistance of ISAE President Ching. In a letter dated May 29, 1997, the School informed Santos of its decision to
terminate her employment on the ground of her failure to meet the standards of the School, which as discussed was
tantamount to gross inefficiency.
In view of the acts and omissions of Santos that constituted gross inefficiency, the Court finds that the School was
justified in not keeping her in its employ. At this point, the Court needs to stress that Santos voluntarily agreed to teach the
Filipino classes given to her when she came back from her leave of absence. Said classes were not forced upon her by the
School. Thus, when she consented to take on the Filipino classes, it was Santoss responsibility to teach them well within
the standards of teaching required by the School, as she had done previously as a teacher of Spanish. Failing in this, she
must answer for the consequences.
It is the prerogative of the school to set high standards of efficiency for its teachers since quality education is a mandate of
the Constitution. As long as the standards fixed are reasonable and not arbitrary, courts are not at liberty to set them aside.
ISSUE:
Whether or not Santos is entitled to separation pay.
HELD:
Yes. In view of the finding that Santos was validly dismissed from employment, she would not ordinarily be entitled to
separation pay. An exception to this rule is when the court finds justification in applying the principle of social justice
according to the equities of the case.
In the instant case, the Court finds equitable and proper the award of separation pay in favor of Santos in view of the
length of her service with the School prior to the events that led to the termination of her employment. To recall, Santos
was first employed by the School in 1978 as a Spanish language teacher. During this time, the records of this case are
silent as to the fact of any infraction that she committed and/or any other administrative case against her that was filed by
the School. Thus, an award of separation pay equivalent to one-half (1/2) month pay for every year of service is awarded
in favor of Santos on grounds of equity and social justice.
GMA V. PABRIGA
G.R. NO. 176419

NOVEMBER 27, 2013

J. DE CASTRO
REGULAR EMPLOYMENT
FACTS:
Private respondents were engaged by petitioner as Television Technicians. On July 19, 1999, due to the miserable working
conditions, private respondents were forced to file a complaint against petitioner before NLRC Cebu.
On 4 August 1999, petitioner received a notice of hearing of the complaint. The following day, petitioners Engineering
Manager, Roy Villacastin, confronted the private respondents about the said complaint. On 9 August 1999, private
respondents were summoned to the office of petitioners Area Manager, Mrs. Susan Alio, and they were made to explain
why they filed the complaint. The next day, private respondents were barred from entering and reporting for work without
any notice stating the reasons.
On 23 August 1999, a reply letter from Mr. Bienvenido Bustria, petitioners head of Personnel and Labor Relations
Division, admitted the non-payment of benefits but did not mention the request of private respondents to be allowed to
return to work. On 8 October 1999, private respondents filed an amended complaint raising the following additional
issues: 1) Unfair Labor Practice; 2) Illegal dismissal; and 3) Damages and Attorneys fees.

42

The Labor Arbiter dismissed the complaint of respondents for illegal dismissal and unfair labor practice, but held
petitioner liable for 13th month pay. Respondents appealed to the NLRC. The NLRC reversed the Decision of the Labor
Arbiter. Petitioner elevated the case to the Court of Appeals via a Petition for Certiorari but the same was denied.
ISSUE:
Whether respondents are considered as regular employees or project employees.
HELD:
Respondents are regular employees.
The principal test for determining whether particular employees are properly characterized as project employees as
distinguished from regular employees, is whether or not the project employees were assigned to carry out a specific
project or undertaking, the duration (and scope) of which were specified at the time the employees were engaged for that
project.
Employers claiming that their workers are project employees should not only prove that the duration and scope of the
employment was specified at the time they were engaged, but also that there was indeed a project.
These jobs and undertakings are clearly within the regular or usual business of the employer company and are not
identifiably distinct or separate from the other undertakings of the company. There is no denying that the manning of the
operations center to air commercials, acting as transmitter/VTR men, maintaining the equipment, and acting as
cameramen are not undertakings separate or distinct from the business of a broadcasting company.
ISSUE:
Whether respondents are entitled to separation pay in lieu of reinstatement.
HELD:
Yes. Since petitioner refused to accept private respondents back to work, reinstatement is no longer practicable. Allowing
private respondents to return to their work might only subject them to further embarrassment, humiliation, or even
harassment.
Thus, in lieu of reinstatement, the grant of separation pay equivalent to one (1) month pay for every year of service is
proper which public respondent actually did. Where the relationship between private respondents and petitioner has been
severely strained by reason of their respective imputations of accusations against each other, to order reinstatement would
no longer serve any purpose. In such situation, payment of separation pay instead of reinstatement is in order
ISSUE:
Whether or not the award of attorneys fees is proper.
HELD:
No. As a matter of sound policy, an award of attorneys fees remains the exception rather than the rule. It must be
stressed, as aptly observed by the appellate court, that it is necessary for the trial court, the NLRC in this case, to make
express findings of facts and law that would bring the case within the exception. In fine, the factual, legal or equitable
justification for the award must be set forth in the text of the decision. The matter of attorneys fees cannot be touched
once and only in the fallo of the decision, else, the award should be thrown out for being speculative and conjectural. In
the absence of a stipulation, attorneys fees are ordinarily not recoverable; otherwise a premium shall be placed on the
right to litigate. They are not awarded every time a party wins a suit.
In the case at bar, the factual basis for the award of attorneys fees was not discussed in the text of NLRC Decision.
PHILASIA SHIPPING AGENCY V. TOMACRUZ
G.R. NO. 181180
AUGUST 15, 2012
J. DE CASTRO
PERMANENT DISABILITY BENEFITS
FACTS:
Andres Tomacruz was a seafarer, whose services were engaged by PHILASIA on behalf of Intermodal Shipping Inc.
(petitioners) as Oiler #1 on board the vessel M/V Saligna. A twelve-month POEA Contract of Employment was duly
signed by the parties on January 9, 2002. This was preceded by four similar contracts. For all five contracts, Tomacruz
43

was required to undergo a pre-employment medical examination and obtain a fit to work rating before he could be
deployed.
Having been issued a clean bill of health, Tomacruz boarded M/V Saligna. However, sometime in September 2002, during
the term of his last contract, Tomacruz noticed blood in his urine. He immediately reported this to the Ship Captain, who
referred him to a doctor in Japan. Tomacruz was subjected to several check-ups and ultrasounds, which revealed a stone
in his right kidney.
Eventually, Tomacruz was repatriated to the Philippines and sent to Micah Medical Clinic & Diagnostic Laboratory. The
November 19, 2002 KUB Ultrasound report of the clinic revealed that he had stones in both his kidneys.
Referred by Micah Medical Clinic to Dr. Nicomedes Cruz, the company-designated physician, Tomacruz went through
more tests, medications, and treatments. On July 25, 2003, Dr. Cruz declared Tomacruz fit to work despite a showing that
there were stones in both his kidneys, and there was the possibility of hematoma.
Intending to get his sixth contract, Tomacruz, armed with the declaration that he was fit to work, proceeded to the office of
the petitioners to seek employment. However, he was told by PHILASIA that because of the huge amount that was spent
on his treatment, their insurance company did not like his services anymore.
Nagging in Tomacruzs mind was the veracity of his fit to work declaration. Thus, he sought the medical opinion of
another physician, Dr. Efren R. Vicaldo who declared that he is unfit to resume work as seaman in any capacity.
Months later, Tomacruz filed a complaint for disability benefits, sickness wages, damages, and attorneys fees against the
petitioners, before the NLRC. The LA dismissed the complaint. As such, the LA said that the company-designated
physicians assessment of Tomacruzs medical condition should be more accurate than that of the subsequent doctors
second medical opinion, which was not supported by sufficient evidence to warrant consideration.
Tomacruz appealed before the NLRC but it agreed with the LAs decision. The CA granted Tomacruzs petition on the
premise that he suffered from permanent total disability.
ISSUE:
Whether or not the CA erred in awarding permanent disability benefits to Tomacruz.
HELD:
No. The Labor Code provision material to this case,
ART. 192. PERMANENT TOTAL DISABILITY
(c) The following disabilities shall be deemed total and permanent: (1) Temporary total
disability lasting continuously for more than one hundred twenty days, except as
otherwise provided for in the Rules.
Rule X, Section 2 of the Rules and Regulations implementing Book IV of the Labor
Code. It states:
SEC. 2. Period of entitlement. (a) The income benefit shall be paid beginning on the
first day of such disability. If caused by an injury or sickness it shall not be paid longer
than 120 consecutive days except where such injury or sickness still requires medical
attendance beyond 120 days but not to exceed 240 days from onset of disability in which
case benefit for temporary total disability shall be paid. However, the System may declare
the total and permanent status at any time after 120 days of continuous temporary total
disability as may be warranted by the degree of actual loss or impairment of physical or
mental functions as determined by the System.
Permanent total disability means disablement of an employee to earn wages in the same kind of work, or work of similar
nature that he was trained for or accustomed to perform, or any kind of work which a person of his mentality and
attainment could do. It does not mean absolute helplessness. In a disability compensation, it is not the injury which is
compensated, but rather it is the incapacity to work resulting in the impairment of ones earning capacity.
A temporary total disability only becomes permanent when so declared by the company[designated] physician within the
periods he is allowed to do so, or upon the expiration of the maximum 240-day medical treatment period without a
declaration of either fitness to work or the existence of a permanent disability.
From the time Tomacruz was repatriated on November 18, 2002, he submitted himself to the care and treatment of the
company-designated physician. When the company-designated physician made a declaration on July 25, 2003 that
Tomacruz was already fit to work, 249 days had already lapsed from the time he was repatriated. As such, his temporary
44

total disability should be deemed total and permanent, pursuant to Article 192 (c)(1) of the Labor Code and its
implementing rule.
MINDANAO TERMINAL AND BROKERAGE SERVICE V. NAGKAHIUSANG MAMUMUNO SA
MINTERBO-SOUTHERN PHILIPPINES FEDERATION
G.R. NO. 174300
DECEMBER 5, 2012
J. DE CASTRO
SEPARATION PAY
FACTS:
Respondent Nagkahiusang Mamumuo sa Minterbro-Southern Philippines Federation of Labor composed of respondents
Manuel Abellana, et al., employees of petitioner Minterbro working on a rotation basis and employed for arrastre and
stevedoring work depending on the actual requirements of the vessels serviced by Minterbro, filed a complaint for
payment of separation pay against Minterbro and De Castro in the NLRC.
At the initial hearing before the Labor Arbiter, Minterbro and De Castro informed the union and its members that the
rehabilitation of the pier had been completed and that they were just awaiting clearance to operate from the PPA. In a
manifestation dated December 12, 1997, the union and its members stated, among others, that they x x x are not anymore
amenable to going back to work with [the] company, for the reason that the latter has not been operating for more than six
(6) months, even if it resumes operation at a later date and would just demand that they be given Retirement or Separation
Pay, as the case may be.
The LA dismissed the complaint for separation pay and declared the 95 complainants to have lost their employment status
for abandonment of work.
The NLRC modified the Decision of the Labor Arbiter. It is admitted by respondent that the last vessel that was serviced
was on April 11-13, 1997 (MV Bosco Polar), and after the rehabilitation of the wharf, on December 22-28, 1997 (MV
Uranus) was served, thereby covering a period of more or less eight months. Respondent cannot conceal or make the
August 1, 1997 formal notice to DOLE or the alleged continued operations of its office personnel until July 31, 1997, an
excuse to evade the mandated six (6) months period since the issue at bar concerns the complainants who became jobless
and penniless because of the December 28, 1996 accident. Complainants cannot be considered to have abandoned their
jobs for the reason that it took respondent a long period [of] time to rehabilitate the wharf causing uncertainties in their
minds which culminated in the filing of the case.
ISSUE:
Whether petitioners are liable for separation pay for effectively dismissing the union members through their prolonged
lay-off of more than six months.
HELD:
Yes. Under Article 286 of the Labor Code, the bona fide suspension of the operation of a business or undertaking for a
period not exceeding six months shall not terminate employment. Consequently, when the bona fide suspension of the
operation of a business or undertaking exceeds six months, then the employment of the employee shall be deemed
terminated. By the same token and applying said rule by analogy, if the employee was forced to remain without work
or assignment for a period exceeding six months, then he is in effect constructively dismissed.
Six months is the period set by law that the operation of a business or undertaking may be suspended thereby suspending
the employment of the employees concerned. The temporary lay-off wherein the employees likewise cease to work should
also not last longer than six months. After six months, the employees should either be recalled to work or permanently
retrenched following the requirements of the law, and that failing to comply with this would be tantamount to dismissing
the employees and the employer would thus be liable for such dismissal.
Petitioners inaction on what they allege to be the unexplained abandonment by Del Monte of its obligations under the
Contract for the Use of Pier coupled with petitioners belated action on the damaged condition of the pier caused the
absence of available work for the union members. As petitioners were responsible for the lack of work at the pier and,
consequently, the layoff of the union members, they are liable for the separation from employment of the union members
on a ground similar to retrenchment.
PROTECTIVE MAXIMUM SECURITY AGENCY V. FUENTES
G.R. NO. 169303
FEBRUARY 11, 2015
45

J. LEONEN
POWER OF THE NLRC; ABANDONMENT
FACTS:
Fuentes was hired as a security guard by Protective sometime in November 1999. At the time of Fuentes' employment,
Protective assigned him to Picop Resources, Inc. He was posted to a security checkpoint designated as Post 33 in Upper
New Visayas, Agusan del Sur.
On July 20, 2000, a group of armed persons ransacked Post 33. Agency-issued uniforms and personal items were also
taken. These armed persons inflicted violence upon Fuentes and the other security guards present at Post 33. On the same
day of the incident, Fuentes and his fellow security guards reported the raid to the PNP in Trento, Agusan del Sur.
After its initial investigation, the PNP found reason to believe that Fuentes conspired and acted in consort with the New
People's Army. Immediately upon the filing of the Complaint, Fuentes was detained at the Mangagoy Police Sub-Station,
Mangagoy, Bislig, Surigao del Sur. During his detention, he alleged that he was "mauled and tied up by the security
officers of Protective."
On August 15, 2001, the Provincial Prosecutor issued the Resolution dismissing the Complaint against Fuentes. On March
14, 2002, Fuentes filed the Complaint "for illegal dismissal, non-payment of salaries, overtime pay, premium pay for
holiday and rest day, 13th month pay, service incentive leave and damages against Protective, Picop, Ernie S. Dolina and
Wilfredo Fuentes.
Fuentes claimed that "right after the criminal complaint for robbery against him was dismissed, he demanded to return to
work but he was refused entry. On the other hand, Protective claims that "as was usual and routine, Fuentes should have
reported to his Team Leader or Officer-in-Charge. Since the incident of July 20, 2000, private respondent has not yet
reported to his Team Leader or to any of the officers of Protective."
Labor Arbiter Legaspi rendered his Decision in favor of Protective. On appeal, the NLRC reversed the Decision of Labor
Arbiter Legaspi and found that Fuentes was illegally dismissed.
Protective asserted that the evidence and the records showed that Fuentes was never dismissed because he had been
missing until the day he filed the Complaint with the Labor Arbiter. It was physically and legally impossible for petitioner
to terminate, constructive, illegal or otherwise, the services of private respondent since the procedure for such an action
have not been initiated. Private respondent had chose not to exercise his rights as an employee and remain unreachable for
reasons known only to him
The CA dismissed the Petition and held that Protective failed to discharge its burden to prove a just cause for dismissal.
Even granting that petitioner was ignorant of private respondent's whereabouts, still it does not suffice to establish
abandonment of work.
Petitioner asserts that the findings of fact of Labor Arbiter Legaspi are binding and conclusive. Petitioner raises that,
between the determination of facts of the NLRC and the Labor Arbiter, the findings of the latter must prevail.
ISSUE:
(3) Whether the CA erred in dismissing the petition for certiorari assailing the decision of the NLRC which
reversed the ruling of the LA.
(4) Whether respondent was justifiably dismissed due to abandonment.
HELD:

(1)

No. The Labor Code vests in the National Labor Relations Commission the authority to reverse the decision
of the Labor Arbiter, provided that the appellant can prove the existence of one of the grounds in Article 223.
The errors in the findings of fact that will justify a modification or reversal of the Labor Arbiter's decision
must be "serious" and, if left uncorrected, would lead to "grave or irreparable damage or injury to the
appellant." Not only must the error be palpable, but there must also be a showing that such error would cause
grave and irreparable injury to the appellant. It should affect the disposition of the cause of the appellant.
In this case, the National Labor Relations Commission decided that there was a serious error in the factual
findings of Labor Arbiter Legaspi.
First, Labor Arbiter Legaspi found that respondent was "unable to perform his duties and responsibilities as
security guard due to the criminal charges that were] filed against him." This led to petitioner replacing
respondent with another security guard. On the other hand, the NLRC found that petitioner's claims that
46

respondent consorted with the New People's Army and committed robbery on July 20, 2000 " were never
substantiated at all. Thus, the refusal to admit respondent to work based on the latter's alleged conspiracy with
the
New
People's
Army
during
the
July
20,
2000
incident
had
no
basis.
Second, as for respondent's absence from work, Labor Arbiter Legaspi found that respondent's whereabouts
were unknown to petitioner. However, Contrary to Labor Arbiter Legaspi's findings, the NLRC found that
petitioner did not exert diligent efforts to locate respondent and afford him his right to due process. It found
that
petitioner
feigned
ignorance
of
the
reason
of
respondent's
absence.
Based on these premises, there was a serious error in the factual determination and conclusions of Labor
Arbiter Legaspi. The errors in the findings of fact directly would affect the primary issues raised by the parties
and their respective claims. If the errors in the findings of fact were not corrected, respondent's right to
security of tenure would have been violated.

(2)

No. The absence of respondent does not constitute abandonment. For a valid finding of abandonment, these
two factors should be present: (1) the failure to report for work or absence without valid or justifiable reason;
and (2) a clear intention to sever employer-employee relationship, with the second as the more determinative
factor which is manifested by overt acts from which it may be deduced that the employees has no more
intention to work. The intent to discontinue the employment must be shown by clear proof that it was
deliberate and unjustified. The burden to prove whether the employee abandoned his or her work rests on the
employer.
The intervening period when respondent failed to report for work, from respondent's prison release to the time
he actually reported for work, was justified. Since there was a justifiable reason for respondent's absence, the
first element of abandonment was not established. Respondent's act of reporting for work after being cleared
of the charges against him showed that he had no intention to sever ties with his employer. Thus, respondent's
actions showed that he intended to resume working for petitioner. The second element of abandonment was
not proven, as well.

CULILI V. EASTERN TELECOMMUNICATIONS


G.R. NO. 165381
FEBRUARY 9, 2011
J. DE CASTRO
REDUNDANCY
FACTS:
Culili was employed by ETPI as a Technician in its Field Operations Department on January 27, 1981. On December 12,
1996, Culili was promoted to Senior Technician in the Customer Premises Equipment Management Unit of the Service
Quality Department and his basic salary was increased.
In 1998, due to business troubles and losses, ETPI was compelled to implement a Right-Sizing Program which consisted
of two phases: the first phase involved the reduction of ETPI's workforce to only those employees that were necessary and
which ETPI could sustain; the second phase entailed a company-wide reorganization which would result in the transfer,
merger, absorption or abolition of certain departments of ETPI.
As part of the first phase, ETPI, offered to its employees who had rendered at least fifteen years of service, the Special
Retirement Program, which consisted of the option to voluntarily retire at an earlier age and a retirement package
equivalent to two and a half (2) months' salary for every year of service. This offer was initially rejected by the Eastern
Telecommunications Employees' Union. On February 8, 1999, ETPI re-offered the Special Retirement Program and the
corresponding retirement package to the one hundred two (102) employees who qualified for the program. Of all the
employees who qualified to avail of the program, only Culili rejected the offer.
The abolition of the Service Quality Department rendered the specialized functions of a Senior Technician unnecessary.
As a result, Culili's position was abolished due to redundancy and his functions were absorbed by Andre Andrada, another
employee already with the Business and Consumer Accounts Department.
Culili discovered that his name was omitted in ETPI's New Table of Organization. Culili, along with three of his coemployees who were similarly situated, wrote their union president to protest such omission. In a letter dated March 8,
1999, ETPI, through its Assistant Vice President Stella Garcia, informed Culili of his termination from employment
effective April 8, 1999.

47

ETPI claimed that it really needed to reduce its workforce at that time and that it had to prepare several letters in advance
in the event that none of the employees avail of the Special Retirement Program. However, ETPI decided to wait for a
favorable response from its employees regarding the Special Retirement Program instead of terminating them.
ISSUE:
Whether or not ETPI is guilty of illegal dismissal.
HELD:
No. Among the requisites of a valid redundancy program are: (1) the good faith of the employer in abolishing the
redundant position; and (2) fair and reasonable criteria in ascertaining what positions are to be declared redundant, such as
but not limited to: preferred status, efficiency, and seniority.
This Court also held that the following evidence may be proffered to substantiate redundancy: the new staffing pattern,
feasibility studies/ proposal on the viability of the newly created positions, job description and the approval by the
management of the restructuring.
In the case at bar, ETPI was upfront with its employees about its plan to implement a Right-Sizing Program. Even in the
face of initial opposition from and rejection of the said program by ETEU, ETPI patiently negotiated with ETEU's officers
to make them understand ETPI's business dilemma and its need to reduce its workforce and streamline its organization.
This evidently rules out bad faith on the part of ETPI.
In deciding which positions to retain and which to abolish, ETPI chose on the basis of efficiency, economy, versatility and
flexibility. It needed to reduce its workforce to a sustainable level while maintaining functions necessary to keep it
operating. The records show that ETPI had sufficiently established not only its need to reduce its workforce and
streamline its organization, but also the existence of redundancy in the position of a Senior Technician. ETPI explained
how it failed to meet its business targets and the factors that caused this, and how this necessitated it to reduce its
workforce and streamline its organization. ETPI also submitted its old and new tables of organization and sufficiently
described how limited the functions of the abolished position of a Senior Technician were and how it decided on whom to
absorb these functions.
ISSUE:
Whether or not ETPI is guilty of unfair labor practice.
HELD:
No. In the past, we have ruled that "unfair labor practice refers to `acts that violate the workers' right to organize.' The
prohibited acts are related to the workers' right to self-organization and to the observance of a CBA." We have likewise
declared that "there should be no dispute that all the prohibited acts constituting unfair labor practice in essence relate to
the workers' right to self-organization." Thus, an employer may only be held liable for unfair labor practice if it can be
shown that his acts affect in whatever manner the right of his employees to self-organize.
There is no showing that ETPI, in implementing its Right-Sizing Program, was motivated by ill will, bad faith or malice,
or that it was aimed at interfering with its employees' right to self-organize. In fact, ETPI negotiated and consulted with
ETEU before implementing its Right-Sizing Program.
By imputing bad faith to the actuations of ETPI, Culili has the burden of proof to present substantial evidence to support
the allegation of unfair labor practice. Culili failed to discharge this burden and his bare allegations deserve no credit.
ISSUE:
Whether or not ETPI was remiss in its duty to observe procedural due process and thus entitled to nominal damages in
addition to his separation pay.
HELD:
Yes. For termination of employment as defined in Article 283 of the Labor Code, the requirement of due process shall be
deemed complied with upon service of a written notice to the employee and the appropriate Regional Office of the
Department of Labor and Employment at least thirty days before effectivity of the termination, specifying the ground or
grounds for termination.
In the case at bar, ETPI, in effecting Culili's termination, simply asked one of its guards to serve the required written
notice on Culili. Culili, on one hand, claims in his petition that this was handed to him by ETPI's vice president, but
previously testified before the Labor Arbiter that this was left on his table. Regardless of how this notice was served on
Culili, this Court believes that ETPI failed to properly notify Culili about his termination. Aside from the manner the
written notice was served, a reading of that notice shows that ETPI failed to properly inform Culili of the grounds for his
termination.
48

Hence, since it has been established that Culili's termination was due to an authorized cause and cannot be considered
unfair labor practice on the part of ETPI, his dismissal is valid. However, in view of ETPI's failure to comply with the
notice requirements under the Labor Code, Culili is entitled to nominal damages in addition to his separation pay.
PFIZER V. VELASCO
G.R. NO. 177467
MARCH 9, 2011
J. DE CASTRO
REINSTATEMENT IMMEDIATELY EXECUTORY
FACTS:
Velasco was employed with petitioner PFIZER, INC. as Professional Health Care Representative since 1 August 1992.
Sometime in April 2003, Velasco had a medical work up for her high-risk pregnancy and was subsequently advised bed
rest which resulted in her extending her leave of absence. Velasco filed her sick leave for the period from 26 March to 18
June 2003, her vacation leave from 19 June to 20 June 2003, and leave without pay from 23 June to 14 July 2003.
On 26 June 2003, while Velasco was still on leave, PFIZER through its Area Sales Manager, herein petitioner Ferdinand
Cortez, personally served Velasco a "Show-cause Notice" dated 25 June 2003. Aside from mentioning about an
investigation on her possible violations of company work rules regarding "unauthorized deals and/or discounts in money
or samples and unauthorized withdrawal and/or pull-out of stocks" and instructing her to submit her explanation on the
matter within 48 hours from receipt of the same, the notice also advised her that she was being placed under "preventive
suspension" for 30 days.
In response, Velasco sent a letter addressed to Cortez dated 28 June 2003 denying the charges. On 16 July 2003, Velasco
sent a letter to PFIZER via Aboitiz courier service asking for additional time to answer the second Show-cause Notice.
That same day, Velasco filed a complaint for illegal suspension with money claims before the Regional Arbitration
Branch. The following day, 17 July 2003, PFIZER sent her a letter inviting her to a disciplinary hearing to be held on 22
July 2003. Velasco received it under protest and informed PFIZER that she had lodged a complaint against the latter. She
likewise opted to withhold answering the Second Show-cause Notice. On 25 July 2003, Velasco received a "Third Showcause Notice," together with copies of the affidavits of two Branch Managers of Mercury Drug, asking her for her
comment within 48 hours. Finally, on 29 July 2003, PFIZER informed Velasco of its "Management Decision" terminating
her employment.
The Labor Arbiter rendered its decision declaring the dismissal of Velasco illegal, ordering her reinstatement with
backwages and further awarding moral and exemplary damages with attorney's fees. On appeal, the NLRC affirmed the
same but deleted the award of moral and exemplary damages. PFIZER moved for reconsideration but its motion was
denied for lack of merit. Undaunted, PFIZER filed with the Court of Appeals a special civil action for the issuance of a
writ of certiorari under Rule 65 of the Rules of Court to annul and set aside the aforementioned NLRC issuances. The
Court of Appeals upheld the validity of respondent's dismissal from employment,
ISSUE:
Whether or not the Court of Appeals committed a serious but reversible error when it ordered Pfizer to pay Velasco wages
from the date of the Labor Arbiter's decision ordering her reinstatement until November 23, 2005, when the Court of
Appeals rendered its decision declaring Velasco's dismissal valid.
HELD:
No. An award or order of reinstatement is immediately self-executory without the need for the issuance of a writ of
execution in accordance with the third paragraph of Article 223 of the Labor Code.
In the case at bar, PFIZER did not immediately admit respondent back to work which, according to the law, should have
been done as soon as an order or award of reinstatement is handed down by the Labor Arbiter without need for the
issuance of a writ of execution. Thus, respondent was entitled to the wages paid to her under the aforementioned writ of
execution. At most, PFIZER's payment of the same can only be deemed partial compliance/execution of the Court of
Appeals Resolution dated October 23, 2006 and would not bar respondent from being paid her wages from May 6, 2005
to November 23, 2005.
ISSUE:
Whether or not respondent should refund the amount she was able to collect by way of garnishment from PFIZER as her
accrued salaries.
HELD:
49

No. An order for reinstatement entitles an employee to receive his accrued backwages from the moment the reinstatement
order was issued up to the date when the same was reversed by a higher court without fear of refunding what he had
received. It cannot be denied that, under our statutory and jurisprudential framework, respondent is entitled to payment of
her wages for the period after December 5, 2003 until the Court of Appeals Decision dated November 23, 2005,
notwithstanding the finding therein that her dismissal was legal and for just cause. Thus, the payment of such wages
cannot be deemed as unjust enrichment on respondent's part.
PAL V. PAZ
G.R. NO. 192924

NOVEMBER 26, 2014

J. REYES
REINSTATEMENT IMMEDIATELY EXECUTORY
FACTS:
Paz was a former commercial pilot of PAL and a member ALPAP, the sole and exclusive bargaining representative of all
the pilots in PAL. On December 9, 1997, ALPAP filed a notice of strike with the NCMB of the DOLE. Pursuant to Article
263(g) of the Labor Code, the DOLE Secretary assumed jurisdiction over the labor dispute and enjoined the parties from
committing acts which will further exacerbate the situation.
Notwithstanding the directive of the DOLE Secretary, the ALPAP officers and members staged a strike and picketed at the
PALs premises. The DOLE Secretary issued a return-to-work order on June 7, 1998, directing all the striking officers and
members of ALPAP to return to work within 24 hours from notice of the order. Even then, the striking members of ALPAP
did not report for work.
On June 25, 1998, , the counsel for ALPAP, informed the members of the union that she has just received a copy of the
return-to-work order and that they have until the following day within which to comply. When the striking members of the
ALPAP reported for work on the following day, the security guards of PAL denied them entry. On June 13, 1998, the
DOLE Secretary issued a resolution on the case from which both parties filed a motion for reconsideration. Pending the
resolution of the motions, PAL filed a petition for approval of rehabilitation plan and for appointment of a rehabilitation
receiver with the SEC, claiming serious financial distress brought about by the strike. Subsequently, on June 23, 1998, the
SEC appointed a rehabilitation receiver for PAL and declared the suspension of all claims against it.
On June 1, 1999, the DOLE Secretary resolved the motions for reconsideration filed by both parties and declared the
strike staged by ALPAP illegal and that the participants thereof are deemed to have lost their employment. On June 25,
1999, the respondent filed a complaint for illegal dismissal against PAL for not accepting him back to work, claiming nonparticipation in the illegal strike.
On March 5, 2001, the LA rendered a Decision, holding that the respondent was illegally dismissed and ordered that he
be reinstated to his former position without loss of seniority rights and other privileges and paid his full backwages
inclusive of allowances and other benefits computed from June 12, 1998 up to his actual reinstatement. PAL appealed the
foregoing decision to the NLRC. Pending appeal, the respondent filed a motion for partial execution of the reinstatement
aspect of the decision. The LA granted the said motion and issued a partial writ of execution on May 25, 2001.
Notwithstanding the reversal of the LA decision, the respondent pursued his move for the issuance of a writ of
execution, claiming that he was entitled to reinstatement salaries which he supposedly earned during the pendency of the
appeal to the NLRC. On August 28, 2001, the LA granted the motion and issued the corresponding writ of execution.
On September 17, 2001, the LA issued an Order, clarifying the respondents entitlement to reinstatement salaries. He
ratiocinated that the order of reinstatement is immediately executory even pending appeal and that under Article 223 of
the Labor Code, the employer has the option to admit the employee back to work or merely reinstate him in the payroll.
Considering, however, that there was no physical reinstatement, the respondent, as a matter of right, must be reinstated in
the payroll. The accrued salaries may now be the subject of execution despite the NLRCs reversal of the decision.
PAL appealed the LA Order dated September 17, 2001 to the NLRC. On June 28, 2002, the NLRC rendered a Resolution,
sustaining the award of reinstatement salaries to the respondent albeit suspending its execution in view of the fact that
PAL was under rehabilitation receivership. PAL filed a motion for reconsideration but the NLRC denied the same. PAL
filed a petition for certiorari with the CA, questioning the NLRC Resolution dated June 28, 2002. Subsequently, in a
Decision dated January 31, 2005, the CA affirmed with modification the NLRC Resolution dated June 28, 2002 in that, in
lieu of reinstatement salaries, PAL is ordered to pay respondent Paz separation pay equivalent to one month salary for
every year of service, to be computed from the time respondent commenced employment with petitioner PAL until the
time the Labor Arbiter issued the writ ordering respondents reinstatement, i.e., on May 25, 2001.
ISSUE:
50

Whether the respondent may collect reinstatement salaries which he is supposed to have received from the time PAL
received the LA decision, ordering his reinstatement
HELD:
No. The rule is that the employee is entitled to reinstatement salaries notwithstanding the reversal of the LA decision
granting him said relief. It is obligatory on the part of the employer to reinstate and pay the wages of the dismissed
employee during the period of appeal until reversal by the higher court. This is so because the order of reinstatement is
immediately executory. Unless there is a restraining order issued, it is ministerial upon the LA to implement the order of
reinstatement. The unjustified refusal of the employer to reinstate a dismissed employee entitles him to payment of his
salaries effective from the time the employer failed to reinstate him.
However, in the Garcia case, the Court somehow relaxed the rule by taking into consideration the cause of delay in
executing the order of reinstatement of the LA. The test is two-fold: (1) there must be actual delay or the fact that the
order of reinstatement pending appeal was not executed prior to its reversal; and (2) the delay must not be due to the
employers unjustified act or omission. If the delay is due to the employers unjustified refusal, the employer may still be
required to pay the salaries notwithstanding the reversal of the Labor Arbiters decision.
It is clear from the records that PAL failed to reinstate the respondent pending appeal of the LA decision to the NLRC. A
scrutiny of the circumstances, however, will show that the delay in reinstating the respondent was not due to the
unjustified refusal of PAL to abide by the order but because of the constraints of corporate rehabilitation. The
inopportune event of PALs entering rehabilitation receivership justifies the delay or failure to comply with the
reinstatement order of the LA.
MONTERO V. TIMES TRANSPORTATION
G.R. NO. 190828
MARCH 16, 2015
J. REYES
VOLUNTARY WITHDRAWAL FOR ILLEGAL DISMISSAL ERASES THE TOLLING OF THE REGLAMENTARY
PERIOD
FACTS:
Respondent TTCI employed herein 21 petitioners as bus drivers, conductors, mechanics, welders, security guards and
utility personnel. Sometime in 1995, the rank-and-file employees of TTCI formed a union named as Times Employees
Union (TEU). In March 1997, members of TEU went on strike but the Labor Secretary assumed jurisdiction over the
labor dispute and certified the same for compulsory arbitration, a return-to-work Order was issued which ended the strike.
The TTCI Board of Directors approved a resolution confirming the authority given to respondent Santiago, to gradually
dispose the assets of the TTCI as a result of its losses. TTCI also adopted a company-wide retrenchment program, which
will take effect on October 1, 1997. Santiago was given the authority to determine the number of excess employees who
would be the subject of retrenchment. After the sale of 25 buses to MENCORP, several union members also received
notices that they were being retrenched effective 30 days from September 16, 1997.
For a second time, TEU declared a strike against TTCI, but the latter merely reiterated the earlier return-to-work order of
the Labor Secretary. For disregarding the said return-to-work order, Santiago issued two notices of termination dated
October 26, 1997 and November 24, 1997. A total of 119 employees were dismissed for participating in the illegal strike.
On December 4, 1997, Santiago served to the DOLE a notice that TTCI would be closing its operations due to heavy
business losses.
On May 14, 1998, petitioners filed several complaints against TTCI and MENCORP before the NLRC. The case was
consolidated but was subsequently withdrawn on March 22, 1999 upon motion by TEUs counsel. 4 years later, several
complaints for unfair labor practice, illegal dismissal with money claims, damages and attorneys fees were filed against
TCCI, Santiago, MENCORP and its general manager Virginia Mendoza before the LA from June to July 2002.
TTCI asserted that the petitioners cause of action had already been barred by prescription because the complaints were
filed only in June 2002 or after almost five years from the date of their dismissal. MENCORP, on the other hand, raised
the defense of lack of employer-employee relationship since it never engaged the services of the petitioners when TTCI
sold to them its buses and the Certificates of Public Convenience..
ISSUE:
Whether petitioners complaints for illegal dismissal have already prescribed.
51

HELD:
Yes. An action predicated upon an injury to the rights of the plaintiff, as contemplated under Article 1146 of the New Civil
Code, which must be brought within four years.
Although the commencement of a civil action stops the running of the statute of prescription or limitations, its dismissal
or voluntary abandonment by plaintiff leaves the parties in exactly the same position as though no action had been
commenced at all. In like manner, while the filing of the complaint for illegal dismissal before the LA interrupted the
running of the prescriptive period, its voluntary withdrawal left the petitioners in exactly the same position as though no
complaint had been filed at all. The withdrawal of their complaint effectively erased the tolling of the reglementary
period.
Hence, while the filing of the said case could have interrupted the running of the four-year prescriptive period, the
voluntary withdrawal of the petitioners effectively cancelled the tolling of the prescriptive period within which to file their
illegal dismissal case, leaving them in exactly the same position as though no labor case had been filed at all. The running
of the four-year prescriptive period not having been interrupted by the filing of the first consolidated labor complaints, the
petitioners cause of action had already prescribed in four years after their cessation of employment on October 26, 1997
and November 24, 1997. Consequently, when the petitioners filed their complaint for illegal dismissal, separation pay,
retirement benefits, and damages in 2002, their claim, clearly, had already been barred by prescription.
THE HERITAGE HOTEL MANILA V. SECRETARY OF LABOR AND EMPLOYMENT
G.R. NO. 172132
JULY 23, 2014
J. BERSAMIN
EMPLOYER AS BYSTANDER IN PETITION FOR CERTIFICATION ELECTION; CANCELLATION OF UNION
REGISTRATION
FACTS:
On October 11, 1995, respondent National Union of Workers in Hotel Restaurant and Allied Industries-Heritage Hotel
Manila Supervisors Chapter (NUWHRAIN-HHMSC) filed a petition for certification election, seeking to represent all the
supervisory employees of Heritage Hotel Manila. The petitioner filed its opposition, but the opposition was deemed
denied on February 14, 1996 when Med-Arbiter Fernando issued his order for the conduct of the certification election.
The petitioner appealed the order of Med-Arbiter Fernando, but the appeal was also denied. A pre-election conference was
then scheduled but it was suspended until further notice because of the repeated non-appearance of NUWHRAINHHMSC.
On January 29, 2000, NUWHRAIN-HHMSC moved for the conduct of the pre-election conference. The petitioner
primarily filed its comment on the list of employees submitted by NUWHRAIN-HHMSC, and simultaneously sought the
exclusion of some from the list of employees for occupying either confidential or managerial positions.
On May 12, 2000, the petitioner filed a petition for the cancellation of NUWHRAIN-HHMSCs registration as a labor
union for failing to submit its annual financial reports and an updated list of members as required by Article 238 and
Article 239 of the Labor Code. It filed another motion on June 1, 2000 to seek either the dismissal or the suspension of the
proceedings on the basis of its pending petition for the cancellation of union registration. The following day, however, the
DOLE issued a notice scheduling the certification elections on June 23, 2000.
The certification election proceeded as scheduled, and NUWHRAIN-HHMSC obtained the majority vote of the
bargaining unit. The petitioner filed a protest (with motion to defer the certification of the election results and the winner),
insisting on the illegitimacy of NUWHRAIN-HHMSC. Petitioner also assails the failure of NUWHRAIN-HHMSC to
submit its periodic financial reports and updated list of its members pursuant to Article 238 and Article 239 of the Labor
Code. It contends that the serious challenges against the legitimacy of NUWHRAIN-HHMSC as a union raised in the
petition for the cancellation of union registration should have cautioned the Med-Arbiter against conducting the
certification election.
ISSUE:
(3) Whether petitioner has the legal personality to assail the proceedings for certification election.
(4) Should the petition for the cancellation of union registration based on mixed membership of supervisors
and managers in a labor union, and the non-submission of reportorial requirements to the DOLE justify
the suspension of the proceedings for the certification elections or even the denial of the petition for the
certification election?
52

HELD:
(1) No. Except when it is requested to bargain collectively, an employer is a mere bystander to any petition for
certification election; such proceeding is non-adversarial and merely investigative, for the purpose thereof is to
determine which organization will represent the employees in their collective bargaining with the employer. The
choice of their representative is the exclusive concern of the employees; the employer cannot have any partisan
interest therein; it cannot interfere with, much less oppose, the process by filing a motion to dismiss or an appeal
from it; not even a mere allegation that some employees participating in a petition for certification election are
actually managerial employees will lend an employer legal personality to block the certification election. The
employer's only right in the proceeding is to be notified or informed thereof.
(2) No. Mixed membership does not result in the illegitimacy of the registered labor union unless the same was done
through misrepresentation, false statement or fraud according to Article 239 of the Labor Code.
Article 239 provides the following grounds for cancellation of union registration: (a) Misrepresentation, false
statement or fraud in connection with the adoption or ratification of the constitution and by-laws or
amendments thereto, the minutes of ratification, and the list of members who took part in the ratification; (b)
Misrepresentation, false statements or fraud in connection with the election of officers, minutes of the election
of officers, and the list of voters; (c) Voluntary dissolution by the members.
R.A. No. 9481 also inserted in the Labor Code Article 242-A are the Reportorial Requirements to be
submitted to the Bureau by the legitimate labor organization concerned: (a) Its constitution and by-laws, or
amendments thereto, the minutes of ratification, and the list of members who took part in the ratification of
the constitution and by-laws within thirty (30) days from adoption or ratification of the constitution and bylaws or amendments thereto; (b) Its list of officers, minutes of the election of officers, and list of voters within
thirty (30) days from election; (c) Its annual financial report within thirty (30) days after the close of every
fiscal year; and (d) Its list of members at least once a year or whenever required by the Bureau. The failure to
comply with the above requirements shall not be a ground for cancellation of union registration but
shall subject the erring officers or members to suspension, expulsion from membership, or any
appropriate penalty.
EXOCET SECURITY AND ALLIED SERVICES CORPORATION V. SERRANO
G.R. NO. 198538
SEPTEMBER 29, 2014
J. VELASCO
CONSTRUCTIVE DISMISSAL; JUST CAUSE
FACTS:
Exocet is engaged in the provision of security personnel to its various clients or principals. By virtue of its contract with
JG Summit, Exocet assigned Serrano on September 24, 1994 as close-in security personnel for one of JG Summits
corporate officers, Johnson Robert L. Go. After eight years, Serrano was re-assigned as close-in security for Lance
Gokongwei, and then to his wife, Mary. On August 15, 2006, Serrano was relieved by JG Summit from his duties. For
more than six months after he reported back to Exocet, Serrano was without any reassignment. On March 15, 2007,
Serrano filed a complaint for illegal dismissal against Exocet with the NLRC.
Exocet denied dismissing Serrano alleging that, after August 15, 2006, Serrano no longer reported for duty assignment as
VIP security for JG Summit, and that on September 2006, he was demanding for VIP Security detail to another client.
Since Exocet did not have clients in need of VIP security assignment, Serrano was temporarily assigned to general
security service. It was Serrano who declined the assignment on the ground that he is not used to being a regular security
guard. Serrano, Exocet added, even refused to report for immediate duty, as he was not given a VIP security assignment.
The Labor Arbiter ruled that Serrano was illegally dismissed. While not actually dismissed, Serrano was placed on a
floating status for more than six months and so, was deemed constructively dismissed. Thus, the Labor Arbiter ordered
Exocet to pay Serrano separation pay.
Serrano appealed the Labor Arbiters Decision to the NLRC. The NLRC initially affirmed the ruling of the Labor Arbiter,
but modified the monetary award to include the payment of backwages for six months that Serrano was not given a
security assignment. Acting on Exocets motion for reconsideration, the NLRC modified its earlier decision by removing
the award for backwages. The NLRC deviated from its earlier findings and ruled that Serrano was not constructively
dismissed, as his termination was due to his own fault, stubborn refusal, and deliberate failure to accept a re-assignment.
The NLRC issued another Resolution denying Serranos motion for reconsideration. Hence, not satisfied with the NLRCs
ruling, Serrano filed a petition for certiorari with the CA. The CA found that Serrano was constructively dismissed, as
Exocet failed to re-assign him within six months after placing him on floating status.
53

ISSUE:
Whether or not Serrano was constructively dismissed.
HELD:
No. The DOLE) issued DO 14-01 which states that the lack of service assignment for a continuous period of six (6)
months is an authorized cause for the termination of the employee, who is then entitled to a separation pay equivalent
to half month pay for every year of service.To validly terminate a security guard for lack of service assignment for a
continuous period of six months, the security agency must comply with the provisions of Article 289 (previously Art. 283)
of the Labor Code, which mandates that a written notice should be served on the employee on temporary off-detail or
floating status and to the DOLE 1 month before the intended date of termination.
While the Court has recognized the security guards right to security of tenure under the floating status rule, the Court
has similarly acknowledged the management prerogative of security agencies to transfer security guards when
necessary in conducting its business, provided it is done in good faith.
There is no question that the security guard, Serrano, was placed on floating status after his relief from his post as a VIP
security by his security agencys client. Yet, there is no showing that his security agency, petitioner Exocet, acted in bad
faith when it placed Serrano on such floating status. What is more, the present case is not a situation where Exocet did
not recall Serrano to work within the six-month period as required by law and jurisprudence. Exocet did, in fact,
make an offer to Serrano to go back to work. It is manifestly unfair and unacceptable to immediately declare the mere
lapse of the six-month period of floating status as a case of constructive dismissal, without looking into the peculiar
circumstances that resulted in the security guards failure to assume another post. This is especially true in the present case
where the security guards own refusal to accept a non-VIP detail was the reason that he was not given an assignment
within the six-month period. The security agency, Exocet, should not then be held liable.
Since Serrano was not actually or constructively dismissed from his employment by petitioner Exocet, it is best that
petitioner Exocet direct him to report for work, if any security assignment is still available to him. If Serrano still refuses
to be assigned to any available guard position, he shall be deemed to have abandoned his employment with petitioner. If
no security assignment is available for respondent, Exocet should comply with the requirements of DO 14-01, in relation
to Art. 289 of the Labor Code, and serve a written notice on Serrano and the DOLE one (1) month before the intended
date of termination, and pay Serrano separation pay equivalent to half month pay for every year of his actual service.
MCMER CORPORATION V. NLRC
G.R. NO. 193421
JUNE 4, 2014
J. PERALTA
CONSTRUCTIVE DISMISSAL
FACTS:
Private respondent was employed by petitioner McMer as Legal Assistant and was eventually promoted as Head of Legal
Department, and concurrently, as Officer-in-Charge of petitioner McMers Legal and Administrative Department.
According to private respondent, he and petitioners, specifically Roque and Alvestir, McMers General Manager and
President, respectively, have been on a cold war brought often by the disagreement in the design and implementation of
company policies and procedures. The rift between heightened when petitioner McMer started verbally and maliciously
imputing against Guiao, Department Head III, Logistics Department, and another officer of the Logistics Department,
Rebulado, Department Head I, certain unfounded score of inefficient performance of duty.
At around noon on July 20, 2007, petitioner Roque gave an immediate summon upon private respondent to proceed to his
office to discuss administrative matters. Private respondent, instead, went to petitioner Alvestirs office, and informed her
of petitioner Roques disposition and his fear of a perceived danger to his person. He requested for petitioner Alvestir to
go to petitioner Roques office instead, of which petitioner Alvestir conceded. Moments later, petitioner Roque, at the
height of anger, confronted private respondent and commanded him to proceed to his office.
As a consequence of the foregoing, private respondent elected to discontinue work that afternoon and immediately
proceeded to the Valenzuela Police Headquarters to report on the incident in the police blotter. Private respondent did not
report for work from July 21, 2007 up to July 30, 2007. Because of this, petitioner McMer, through petitioner Alvestir,
issued a Memorandum dated July 30, 2007 directing private respondent to explain within 5 days why no disciplinary
action should be imposed upon him for being in absence without official leave. In response, private respondent sent a
letter dated August 6, 2007 explaining the reason why he refused to report for work during the aforesaid period.

54

On August 6, 2007, private respondent filed a complaint for unfair labor practices, constructive illegal dismissal, nonpayment of 13th month pay and separation pay, moral and exemplary damages, as well as attorneys fees, against
petitioners McMer Corporation, Inc., Roque, and Alvestir.
A conciliary meeting was held inside petitioners premises to discuss the possibility of amicable settlement. In the end,
private respondent was informed verbally by petitioner Alvestir that on account of strained relationship brought about by
the institution of a labor case against petitioners, the latter is inclined to dismiss him from office. Private respondent was
offered a separation pay in the sum of P55,000.00.
ISSUE:
Whether or not the CA seriously erred in sustaining the NRLCs finding that private respondent was constructively
dismissed, and entitled to full backwages, separation pay in lieu of reinstatement, and moral, exemplary and nominal
damages.
HELD:
No. The test of constructive dismissal is whether a reasonable person in the employees position would have felt
compelled to give up his position under the circumstances. It is an act amounting to dismissal but made to appear as if it
were not. Constructive dismissal is, therefore, a dismissal in disguise. As such, the law recognizes and resolves this
situation in favor of employees in order to protect their rights and interests from the coercive acts of the employer. In fact,
the employee who is constructively dismissed may be allowed to keep on coming to work.
As may gleaned from the records, what transpired on July 20, 2007 was not merely an isolated outburst on the part of
petitioner Roque. The latters behaviour towards his employees shows a clear insensibility rendering the working
condition of private respondent unbearable. Private respondent had reason to dawdle and refuse to comply with the
summon of petitioner Roque out of severe fear that he will be physically harmed. In fact, the same was clearly manifested
by his immediate reaction to the situation by going to the Valenzuela Police to report the incident.
Moreover, private respondent has exhibited a strong opposition to some company practices resulting in a severe marginal
distance between him and petitioners Roque and Alvestir at the workplace. This, together with the harassment and
intimidation displayed by petitioner Roque to his employees, became so unbearable for private respondent to continue his
employment with petitioner McMer. For private respondent, retaining the employment despite his despair was a matter of
principle.
The NLRC and the CA, therefore, correctly appreciated the foregoing events as badges of constructive dismissal, since
private respondent could not have given up a job he has engaged in for eight years unless it has become so unbearable for
him to stay therein. Indeed, private respondent felt compelled to give up his employment.
ARRIOLA V. PILIPINO STAR NGAYON
G.R. NO. 175689
AUGUST 13, 2014
J. LEONEN
PRESCRIPTIVE PERIOD; ABANDONMENT
FACTS:
In July 1986, Pilipino Star Ngayon, Inc. employed Arriola as correspondent assigned in Olongapo City and Zambales.
Arriola had held various positions in Pilipino Star Ngayon, Inc. before becoming a section editor and writer of its
newspaper. He wrote Tinig ng Pamilyang OFWs until his column was removed from publication on November 15,
1999. Since then, Arriola never returned for work.
On November 15, 2002, Arriola filed a complaint for illegal dismissal, non-payment of salaries/wages, moral and
exemplary damages, actual damages, attorney's fees, and full backwages with the NLRC.
Pilipino Star Ngayon, Inc. denied Arriolas allegations. They alleged that around the third week of November 1999,
Arriola suddenly absented himself from work and never returned. After a few months, they learned that Arriola transferred
to a rival newspaper publisher, Imbestigador, to write Boses ng Pamilyang OFWs.
The LA decided the case. She ruled that laches had set in, emphasizing that Arriola took three years and one day to file
his complaint. The LA also found that Arriola abandoned his employment with Pilipino Star Ngayon, Inc. to write for a
rival newspaper publisher. She also noted Arriolas admission that he did not contemplate the filing of an illegal dismissal
complaint but nevertheless filed one upon his lawyers advice.

55

On Arriolas money claims, the LA ruled that they have already prescribed. She cited Article 291 of the Labor Code,
which requires that all money claims arising from employer-employee relations be filed three years from the time the
cause of action accrued. Since Arriola filed his complaint on November 15, 2002, which was three years and one day
from his alleged illegal dismissal on November 15, 1999, the LA ruled that his money claims were already barred.
On Arriolas appeal, the NLRC sustained the LAs findings. The Commission likewise denied Arriolas motion for
reconsideration. Arriola filed a petition for certiorari with the Court of Appeals.
The CA ruled that Arriola was not illegally dismissed. Pilipino Star Ngayon, Inc. had the management prerogative to
determine which columns to maintain in its newspaper. Its removal of Tinig ng Pamilyang OFWs from publication did
not mean that it illegally dismissed Arriola. His employment, according to the appellate court, did not depend on the
existence of the column. The CA ruled that it was Arriola who abandoned his employment. The CA likewise ruled that his
money claims have all prescribed based on Article 291 of the Labor Code.
ISSUE:
(5) Whether Arriolas money claims have prescribed.
(6) Whether Arriola was illegally dismissed.
HELD:
(1) No for backwages and damages. Art. 291. MONEY CLAIMS. All money claims arising from employeremployee relations accruing during the effectivity of this Code shall be filed within three (3) years from the time
the cause of action accrued; otherwise they shall be forever barred. Article 291 covers claims for overtime pay,
holiday pay, service incentive leave pay, bonuses, salary differentials, and illegal deductions by an employer. It
also covers money claims arising from seafarer contracts.
The provision, however, does not cover money claims consequent to an illegal dismissal such as backwages. It
also does not cover claims for damages due to illegal dismissal. These claims are governed by Article 1146 of the
Civil Code of the Philippines. Under Article 1146, an action based upon an injury to the rights of a plaintiff must
be filed within four years. Actions for damages due to illegal dismissal are likewise actions upon an injury to the
rights of the plaintiff. Article 1146 of the Civil Code of the Philippines, therefore, governs these actions.
Yes for unpaid salaries. Arriola filed his complaint three years and one day from the time he was allegedly
dismissed and deprived of his salaries. Since a claim for unpaid salaries arises from employer-employee relations,
Article 291 of the Labor Code applies. Arriolas claim for unpaid salaries was filed beyond the three-year
prescriptive period.
(2) No. A newspaper publisher has the management prerogative to determine what columns to print in its newspaper.
Arriola abandoned his employment with Pilipino Star Ngayon, Inc.
Abandonment is the clear, deliberate and unjustified refusal of an employee to continue his employment, without
any intention of returning. It has two elements: first, the failure to report for work or absence without valid or
justifiable reason and, second, a clear intention to sever employer-employee relations exists. The second element
is the more determinative factor and is manifested by overt acts from which it may be deduced that the employee
has no more intention to work.
Assuming that Arriola started writing for Imbestigador only on February 17, 2003, he nonetheless failed to report
for work at Pilipino Star Ngayon, Inc. after November 15, 1999 and only filed his illegal dismissal complaint on
November 15, 2002. He took three years and one day to remedy his dismissal. This shows his clear intention to
sever his employment with Pilipino Star Ngayon, Inc.
GOODYEAR PHILIPPINES V. ANGUS
G.R. NO. 185449
NOVEMBER 12, 2014
J. DEL CASTILLO
RETIREMENT BENEFITS; SEPARATION PAY
FACTS:
Angus was employed by Goodyear on November 16, 1966 and occupied the position of Secretary to the Manager of
Quality and Technology. Goodyear implemented cost-saving measures which included the streamlining of its workforce.
On September 19, 2001, Angus received from Ramos, the HR Director of Goodyear, informing him that the position of
56

Secretary to the Manager of Quality & Technology is already redundant. Considering that he has rendered 34.92 years of
service to the company have reached the minimum age of 55, he was granted early retirement benefit at 47 days per year.
In his letter-reply, Angus suggested that he be given a premium of additional 3 days for every year of service or a total of
50 days. In connection with the retrenchment of Angus, an Establishment Termination Report was filed by Goodyear with
the DOLE. Angus accepted under protest the checks which covered payment of her retirement benefits computed at 47
days' pay per year of service and other company benefits.
In response to Angus' protest, Ramos wrote her a letter explaining that the company has already offered her the most
favorable separation benefits due to redundancy, that is, 47 days' pay per year of service instead of the applicable rate of
45 days' pay per year of service. And based on the Retirement Plan under the CBA and the parties' Employment Contract,
Angus is entitled to only one of the following kinds of separation pay: (1) normal retirement which is payable at 47 days'
pay per year of service; (2) early retirement at a maximum of 47 days' pay per year of service; (3) retrenchment,
redundancy, closure of establishment at 45 days' pay per year of service; (4) medical disability at 45 days' pay per year of
service; or (5) resignation at 20 days' pay per year of service. Because of these, Ramos informed Angus that the company
cannot anymore entertain any of her additional claims.
Angus reiterated her claim for both termination pay and early retirement benefits. Angus filed with the Labor Arbiter a
complaint for illegal dismissal with claims for separation pay, damages and attorney's fees against petitioners. Angus
claimed that the separation pay to which she is entitled by law is entirely different from the retirement benefits that she
received; that nothing in the company's Retirement Plan under the CBA, the CBA itself or the Employment Contract
prohibits the grant of more than one land of separation pay. Angus presented a copy of the latest CBA between Goodyear
and Unyon ng mga Manggagawa sa Goma sa Goodyear Phils., Inc. effective for the period July 25, 2001 to July 24, 2004,
to show that the provisions alluded to by the petitioners do not exist. In contrast, she pointed to Section 5, Article VIII of
the latest CBA which she claimed to be the one applicable to her case, viz:
SECTION

5.

Retirement

Plan.

At normal retirement age of 60 years, a worker shall be entitled to a lump sum retirement benefit in an amount
equivalent to his daily rate (base rate x 8) multiplied by 47 days, and further multiplied by his years of service.
A worker who is at least 50 years old and with at least 15 years of service, and who has been recommended by the
President of the UNION for early retirement and duly approved by the Human Resources Director, shall be paid a
lump sum retirement benefit as follows:
Years
Service Rendered
15 - less than 21
21 - less than 26
26 - less than 31
31 and up

of

Retirement
Benefit
Equivalent to
34 days pay per year of service
35 days pay per year of service
36 days pay per year of service
47 days pay per year of service

The Labor Arbiter upheld the validity of Angus' termination from employment. It likewise declared that the amount she
received from the company was actually payment of separation pay due to redundancy, only that it was computed under
the CBA's retirement plan since the same was more advantageous to her. Anent her claim for both separation pay and
retirement benefits, the Labor Arbiter held that the grant of both is not allowed under the Retirement Plan/CBA. The
NLRC affirmed the ruling of the Labor Arbiter. Angus filed a motion for reconsideration, but was denied. Angus filed a
Petition for Certiorari with the CA. The CA rendered a Decision partially granting Angus' Petition. While it found her
dismissal valid in both substance and procedural aspects, it declared Angus entitled to separation pay in addition to the
retirement pay she already received.
Petitioners argue that the CA erred in ordering them to still pay Angus separation pay as she was already paid the same at
the rate used for computing early retirement benefits. They insist that Angus is entitled to only one kind of pay as the
recovery of both retirement benefits and separation pay is proscribed by the company's CBA. Petitioners further argue that
Angus is not entitled to retirement pay because she does not meet the requirements enumerated in the Retirement Plan
provision of the CBA.
ISSUE:
Whether Angus is entitled to both separation pay and early retirement benefit.
HELD:
Yes. An employee is entitled to recover both separation pay and retirement benefits in the absence of a specific prohibition
in the Retirement Plan or CBA. Retirement benefits and separation pay are not mutually exclusive. Retirement benefits
are a form of reward for an employee's loyalty and service to an employer and are earned under existing laws, CBAs,
employment contracts and company policies. On the other hand, separation pay is that amount which an employee
57

receives at the time of his severance from employment, designed to provide the employee with the wherewithal during the
period that he is looking for another employment and is recoverable only in instances enumerated under Articles 283 and
284 of the Labor Code or in illegal dismissal cases when reinstatement is not feasible. In the case at bar, Article 283
clearly entitles Angus to separation pay apart from the retirement benefits she received from petitioners.
The CBA does not contain any restriction on the availment of benefits under the company's Retirement Plan and of
separation pay. Indeed, the Labor Arbiter and the NLRC erred in ignoring this material piece of evidence which is decisive
of the issue presented before them.
Further, while it is obvious that Angus is not entitled to compulsory retirement as she has not yet reached the age of 60,
there is no denying that she is qualified for early retirement. Under the provision of the Retirement Plan of the CBA, a
worker who is at least 50 years old and with at least 15 years of service, and who has been recommended by the President
of the Union for early retirement and duly approved by the HR Director, shall be entitled to lump sum retirement benefits.
At the time of her termination, Angus was already 57 years of age and had been in the service for more than 34 years. All
the requirements for Angus' availment of early retirement under the Retirement Plan of CBA were substantially complied
with.
RADIO MINDANAO NETWORK V. AMURAO
G.R. NO. 167225 OCTOBER 22, 2014
J. BERSAMIN
VALIDITY OF QUITCLAIM
FACTS:
On February 16, 1989, petitioner RMN hired respondent Michael as a radio broadcaster for its DWKC-FM station and
production manager for its metropolitan radio operations at a monthly salary of P28,400.00. Years later, RMN decided to
reformat and restructure the programming of its DWKC-FM. On April 25, 2002, the president of RMN met with Michael
and other personnel of the station to inform them of the managements decision, advising them that the reformatting and
restructuring of the stations programs would necessarily affect their employment; but assuring that they would be paid
their retirement pay and other benefits. To formalize the discussions had in their meeting, RMN furnished Michael and
other personnel separate letters.
However, Michael and the other personnel refused to sign in receipt when the letters were served on them. Not long after,
however, they accepted the offer of RMN and executed affidavits relinquishing all their claims against the employer.
The Labor Arbiter rendered a decision declaring the dismissal of Michael as illegal on the ground that the reformatting and
restructuring of RMNs radio programming did not fall under any of the just or authorized and that the quitclaim Michael
signed was void because it was not voluntarily executed.
RMN appealed to the NLRC, contending that the decision of the Labor Arbiter was premature and that the quitclaim
signed in its favor was valid and binding because it represented a voluntary and reasonable settlement of Michaels claims.
The NLRC found no merit in the contention of RMN that the appealed decision was prematurely rendered. It held that the
quitclaim was null and void for not being voluntarily executed and modified the decision of the Labor Arbiter in that the
amount already received by Michael was to be deducted from the monetary benefits awarded to him; and deleted the
awards for moral and exemplary damages. RMN moved for reconsideration, but the NLRC denied its motion.
Consequently, RMN filed with the CA its petition for certiorari but the same was dismissed.
ISSUE:
Whether the quitclaim executed by the employee was valid and effective.
HELD:
Yes. Not all quitclaims are per se invalid or against public policy. A quitclaim is invalid or contrary to public policy only:
(1) where there is clear proof that the waiver was wrangled from an unsuspecting or gullible person; or (2) where the
terms of settlement are unconscionable on their face. In instances of invalid quitclaims, the law steps in to annul the
questionable waiver. Indeed, there are legitimate waivers that represent the voluntary and reasonable settlements of
laborers claims that should be respected by the Court as the law between the parties. Where the party has voluntarily
made the waiver, with a full understanding of its terms as well as its consequences, and the consideration for the quitclaim
is credible and reasonable, the transaction must be recognized as a valid and binding undertaking, and may not later be
disowned simply because of a change of mind. A waiver is essentially contractual.

58

The requisites for the validity of Michaels quitclaim were satisfied. Firstly, Michael acknowledged in his quitclaim that
he had read and thoroughly understood the terms of his quitclaim and signed it of his own volition. Being a radio
broadcaster and production manager, he occupied a highly responsible position in the company. The language and content
of the quitclaim were clear and uncomplicated such that he could not claim that he did not understand what he was
signing. Secondly, the settlement pay of P311,922.00 was credible and reasonable considering that Michael did not even
assail such amount as unconscionably low, or even state that he was entitled to a higher amount. Thirdly, that he was
required to sign the quitclaim as a condition to the release of the settlement pay did not prove that its execution was
coerced. Having agreed to part with a substantial amount of money, RMN took steps to protect its interest and obtain its
release from all obligations once it paid Michael his settlement pay, which it did in this case. And, lastly, that he signed
the quitclaim out of fear of not being able to provide for the needs of his family and for the schooling of his children did
not immediately indicate that he had been forced to sign the same. Dire necessity should not necessarily be an acceptable
ground for annulling the quitclaim, especially because it was not at all shown that he had been forced to execute it.
BANKARD V. NLRC
G.R. NO. 171664
MARCH 6, 2013
J. MENDOZA
UNFAIR LABOR PRACTICE
FACTS:
On June 26, 2000, respondent Bankard Employees Union-AWATU (Union) filed before the NCMB its first Notice of
Strike (NOS), alleging commission of unfair labor practices by petitioner Bankard, to wit: 1) job contractualization; 2)
outsourcing/contracting-out
jobs;
3)
manpower
rationalizing
program;
and
4)
discrimination.
On July 3, 2000, the initial conference was held where the Union clarified the issues cited in the NOS. On July 5, 2000,
the Union held its strike vote balloting where the members voted in favor of a strike. On July 10, 2000, Bankard asked the
Office of the Secretary of Labor to assume jurisdiction over the labor dispute or to certify the same to the NLRC for
compulsory arbitration. On July 12, 2000, the Labor Secretary of the DOLE issued the order certifying the labor dispute to
the NLRC.
On July 25, 2000, the Union declared a CBA bargaining deadlock. The following day, the Union filed its second NOS,
alleging bargaining in bad faith on the part of Bankard. The Union, despite the two certification orders issued by the Labor
Secretary enjoining them from conducting a strike or lockout and from committing any act that would exacerbate the
situation, went on strike on August 11, 2000.
It was Bankards position that job contractualization or outsourcing or contracting-out of jobs was a legitimate exercise of
management prerogative and did not constitute unfair labor practice. It had to implement new policies and programs, one
of which was the Manpower Rationalization Program (MRP) in December 1999, to further enhance its efficiency and be
more competitive in the credit card industry. The MRP was an invitation to the employees to tender their voluntary
resignation, with entitlement to separation pay equivalent to at least 2 months salary for every year of service. Those
eligible under the companys retirement plan would still receive additional pay. Thereafter, majority of the Phone Center
and the Service Fulfilment Division availed of the MRP. Thus, Bankard contracted an independent agency to handle its
call center needs.
Bankard also denied that there was bad faith on its part in bargaining with the Union. It came up with counter-offers to the
Unions proposals, but the latters demands were far beyond what management could give.
ISSUE:
Whether Bankard committed acts of unfair labor practice.
HELD:
No. The Court has ruled that the prohibited acts considered as ULP relate to the workers right to self-organization and to
the observance of a CBA. It refers to acts that violate the workers right to organize. Without that element, the acts, even
if unfair, are not ULP. Thus, an employer may only be held liable for unfair labor practice if it can be shown that his acts
affect in whatever manner the right of his employees to self-organize.
In this case, the Union claims that Bankard, in implementing its MRP which eventually reduced the number of employees,
clearly violated Article 248(c) of the Labor Code which states that:
(c) To contract out services or functions being performed by union members when such will interfere
with, restrain or coerce employees in the exercise of their rights to self-organization.
59

However, aside from the bare allegations of the Union, nothing in the records strongly proves that Bankard intended its
program, the MRP, as a tool to drastically and deliberately reduce union membership. True, the program might have
affected the number of union membership because of the employees voluntary resignation and availment of the package,
but it does not necessarily follow that Bankard indeed purposely sought such result. It must be recalled that the MRP was
implemented as a valid cost-cutting measure, well within the ambit of the so-called management prerogatives. Bankard
contracted an independent agency to meet business exigencies.
Contracting out of services is an exercise of business judgment or management prerogative. Absent any proof that
management acted in a malicious or arbitrary manner, the Court will not interfere with the exercise of judgment by an
employer. Furthermore, bear in mind that ULP is punishable with both civil and/or criminal sanctions. As such, the party
so alleging must necessarily prove it by substantial evidence.
BARTOLOME V. SSS
G.R. NO. 192531
NOVEMBER 12, 2014
J. VELASCO
BENEFICIARY UNDER SSS LAW
FACTS:
John, born on June 9, 1983, was employed as electrician by Scanmar Maritime Services, Inc., on board the vessel Maersk
Danville, since February 2008. As such, he was enrolled under the government's Employees' Compensation Program
(ECP). Unfortunately, on June 2, 2008, an accident occurred on board the vessel whereby steel plates fell on John, which
led to his untimely death the following day.
John was, at the time of his death, childless and unmarried. Thus, petitioner Bernardina P. Bartolome, John's biological
mother and, allegedly, sole remaining beneficiary, filed a claim for death benefits under PD 626 with the SSS. However,
the SSS denied the claim. The denial was appealed to the Employees' Compensation Commission (ECC), which affirmed
the ruling of the SSS.
In denying the claim, both the SSS and the ECC ruled against petitioner's entitlement to the death benefits sought after
under PD 626 on the ground she can no longer be considered John's primary beneficiary. As culled from the records, John
and his sister Elizabeth were adopted by their great grandfather, petitioner's grandfather, Cornelio. The decree of adoption
attained finality. Consequently, as argued by the agencies, it is Cornelio who qualifies as John's primary beneficiary, not
petitioner.
Neither, the ECC reasoned, would petitioner qualify as John's secondary beneficiary even if it were proven that Cornelio
has already passed away. The ECC ratiocinated:
Under Article 167 (j) of P.D. 626, as amended, provides that beneficiaries are the "dependent spouse until
he remarries and dependent children, who are the primary beneficiaries. In their absence, the dependent
parents and subject to the restrictions imposed on dependent children, the illegitimate children and
legitimate descendants who are the secondary beneficiaries; Provided; that the dependent acknowledged
natural child shall be considered as a primary beneficiary when there are no other dependent children who
are
qualified
and
eligible
for
monthly
income
benefit."
The dependent parent referred to by the above provision relates to the legitimate parent of the covered
member, as provided for by Rule XV, Section 1 (c) (1) of the Amended Rules on Employees'
Compensation. Petitioner is not considered a legitimate parent of the deceased, having given up the
latter for adoption to Mr. Cornelio C. Colcol. Thus, in effect, the adoption divested her of the status
as the legitimate parent of the deceased.
It appears that Cornelio adopted John in 1985 when the latter was just 2 years old. However, Cornelio died less than 3
years after the adoption decree when John was still a minor.
ISSUE:
Whether or not petitioner is entitled to the death benefits claim in view of John's work-related demise.
HELD:
Yes. The term "parents" in the phrase "dependent parents" in Article 167 (j) of the Labor Code is used and ought to be
taken in its general sense and cannot be unduly limited to "legitimate parents" as what the ECC did. The phrase
"dependent parents" should, therefore, include all parents, whether legitimate or illegitimate and whether by nature or by
60

adoption. When the law does not distinguish, one should not distinguish. Plainly, "dependent parents" are parents, whether
legitimate or illegitimate, biological or by adoption, who are in need of support or assistance.
Moreover, the same Article 167 (j), as couched, clearly shows that Congress did not intend to limit the phrase "dependent
parents" to solely legitimate parents. At the risk of being repetitive, Article 167 provides that "in their absence, the
dependent parents and subject to the restrictions imposed on dependent children, the illegitimate children and legitimate
descendants who are secondary beneficiaries." Had the lawmakers contemplated "dependent parents" to mean legitimate
parents, then it would have simply said descendants and not "legitimate descendants." The manner by which the provision
in question was crafted undeniably show that the phrase "dependent parents" was intended to cover all parents legitimate, illegitimate or parents by nature or adoption.
Political law issue: Equal Protection. There is no compelling reasonable basis to discriminate against illegitimate parents.
The rule promulgated by the ECC that limits the claim of benefits to the legitimate parents miserably failed the test of
reasonableness since the classification is not germane to the law.
GAMOGAMO V. PNOC SHIPPING AND TRANSPORT
G.R. NO. 141707
MAY 7, 2002
CJ. DAVIDE
COMPUTATION OF RETIREMENT BENEFITS; TOTALIZATION OF SERVICE CREDITS
FACTS:
On 23 January 1963, Petitioner Gamogamo was first employed with the DOH as Dental Aide. On 22 February 1967, he
was promoted to the position of Dentist 1. He remained employed at the DOH for fourteen years until he resigned on 2
November 1977.
On 9 November 1977, petitioner was hired as company dentist by LUSTEVECO, a private domestic corporation.
Subsequently, respondent PNOC Shipping and Transport Corporation acquired and took over the shipping business of
LUSTEVECO, and on 1 August 1979, petitioner was among those who opted to be absorbed by the Respondent. In a
letter dated 1 August 1979, Respondent assumed without interruption petitioners service credits with LUSTEVECO.
On 10 June 1993, then President Fidel V. Ramos issued a memorandum approving the privatization of PNOC subsidiaries,
including Respondent. Eventually, petitioner retired after serving the Respondent and LUSTEVECO for 17 years and 4
months upon reaching his 60th birthday, on 1 April 1995. He received a retirement pay of P512,524.15, which is
equivalent to one month pay for every year of service and other benefits.
Petitioner filed a complaint at the NLRC for the full payment of his retirement benefits and argued that his service with
the DOH should have been included in the computation of his years of service. Hence, with an accumulated service of 32
years he should have been paid a two-month pay for every year of service per the retirement plan and thus should have
received at least P1,833,920.00.
The Labor Arbiter dismissed petitioners complaint. On appeal, however, the NLRC reversed the decision of the Labor
Arbiter. Unsatisfied with the reversal, Respondent filed with the Court of Appeals a special civil action for certiorari. The
Court of Appeals set aside the NLRC judgment.
Petitioner maintains that his government service with the DOH should be recognized and tacked in to his length of service
with Respondent because LUSTEVECO, which was later bought by Respondent, and Respondent itself, were
government-owned and controlled corporations and were, therefore, under the Civil Service Law. Prior to the separation
of Respondent from the Civil Service by virtue of the 1987 Constitution, petitioners length of service was considered
continuous.
Petitioner asserts that with the tacking in of his 14 years of service with the DOH to his 17 years and 4 months service
with LUSTEVECO and Respondent, he had 31 years and 4 months creditable service as basis for the computation of his
retirement benefits. Petitioner likewise asserts that the principle of tacking is anchored on Republic Act No. 7699
(Limited Portability).
Respondent asserts that R.A. No. 7699 is not applicable. Under this law an employee who has worked in both the private
and public sectors and has been covered by both the GSIS and the SSS, shall have his creditable services or contributions
in both Systems credited to his service or contribution record in each of the Systems, which shall be summed up for
purposes of old age, disability, survivorship and other benefits in case the covered member does not qualify for such
benefits in either or both Systems without the totalization.
ISSUE:
61

Whether petitioners service with the DOH should be included in the computation of his retirement benefits.
HELD:
No. Retirement results from a voluntary agreement between the employer and the employee whereby the latter after
reaching a certain age agrees to sever his employment with the former.
In this case, petitioner was absorbed by Respondent from LUSTEVECO on 1 August 1979. Ordinarily, his creditable
service shall be reckoned from such date. Since Respondent took over the shipping business of LUSTEVECO and agreed
to assume without interruption all the service credits of petitioner with LUSTEVECO, petitioners creditable service must
start from 9 November 1977 when he started working with LUSTEVECO until his day of retirement on 1 April 1995.
Thus, petitioners creditable service is 17.3333 years.
We cannot uphold petitioners contention that his fourteen years of service with the DOH should be considered because
his last two employers were government-owned and controlled corporations, and fall under the Civil Service Law.
Respondent and LUSTEVECO are government-owned and controlled corporations with no original charters; hence they
are not under the Civil Service Law.
Petitioners contention that the principle of tacking of creditable service is mandated by Republic Act No. 7699 (Limited
Portability) is baseless. Totalization of service credits is only resorted to when the retiree does not qualify for benefits in
either or both of the Systems. Here, petitioner is qualified to receive benefits granted by the GSIS. In any case,
petitioners fourteen years of service with the DOH may not remain uncompensated because it may be recognized by the
GSIS. Since petitioner may be entitled to some benefits from the GSIS, he cannot avail of the benefits under R.A. No.
7699.

62

Вам также может понравиться